Jump to content

Wikipedia:Reference desk/Humanities

From Wikipedia, the free encyclopedia

This is an old revision of this page, as edited by 212.183.128.45 (talk) at 23:00, 23 September 2011 (→‎unanswered questions). The present address (URL) is a permanent link to this revision, which may differ significantly from the current revision.

Welcome to the humanities section
of the Wikipedia reference desk.
Select a section:
Want a faster answer?

Main page: Help searching Wikipedia

   

How can I get my question answered?

  • Select the section of the desk that best fits the general topic of your question (see the navigation column to the right).
  • Post your question to only one section, providing a short header that gives the topic of your question.
  • Type '~~~~' (that is, four tilde characters) at the end – this signs and dates your contribution so we know who wrote what and when.
  • Don't post personal contact information – it will be removed. Any answers will be provided here.
  • Please be as specific as possible, and include all relevant context – the usefulness of answers may depend on the context.
  • Note:
    • We don't answer (and may remove) questions that require medical diagnosis or legal advice.
    • We don't answer requests for opinions, predictions or debate.
    • We don't do your homework for you, though we'll help you past the stuck point.
    • We don't conduct original research or provide a free source of ideas, but we'll help you find information you need.



How do I answer a question?

Main page: Wikipedia:Reference desk/Guidelines

  • The best answers address the question directly, and back up facts with wikilinks and links to sources. Do not edit others' comments and do not give any medical or legal advice.
See also:


September 18

Prime Minister of Canada Jean Chretien and Middle East

I remember that Jean Chretien went to Middle East like UAE when he was the Prime Minister. What was the reason of the visit to the Middle East? — Preceding unsigned comment added by 65.92.152.96 (talk) 02:42, 18 September 2011 (UTC)[reply]

After some quick googling, apparently the reason for the visit is to expand Canada's trading partners and soothe tensions in the Middle East (unsuccessfully). Royor (talk) 11:30, 18 September 2011 (UTC)[reply]

Turkic people diaspora

I notice that Turks and Azeris have their diaspora page but what about Kazakhs, Uzbeks, Kyrgyz and Turkmens? — Preceding unsigned comment added by 65.92.152.96 (talk) 02:45, 18 September 2011 (UTC)[reply]

That's more for Help Desk. If you want, you can make the pages about their diasporas. :) You will need an account to create pages though I'm afraid. Sir William Matthew Flinders Petrie | Say Shalom! 02:47, 18 September 2011 (UTC)[reply]
See Category:Diasporas.
Wavelength (talk) 19:22, 18 September 2011 (UTC)[reply]

Orillion Bastion

Does anyone know what an "Orillion Bastion" is? Aside from them being around in the 16th century I can't find any info on what distinguishes an Orillion Bastion from any other type of Bastion.©Geni 03:18, 18 September 2011 (UTC)[reply]

They were bastions with ears meltBanana 03:38, 18 September 2011 (UTC)[reply]

VC by balloting?

According to Ernest Herbert Pitcher, he was awarded the Victoria Cross "by balloting". The sole reference states "P.O. Pitcher was selected by the crew of a gun of one of H.M. Ships to receive the Victoria Cross under Rule 13 of the Royal Warrant dated 29th January, 1856." Does this mean his crew mates got to pick him for the highest award in the British military? This seems to imply that the powers that be figured one was enough for the entire crew. What gives? Clarityfiend (talk) 04:39, 18 September 2011 (UTC)[reply]

Duuh, never mind. It's described in the Victoria Cross article. Clarityfiend (talk) 04:53, 18 September 2011 (UTC)[reply]

Literacy rates

Almost every human being can speak a language fluently, but in developing countries, only a much smaller percentage knows how to read and write. It seems that one way to instantly make the illiteracy rate 0 is to invent an alphabet and spelling system such that there's a one-to-one correspondence between a word's pronunciation and its spelling.

Why is this not being done? Also, why are so few languages like this, despite the obvious benefits of such a system? --140.180.16.144 (talk) 05:14, 18 September 2011 (UTC)[reply]

likely because there are so many possible sounds it would be impossible. Hot Stop talk-contribs 05:42, 18 September 2011 (UTC)[reply]
Interesting theory. A couple of points. Such languages have sort of been "invented". In fact, most languages are better than English in this regard. Languages like Malay and Indonesian, where the application of the western alphabet to the sounds is relatively recent, behave very consistently. Trouble is, there are many factors that contribute to literacy. A person has to want to read, and has to be given the opportunity to learn. With television rather than the print media being the common source of much information today, the motivation to read is lower. HiLo48 (talk) 05:48, 18 September 2011 (UTC)[reply]
As an aside, the IPA has one sound per symbol. Hot Stop talk-contribs 05:50, 18 September 2011 (UTC)[reply]
Hangul was created for exactly the purpose the OP posited. It's a phonetic alphabet invented because Chinese ideograms were very difficult to learn for ordinary Koreans back in the 15th century. It makes you realize that the Latin alphabet, which is another phonetic alphabet, is already quite adequate for most purposes.
Phonetic alphabets are the simplest you can get if you want a one-to-one correspondence with sounds and symbols, and it's already very widespread. Yet it doesn't affect literacy rates in say, Africa. The answer is not because the Latin alphabet is complex, but because there simply aren't enough educational systems to teach it to people in the first place.
And for what it's worth, the apparent disparity between written and spoken language (words pronounced differently from how they are written) particularly in English and French, does not affect other languages like you think it does. It's the result of spoken languages evolving faster than written languages. Usually, the longer a language has been associated with a particular writing system, the larger the disparity. In languages that just recently adopted Latin script, it's not a problem.
And a side-note: comparable to Bahasa Malaysia and Bahasa Indonesia, our own national language - Filipino (which is really just Tagalog despite protestations of legislators otherwise), was itself an artificial construct. Unlike those languages, however, Filipino languages lost the native rudimentary writing systems (Baybayin) very early in the Philippine colonial history. Latin script came with the Spanish in the 16th century and has remained the only form of writing for majority of the islands (a notable exception are the predominantly Muslim autonomous regions in Southern and Western Mindanao which use Arabic scripts). When Filipino was first proposed, it was not to make it easier for ordinary people to learn to read and write, it was purely for nationalistic reasons. A means of pulling together a very young nation fresh from the clutches of two colonial powers. Legislators retained the Latin alphabet (whew), but culled consonants they considered 'foreign' - f, z, c, etc. They also recommended phonetically spelling foreign loanwords such that 'Airplane' became 'Erpleyn'. The goal was to erase traces of European influence as much as possible. The result was not an increase in literacy (it was already quite high in the first place, from Spanish education systems reinforced by American Thomasites), but an increase instead in the propensity of people to misspell and mispronounce foreign words. The only thing they accomplished was make the language cruder.-- Obsidin Soul 06:45, 18 September 2011 (UTC)[reply]
If English adopted phonetic spelling the written language would be hideously complex to learn - not only would one need many extra symbols, but most words would have to be spelt differently in different places to account for the different pronunciations in different accents. DuncanHill (talk) 09:19, 18 September 2011 (UTC)[reply]
Isn't Turkish like that? By which I mean, latin alphabet with all sorts of phonetic things. Sir William Matthew Flinders Petrie | Say Shalom! 19:14, 18 September 2011 (UTC)[reply]
Vietnamese is a lot worse than Turkish in that regard. -- Jack of Oz [your turn] 20:01, 18 September 2011 (UTC)[reply]

I doubt Vietnamese language is worse. Speaking as a person that fluent in English (not like master it but i can speak, write, read pretty well. Good enough for typical conversation) and Vietnamese. Vietnamese has exactly same alphabet as English except we don't use z and f. We added some accents, like those little marks above or under the words to make new words. Vietnamese is a easiest language to learn, it's not because i learn it first so i think it's easy. Every word has only 1 syllable. There is no past tense or anything like that. Example i can say i run today, i run yesterday, i run before. Not like most languages there are past tense, perfect tense... (all kinds of tenses) Words always stay the same as they are. They never change to different words. There is no exception, every words follow the rules. The word system we used today was developed in 17th century by a french guy. Before that we use Chinese characters. So since it was created recently, so people tried to make it super easy and organize unlike English as an example with thousands of years so there is no actual organize. In Vietnam, most kids would know how to spell and write all the words in about 3rd or 5th grade. You don't learn new vocab at school because there aren't any. There is no spelling bee because everyone knows how to spell every words. People know how to spell and write ALL the words but it doesn't mean they understand what they mean. Vietnamese people use 2 old words combined them together to get a new word. Example a word "cat" stand alone mean an animal and "finger" stand alone mean an organ in your hand. They combined together mean something else(i made up this example). So basically if there are new words you never see before in Vietnamese, they will be the combination of 2 old words you have seen before. So as the conclusion it's the easiest language to learn but the hardest part is the pronunciation. People just can't pronounce the words correctly because their tongues were not fit to it.Trongphu (talk) 02:19, 19 September 2011 (UTC)[reply]

I don't dispute any of that. My remark was purely in response to the "Latin alphabet with all sorts of phonetic things" Sir William referred to. There are clearly more diacritics per average word in Vietnamese than in Turkish. -- Jack of Oz [your turn] 02:57, 19 September 2011 (UTC)[reply]
(OP here) Interesting. So at least two languages--Hangul and Vietnamese--were invented because Chinese characters were too difficult to learn. It's obvious that Chinese itself could be replaced by a phonetic system--in fact, pinyin can represent the pronunciation of every valid character, plus a few sounds that don't correspond to any characters. I could write in pinyin when I was 6, as could almost every other Chinese child, but couldn't read or write in actual characters until much later. I'm pretty sure that any foreigner could learn pinyin in two days, so teaching a Chinese child how to read and write would be a trivial task if it were to replace the character system.
However, Chinese is tonal, which makes it very easy to use a tiny alphabet plus some accents to represent every sound. English and the Romance languages are not, so I wonder whether the same is true for these languages, or whether any phonetic alphabet would be too cumbersome to be useful. --140.180.16.144 (talk) 03:59, 19 September 2011 (UTC)[reply]
One writing system, a fairly recent invention (compared to most writing systems) which does match the spoken language rather closely is the Cherokee syllabary. Most languages writing systems were put into place so long ago that the natural and gradual changes which occur in spoken language aren't picked up by the writing system. English, for example, underwent the Great Vowel Shift during the 14th-15th century; English writings from the 1200s are understandable by modern speakers, but the language would have been almost impossible to understand to modern ears. Most languages have undergone similar changes over time to the way they are spoken, but the orthography (writing system) often doesn't keep up; the result is a situtation where what may have started out as a consistent system of writing, where the sounds matched the letters on the paper, drifts and changes to the point where the writing system no longer has a consistent one-to-one correspondence that it once did. --Jayron32 04:45, 19 September 2011 (UTC)[reply]
Ah, Jayron beat me again, I was going to mention the Cherokee syllabary. From what I've read it was fairly easy for the Cherokee of the time (early 1800s) to get the hang of, even for adults who had been illiterate all their lives. Some sources on this: [1]; [2] "within fourteen years of its introduction, and seven years of the first printing, more than half of all households in the Cherokee Nation had a reader of Cherokee"; [3], "Cherokees began learning the syllabrary 'almost overnight', and its use became widespread...almost everyone learned to read and write in their native language..." Of course the rapid adoption was not just because it was relatively easy and phonetic, but also due to the efforts of missionaries promoting it and the publishing of Cherokee Phoenix newspaper. Still, the tale of a people who had been almost totally illiterate (and even somewhat hostile to the idea of reading and writing) gaining significant levels of literacy within a decade or two is moving. Pfly (talk) 09:48, 19 September 2011 (UTC)[reply]
Also, even in just the United States there are regional differences in pronunciations. It's not uncommon, for example, to hear something like "I axed you for a pin". Pfly (talk) 09:52, 19 September 2011 (UTC)[reply]

Why are numbers written in the five (5) format?

This has been perplexing me for years: on most U.S. official documents, numbers are first spelled out and then written using a numeral in a bracket-e.g. five(5). An example of this would be File:Anthraxreward.jpg, but also school report cards, government forms and the like. Why? I'd presume anyone with sufficient literacy and language ability to read the document would also know how to count to ten. For whose benefit is the numeral included? Thanks, Puchiko (Talk-email) 09:56, 18 September 2011 (UTC)[reply]

Simply to avoid confusion. Writing out amounts makes something more legible. Redundancy also makes it easier to spot errors. e.g. Five thousand (50000) < you will immediately know that the extra 0 is most likely a typo. The same reason why you write out numbers on checks. You wouldn't want to be paying $50,000 on a $5000 item just because you were scatterbrained that day and wrote an extra zero in.-- Obsidin Soul 10:11, 18 September 2011 (UTC)[reply]
That reminds me. In my local bank, we have to fill in the withdrawal slips ourselves (because billions and billions of bailout money just isn't enough). I went to the bank the other day and tried to draw out a very small amount - less than you can get from an ATM, which is why I went into the bank itself. The girl at the window proceeded to count out £400 for me. I only wanted £4:00....... --KägeTorä - (影虎) (TALK) 17:51, 18 September 2011 (UTC)[reply]
IIRC, it's usually the other way around, e.g. 5 (five). This is to stop numbers being altered by, for example, people adding zeroes, and so on. I've never seen it done like 'five (5)'. --KägeTorä - (影虎) (TALK) 13:13, 18 September 2011 (UTC)[reply]
lol @ Yackmoore Phone Company. Bus stop (talk) 13:20, 18 September 2011 (UTC)[reply]
I've never seen the "5 (five)" example; all my experience has been "five (5)." See the Apple One (1) Year Limited Warranty for example. Nyttend (talk) 19:08, 18 September 2011 (UTC)[reply]
One place I've seen them is on medicine, for example "three (3) doses daily"; I've always assumed that's so if one is obscured the right amount is still taken. Grandiose (me, talk, contribs) 20:37, 18 September 2011 (UTC)[reply]
And for a hand-written prescription, hopefully one or the other will be legible, despite the doctor's best effort at poor penmanship. StuRat (talk) 20:42, 18 September 2011 (UTC)[reply]

In Final Score and similar programmes, when football (soccer) teams score seven goals, it's usually rendered "7 (seven)". This is because the 7 looks similar to a 1 and the latter is common and the former very unusual, increasing the chances of a mistake. --Dweller (talk) 10:21, 19 September 2011 (UTC)[reply]

Wow, thanks for all the answers :) I didn't expect so many. Puchiko (Talk-email) 13:32, 21 September 2011 (UTC)[reply]

New ways to make money for those in financial straits? (Disability = hard to employ)

So I tried unconventional ways to earn money; turns out, I can't donate blood/plasma if I've been to Germany, even though I haven't been there for 21 years. (No signs of CJD after 21 years = very likely no CJD at all. Why can't the FDA acknowledge that?)

There is no sperm bank in 66502.
There is no skin bank in 66502.
I haven't seen a consignment secondhand goods store around town.

Disability + no work since 2009 = hardly a chance to get a job. (Thanks to the economy, the ratio of job-seekers to openings is so incredibly out of proportion, that even "not being employed for over 2 years" is enough of an excuse not to hire me, because even if they whittle down the criteria to "only consider those who currently have another job as of application date," there's still too many of them to interview.

So are there some un-common (legal) ways to get a better living? Paying off a private student loan with a federal student loan may be better interest-wise, but that's still paying off debt with debt. (Like "fighting fire with fire?")

More like "Borrowing from Peter to pay Paul". StuRat (talk) 21:26, 18 September 2011 (UTC) [reply]
Pharmacological testing a is traditional source of income, though if you're not healthy you may have problems being accepted.
Gold farming is making money by earning items and experience on online role-playing games. --Colapeninsula (talk) 14:17, 20 September 2011 (UTC)[reply]
You may be able to make a small living, or at least get some interesting stuff, by searching for online competitions and free product samples and applying for lots of stuff. There are specialist websites that will help. --Colapeninsula (talk) 14:17, 20 September 2011 (UTC)[reply]

Youtube for $?

I read that some make a living off of posting to YouTube. However, what are the odds of getting that to happen to me? How do I increase said odds? Where can I take (free) courses on how to edit videos so they all can look more professional and appealing to a wider audience? (What websites can I learn this, if no physical locations?)

What other great websites that I might not even know too much about, could I get paid to freelance, etc.?

There could be freelance writing; I can do that. What websites offer this though?

Can there be a selling venue of common intellectual property? (like drawings on MS Paint, or more stories, etc. to write?)

(Lastly, as for auctions, eBay/FeePay has gotten too expensive to sell there; they only seem to care about the big-wig sellers, not the everyday ma-and-pas. Is there a popular auction site that will cater to people like us? Thanks.) --70.179.163.168 (talk) 19:54, 18 September 2011 (UTC)[reply]

Freelancing web sites include http://elance.com, http://freelancer.com, http://guru.com, http://odesk.com, and http://vworker.com. 70.91.171.54 (talk) 20:46, 18 September 2011 (UTC)[reply]
For selling many cheap things, like under $10 each, you may do better to sell them locally, as the shipping costs eat up any profit, otherwise. Perhaps a garage sale or swap meet ? (Does your disability allow you to leave the house ?) StuRat (talk) 20:48, 18 September 2011 (UTC)[reply]
For writing, you might want to do something like proofreading student's papers for them before they turn them in, so they can fix all their mistakes. StuRat (talk) 20:56, 18 September 2011 (UTC)[reply]
You could also find ways of dealing with this disability, and go for a 'normal' job. Quest09 (talk) 00:51, 19 September 2011 (UTC)[reply]
63336 employs people to find answers to questions; you work at home on your computer. Probably there are other organisations. --Colapeninsula (talk) 14:26, 20 September 2011 (UTC)[reply]

Buttons, buttons.

As a female blond, I should know this - but I don't. It shows you how dumb I am. Why are the buttons on a female's blouse on the left side and on a man's shirt on the right side?--Christie the puppy lover (talk) 20:32, 18 September 2011 (UTC)[reply]

Some people say it's because women didn't carry swords or other weapons - for men, the sword (usually carried in a scabbard on the left, and drawn with the right hand) would catch on the 'lip' of the shirt if it buttoned the other way. Others say it's because in the old days, women had maids who would dress them, so it was easier for the maid if the buttons were on the other side. No-one actually knows. --KägeTorä - (影虎) (TALK) 20:44, 18 September 2011 (UTC)[reply]
(ec with update to KageTora's answer) Or because women who wore buttoned clothes used to be dressed by maids. Or nobody really knows. AndrewWTaylor (talk) 20:51, 18 September 2011 (UTC)[reply]
My understanding is that it has more to do with jackets than shirts. If you want to reach into your jacket and draw your sword, you need the left hand side of the jacket to be over the right hand side. The shirt then just buttons the same way as the jacket. --Tango (talk) 21:59, 18 September 2011 (UTC)[reply]
That is a possibility. The thing is, swords were usually worn outside everything, including jackets. I think it would be very uncomfortable to be wearing a jacket over your sword. A long winter coat, maybe, but only if it's open, defeating the purpose of where the buttons go. "Sir, please be a gentleman and wait while I unbutton my jacket so I can get my sword out to parry your wiley surprise attack, what!". :-) --KägeTorä - (影虎) (TALK) 22:27, 18 September 2011 (UTC)[reply]
While those might explain the origin, the reason it remains this way is that we feel the need to have gender-specific clothing, the classic example being pink and blue clothes for baby boys and girls. If our society valued unisex clothing, then our clothes would all button on the same side (or both button on either side). StuRat (talk) 20:54, 18 September 2011 (UTC)[reply]
Ironically, Stu, according to QI, pink was originally for boys, and blue for girls. Everything changed around the beginning of the 20th century, I believe. KägeTorä - (影虎) (TALK) 21:01, 18 September 2011 (UTC)[reply]
So The Blue Boy was a cross-dresser ? :-) StuRat (talk) 21:24, 18 September 2011 (UTC) [reply]
Only in the same way as Pinkie was. ;-) --KägeTorä - (影虎) (TALK) 21:41, 18 September 2011 (UTC)[reply]

Some detailed discussion of the topic here. The answer seems to be that nobody knows. Alansplodge (talk) 22:28, 18 September 2011 (UTC)[reply]

Thank you boys and girls for clearing that up. I guess I am not as dumb as I thought I was.--Christie the puppy lover (talk) 22:39, 18 September 2011 (UTC)[reply]

Why doesn't capitalism work?

The protesters in New York City recently are asking a question I'd like to know the answer to myself: why doesn't capitalism work? And I'll add, when did it stop working? For example:

1) Supposedly gas prices are the outcome of supply and demand. So why did the fall of oil prices from $100 a barrel to $82 a barrel produce no relief in prices? Why do governments which have taken active steps to manipulate gas prices not experience shortages, but instead the companies simply keep selling at the lower rate? (For example Honduras exercised a contract clause to take over oil storage tanks in 2007 and announced a 42-cent price decrease, [4] but the only news I see about gas shortages came with the ensuing coup against Manuel Zelaya in 2009)

2) Supposedly labor is a good traded on the market. So why can't the unemployed simply lower their price and get back to work at any time?

3) Supposedly profit encourages businesses to expand and make more profit. So why is it that businesses in the U.S. have been making record profits, but do nothing to expand and hire people?

4) In Republican fantasy, when they explain why millionaires must continue to pay lower tax rates than the middle class, they say that this is necessary because they are "job creators". But where are the jobs? And I've heard that small businesses lead employment recovery - doesn't that mean the middle class is the job creator? Why can't the middle class create jobs?

Was capitalist theory always this irrelevant to reality? If not, what changed to make it that way? Wnt (talk) 20:36, 18 September 2011 (UTC)[reply]

this answer is funded in part by paragraph breaks supplied by stu rat
It is simply a mistake to say that "capitalism doesn't work". Capitalism is about using resources that are worth something and owned by someone using them to create value that did not exist before the person or entity using that resource created that value. In other words, capitalism is not prescriptive, it is descriptive of what happens when you have de facto or legally recognized private property.
There is no way to allow private property and free selling and buying without resulting in capitalism. It is a simple fact of nature that if people can have private property, they can create value, and the minute you let them buy and sell it freely, you have people using resources of value to create more value -- bam, you have capitalism. The only way to eliminate capitalism is to make it illegal for people to own private property.
Even this does not work: people will continue to own themselves; they (or their families) can invest, if nothing else, time, into their education, thereby making their personal resource worth more. They thereby create value. Think of the city you're in: would you be worth any more if you learned French fluently and could teach it to people who didn't know it yet? Of course you would be worth more. You just used resources (time and maybe money) to create value. Unfortunately for communist countries, this, and only a few other ways of creating value, were "allowed", and the state had to steal every other means of production.
Here's an answer to some of your questions:
"Supposedly labor is a good traded on the market. So why can't the unemployed simply lower their price and get back to work at any time?"
At what price would you take French lessons from a Persian Cat who grew up in Paris? Is it twenty cents an hour? Would you take two hours of lessons from that French cat for 40 cents? You would not. The cat is not able to create 40 cents worth of value by giving French lessons. Likewise, the unemployed might not be able to create even $1 of value per hour. If they or their families invested time and money into their education and increasing their skills, this value would increase.
Under communism, where every other means of production must be stolen by the state, this is the only acceptable means for increasing a person's value. In a free country, a person can increase his value in a multitude of other ways: such as by dressing respectably and being hired due to looking presentable (under communism private clothes are illegal, and the state must determine and supply all clothing: you can't just be having object fetishism willy nilly, which is what good clothes sold at a good shop front would be).
Or you could getting a van and open a business using it on it (you would at a minimum need some special license I guess under communism, you can't just buy and sell whatever service you want just by registering a company -- after all, what if you employed someone? Only the state may employ someone, since anything else is slavery), etc etc.
Or you could get a printing press and print nice posters and make photocopies etc. But not under communism, where you can't own a private press and buy and sell a good freely.
The key thing about capitalism is freedom: you have to actively suppress a person's right to buy and sell property and services, including their labor, to prevent capitalism. Anything else is capitalism by definition automatically.
"Supposedly profit encourages businesses to expand and make more profit. So why is it that businesses in the U.S. have been making record profits, but do nothing to expand and hire people"
Businesses can do whatever they like. Why would a family with two children who are in their twenties and rich lawyers not want to produce a third child to eventually make a lot of money, if the parents are still fertile? Just because you are doing well does not mean you want to expand operations.
"Was capitalist theory always this irrelevant to reality? If not, what changed to make it that way?"
In fact, you don't have to believe "capitalist theory" for capitalist facts to be true. 82.234.207.120 (talk) 20:52, 18 September 2011 (UTC)[reply]
Have a few paragraph breaks, on me: ,,. :-) StuRat (talk) 21:17, 18 September 2011 (UTC) [reply]
Thanks for the resource.
Also, to put a real monkey wrench in the thing. Even though if you have ANY private property and right to buy and sell property, you automatically have capitalism, in my opinion it is completely obvious that you have to take SOME of these rights away if you want anything like a well-functioning society or progress.
That's because why doesn't someone have the right to come up to you as you're setting up that copier or poster printer I mentioned above and say, "Look, kid, you're attracted by the $40/poster market rate, aren't you. You figure you can get in for $2000 worth of machinery and break even in a year.
Well, thing is, we would like to keep these $40 and actually us producers are moving up to $50 two months from now. But we can only do that if everyone is in. How's about I pay you $2000 right now to keep hush hush and follow our pricing, welcome to our trust." You'll take it, won't you, after all he has just removed all your risk or you can repay your loan or backer. But this reasoning works for everything from sugar to oil to metal.
If we allowed people this "freedom", we never would have computers, as potentially every single input into that would have been priced out of commodity prices and into unattainable land. So, it's pretty obvious to me that you can't just allow someone to form a trust and create a monopoly -- we played this game in the 19th century, and everyone lost out. So, even though ANY amount of freedom to own goods and buy and sell goods and services instantly results in capitalism, it's obvious that you have to remove at least parts of these freedoms if you want a society worth living in. That's what the question almost always revolves around -- how much of these freedoms to remove. 82.234.207.120 (talk) 23:39, 18 September 2011 (UTC)[reply]
I numbered your questions so I can respond in kind:
1) Price fixing can cause prices to be higher than competition would produce. If so, then government limits on prices would work, until the profit margin was so low that the companies could make more money by investing elsewhere.
2) Minimum wage doesn't allow this, and having a social safety net means people might prefer to live on welfare rather than take such low-paying jobs. Also, the large number of illegal immigrants means the low-paying jobs are already taken.
3) They've been expanding overseas. Also, expanding only makes sense where you have an expanding market or market share.
4) Whether small business owners are middle-class or rich depends on the politician talking and which way the wind is blowing. And giving money to rich people doesn't help the economy nearly as much as it does to give it to the poor. The reason is that the poor tend to spend all the money they get immediately, and locally, thus helping the local economy. StuRat (talk) 21:05, 18 September 2011 (UTC)[reply]
Lack of competition from consolidation and de facto collusion troubles the petroleum markets, but there is nothing antitrust regulators can do. It's congress's job to impose more effective taxes in the absence of clear evidence of conspiracy in the face of record profits and rising gas prices on declining oil prices. But that brings us to the real problem, which is the lack of public campaign financing which makes congresspeople beholden to contributors from large companies and the rich likely invested in them. Several major reforms (health care, tax, renewable energy, sentencing, patent, etc.) all are heavily inhibited by campaign contributions. 70.91.171.54 (talk) 21:07, 18 September 2011 (UTC)[reply]


(edit conflict)In general people are indifferent. The answers to your questions are amazingly pragmatic, i.e. completely emotionless.
  • I don't know enough about this topic, so sorry
  • Price ceilings and price floors are impediments to free trade, so it is said. Even if a worker wished to trade his labor for money at less thean minimum wage it would be illegal, as far as I understand
  • The business I am employed by is making "profit." Is accounting the same as reality? The company's debts are owed to the personal funds of the owner.
  • The jobs are in a trust fund to be doled out when there is trust in funds.
Lastly, government run by man has always always always failed. "Their" way is deficient; so is yours. Schyler (exquirere bonum ipsum) 21:13, 18 September 2011 (UTC)[reply]
"Capitalism," such that it is, works just fine. It still keeps a small group of people who control the means of production in control (this is of course a very simplistic explanation). Your fourth question is the most telling in this regard, the middle class can't create jobs because they don't control the means to produce much of anything. Capitalism isn't intended to be good for everyone and in my opinion is actually bad for virtually everyone, but that's a different question. --Daniel 22:07, 18 September 2011 (UTC)[reply]
I don't buy the minimum wage explanation. If that were the explanation, the occupations with high unemployment would be those already paying minimum wage, but those with higher wages would see a free market wage drop and an increase in the number of people employed. I don't see an indication that this is what happened. Wnt (talk) 00:07, 19 September 2011 (UTC)[reply]
Yes, the assumption that employers would all hire more workers if they were cheaper (or an infinite number if they were free) is faulty. First off, the wages are only a small portion of the total cost incurred by an employee. There are also benefits, administrative costs, managers needed to manage them, risk of lawsuits, supplies, training, etc.
And more employees doesn't necessarily make your company more productive. It rather depends on the type of work being done. For physical labor, more hands is probably helpful, but not so with mental work. Hiring two computer programmers to write the same program isn't likely to get it done in half the time, it may even take longer.
Finally, the assumption that companies want to be more productive is not always true, either. This is only the case if they have more demand for their product or service than they can meet with their current staff. Let's say the employer is a newspaper. If they hire more columnists, are they going to make the paper thicker ? A newspaper with twice as many columnists isn't likely to be worth twice as much to customers, so such a decision might not be in the best interest of the financial health of the company. StuRat (talk) 04:47, 19 September 2011 (UTC)[reply]
Capitalism is the very worst economic system ... with the exception of every other system that has ever been tried. -- Mwalcoff (talk) 23:51, 18 September 2011 (UTC)[reply]

When you say protestors do you mean the Israelis protesting in Time Square? I haven't heard of any other protests recently (King Michael is good about keeping those in check). Sir William Matthew Flinders Petrie | Say Shalom! 23:46, 18 September 2011 (UTC)[reply]

Saturday "Anonymous" protest against Wall Stret, bankers, and plutocratic greed: Occupy Wall Street... AnonMoos (talk) 00:17, 19 September 2011 (UTC)[reply]
My buddy Nolan explained to me about these guys, and how they say they were inspired by the Tahrir Square guys, but the motivations and behaviours are actually hand in hand with those of the Israeli protests, but considerably less effective given how few there are. He then lamented the fact that most Americans are too lazy and apathetic to join, etc. and stated his belief that the protestors should riot (which the NYPD are really huge fans of btw; their nightsticks are even bigger fans) I'm not sure we have seen an actual failure of capitalism as well to be honest, but my mum is the one with a BA in Econ (and a JD in ERISA :p). We are just seeing the perfect storm of economic cock-ups. It might mean that it takes longer to recover, but as far as I know, there will be recovery (as part of the holy boom-bust cycle). Sir William Matthew Flinders Petrie | Say Shalom! 21:06, 19 September 2011 (UTC)[reply]
Capitalism was designed to not work. It is proven every 80 years, and has been written above in terms of linen and coats. →Στc. 01:30, 20 September 2011 (UTC)[reply]
Well, it depends on what we mean by work here. If we mean it's meant to enrich people, then it does (a small number of people though). Then again, I guess that my family and I count as owners of the means of production so it works in our eyes (but not, say, for my buddy Nolan who has to work in retail even though he is well-educated and very clever; or my genius buddy Michael who cannot attend college as he is too poor and not stupid enough to take out college loans). Hmmm, wait, that doesn't work well at all. =/ Given that this whole topic invites people to give opinions, I think it's a mix of capitalism and socialism that helps to support the less wealthy while still encouraging people to compete and make money (with lots of nice regulation on private enterprise to ensure there are no companies that get too powerful, raise prices too high or sacrifice quality). How do you define working in this case my good Sigma? Sir William Matthew Flinders Petrie | Say Shalom! 01:41, 20 September 2011 (UTC)[reply]
I have discussed my thoughts in my earlier days on the wiki. The conversations have been archived in two subpages of my talk page. You are free to look at them if you wish, though I do not bandy about my past of inappropriate usage of talk pages as a platform for irrelevant material. →Στc. 01:54, 20 September 2011 (UTC)[reply]

If, by capitalism, you mean laissez faire capitalism, the petroleum industry is the worst of possible examples. It is hugely regulated by the government regarding additives, building refineries, and limits on drilling. It is very highly taxed. A much better example of laissez faire would be the internet, which, at least until recently, has been untaxed and unregulated. If you want to read a standard Austrian school defense of laissez faire capitalism, here is a free pdf of a definitive modern defense from George Reisman's http://www.capitalism.net/. What we have in the modern west are various forms of mixed economies with the government usually controlling somewhere around or more than 50% of the GDP. The US was closest to laissez faire capitalism from the end of the civil war 1865-1890 to the adoption of the first anti-trust legislation, with monopolies and subsides given to railroads in that era being a notable exception. Grover Cleveland was the greatest actual and effective champion of laissez faire. The progressives Teddy Roosevelt and Woodrow Wilson ended the capitalist era in the US with their militarism, trust busting, regulation, bureaucratic racism, the drug war, the institution of the draft and modern central banking with the creation of the Federal Reserve, which caused the boom that lead to the depression. Nixon finally broke all ties with reality by going off the gold standard. Ironically, Communist China is much less regulated and the Communist Party only controls about 15% of the GDP, hence their boom. Were they to eschew militarism and adopt objective law and an independent judiciary to protect the property rights of all individuals they would have the freest society on earth. μηδείς (talk) 01:18, 22 September 2011 (UTC)[reply]

If you're holding up the 1890s as the acme of capitalism in the U.S., then many people would have little desire to return to the societal and business structures of that period. Anyway, I think it can be agreed that in most circumstances (not all) unrestrained capitalism is great at generating economic activity, but not so good at ensuring that such activity doesn't have many overall negative social consequences... AnonMoos (talk) 05:02, 22 September 2011 (UTC)[reply]
The point was that trade was the freest in the US during that period, not that there was something magical about all the other cultural aspects of that time. It's rather odd to imply that a return to free trade would mean resurrecting corsets and reservation wars. As for negative social consequences, I doubt you can point out any that weren't actually caused or worsened by government interference rather than free trade itself. Free trade simply means that--trade which is neither prohibited by nor subsidized by the government. Free trade doesn't mean social conservativism, special favors for big business, Jim Crow, allowing polluters to destroy property without having to pay to clean up the messes they make, the freedom to commit fraud in sales or advertising, or any of the other sorts of things leftists like to tar it with. It just means free trade, the goverment neither stopping people from voluntarily buying and selling nor forcing such buying and selling. The hidden premise behind this thread is the equation of "big business" with capitalism as such, and the implication that because such government-created and regulated monsters as the Oil industry have problems that free trade has failed. Monsters like BP, which got government favors from Britain in regard to Libya and the Lockerbie bomber and from the US which bore the cost of the Gulf oil spill are not the products of free trade but of cronyism at the highest levels. Criticisms of such entities are not criticisms of free trade but of government corruption and incompetence at the highest levels. μηδείς (talk) 16:29, 22 September 2011 (UTC)[reply]
Actually, I had in mind plutocratic "robber barons", monopolistic "trusts", and brazenly-corrupt politics much more than corsets and handlebar moustaches. There was a certain kind of unsavory and exploitative individual such as Charles Yerkes who seemed to flourish in the conditions of the 1890s -- no matter how many times the voters of Chicago unequivocally sent the message at elections that they wanted to get rid of the hated "traction railway" monopolies on public transit (i.e. low-quality shoddy accident-prone level-ground cable cars), it took them well over a decade to make any progress at all, and in the meantime Charles Yerkes retreated unobstructed to Philadelphia with a very large bag of cash... AnonMoos (talk) 18:17, 22 September 2011 (UTC)[reply]
Well, the problem is that terms like robber barons and "monopolistic" trusts are inherently derogatory and rife with POV. Companies like Standard Oil became large by providing a new good at a low price which appealed to all consumers. They had no more "monopolistic" ability to force people to buy their products or need to steal their land with government backing than does (or did) Netflix. Actual monopolies are only created when they government grants and defends such privileges (i.e., private laws) with the threat of legal penalties. Industries like many of the railroads (notorious in California) were granted government subsidies and allowed to seize private land by corrupt officials. Tarring both as robber barons simply because they were big businesses is hardly fair. Government intervention on the side of one business against another or against the public or to further illiberal ends such as racial segregation (note that the racial segregation of passenger rails was forced by government regulation against the wishes of the industry) is the antithesis of free trade. One has to give specific examples of such evils. When you find them you will always find either short-term self-liquidating criminality (Enron--brought to you by Paul Krugman) or stupidity (Netflix), which is rightly punished by the courts or the markets, or, more likely, politically connected corruption at a grand scale such as Trent Lott's obstruction of dealing with the Worldcom/MCI fraud and bankruptcy or the government seizure of GM with money legally owed to bondholders under bankruptcy law funneled to the UAW instead. For real long term robber baronism and abusive monopolies look at the Soviet Union and our man from the KGB, Vladimir Putin. In Russia the capitalists have all been jailed or assassinated. There is plenty of outrage to go around at such corruption, but the ones to blame for institutionalized corruption are the ones accepting the campaign contributions--and who hold the monopoly on jails and guns.μηδείς (talk) 18:07, 23 September 2011 (UTC)[reply]

What is the ESTA good for?

What is the point of asking questions like this: "Have you ever been or are you now involved in espionage or sabotage; or in terrorist activities; or genocide; or between 1933 and 1945 were you involved, in any way, in persecutions associated with Nazi Germany or its allies? " Would someone ever answer yes? Has someone been caught with that silly question? What happen if you answer yes, just as a joke? Besides that, the question is poorly written, I think. If you persecuted Nazis, like Simon Wiesenthal, you should also answer yes. But maybe it's just me being a Grammar Nazi. Quest09 (talk) 22:05, 18 September 2011 (UTC)[reply]

I'm struggling to understand how Wiesenthal persecuted anyone, but perhaps that's just me being a semantics fascist. Orange Suede Sofa (talk) 23:42, 18 September 2011 (UTC)[reply]
Persecution doesn't imply unlawful persecution. If you are trying that a group of people (drug dealers, child molesters, war criminals) get imprisoned, that's persecution too. Quest09 (talk) 23:48, 18 September 2011 (UTC)[reply]
To me (and our article on the topic), mistreatment is an element of persecution. One could argue that someone being tracked down for prosecution is being mistreated (because it is stressful, limits a normal life, etc.) but then society as a whole would be persecuting a pretty wide range of individuals, yet nobody seems to describe it that way (although I suppose even that is up for debate). Orange Suede Sofa (talk) 00:12, 19 September 2011 (UTC)[reply]
Using the word persecution, as a legitimate persecution, is not that rare: "The additional available resources will allow police to focus enforcement on other issues such as persecuting drug dealers (...)" or "They bomb us under the pretext that first, they are eliminating the guerilla forces and second, they are persecuting drug dealers, ..." or "Caribbean nations, including Jamaica, that allow their ships to enter their territorial water persecuting drug dealers, but drug traffic keeps growing (...)" Or even better: Simon Wiesenthal persecuted a single Pole"Quest09 (talk) 00:25, 19 September 2011 (UTC)[reply]
I still cringe when I see that usage, but I concede the point. Orange Suede Sofa (talk) 00:36, 19 September 2011 (UTC)[reply]
Looks to me that ESTA is good at getting the US government $14 for each person who gets it... Googlemeister (talk) 18:56, 19 September 2011 (UTC)[reply]
It's my understanding that once someone has entered the US under ESTA, simply being in one of those categories (that make someone ineligible for "visa free travel") isn't grounds alone for prompt deportation. But lying on the form is. So, supposedly, it streamlines the removal of someone who wasn't eligible but who got in anyway. -- Finlay McWalterTalk 22:22, 18 September 2011 (UTC)[reply]
Nobody really expects that you will answer in a positive manner if you did or are doing any of these things. Their purpose is to give the government a legal reason to strip you of your new citizenship and to expel you from the country (if you're an immigrant) when someone finds that you're guilty of any of these things. An US citizen is obliged by law to inform the government if he is working for any foreign government. With such a law the government has something to prosecute a spy because he obviously failed to obey the law. Flamarande (talk) 22:26, 18 September 2011 (UTC)[reply]
"involved" is an ambiguous word; it can mean either "implicated" or "concerned in some way with". They mean the former, but I'm not sure what they'd say to be clearer; "implicated" or "concerned with the commission of" is perhaps better, but not perfect either. -- Finlay McWalterTalk 22:29, 18 September 2011 (UTC)[reply]
The really complicated part is the "moral turpitude" question. Very few applicants are, our could be argued to be, war criminals or enemy spies. Given the context, you'd think "moral turpitude" means crimes that are also fairly rare and particularly vile; when I first saw it on an I-94/W I (like I imagine most people) thought it meant something akin to "are you a child molester?" But as the Wikipedia article shows, moral turpitude can be a complicated and rather surprising one. Someone with a 40 year old conviction for burglary would (on the face of it) fail, but someone who'd recently been convicted of loan sharking would pass. -- Finlay McWalterTalk 23:09, 18 September 2011 (UTC)[reply]
As a foreigner who has answered that question the OP describes a few times myself, and been in the company of several others when they have answered it, I observe that almost every time it has to be answered it leads to declarations about what a dumb question it is, and how stupid the American government is for asking such a dumb question. Now, to Americans reading this, please don't shoot the messenger here. I'm simply telling you how some foreigners perceive this question. And while you can easily say "We don't care", I suspect that at some level at least some of you do. Maybe the "real" reason (Is it outlined well enough up above yet?) for this odd question could be made clearer to those having to answer it. HiLo48 (talk) 00:25, 19 September 2011 (UTC)[reply]
I also dread entering the US and dealing with annoying forms. But the underlying requirements are not unique to the US. Canada has a very similar set of rules; you just don't have to fill out a form when you cross the border. If a Canadian border guard asked the corresponding Canadian question, would it be dumb for the same reasons outlined above? Is it the use of "moral turpitude," the meaning of which is absolutely opaque to anyone entering the country? To me, it's the combination of both: the form plus the impenetrable wording. The rules themselves are not that big of a deal. Orange Suede Sofa (talk) 02:02, 19 September 2011 (UTC)[reply]
The rules are OK, although overdone at times. It's the dumb questions that are the problem. They lead to mockery and a lessening of respect, where it's the opposite that is presumably desired. HiLo48 (talk) 02:28, 19 September 2011 (UTC)[reply]
As previously stated, the question exists as part of the regime for kicking war criminals out of the country once they're already in the country, not for keeping them out of the country to begin with. For example, the granting of citizenship is generally irrevocable -- unless the government can prove that the person lied to get into the country or obtain citizenship. This is how John Demjanjuk had his citizenship revoked. When he applied to immigrate in 1951, he didn't mention being a death-camp guard. -- Mwalcoff (talk) 03:07, 19 September 2011 (UTC)[reply]
Also as previously stated, the question makes the American government look like a bunch of fools. It may serve a purpose, but it gives a lousy impression to the millions of foreigners who are forced to answer it every year. HiLo48 (talk) 10:26, 19 September 2011 (UTC)[reply]
Demjanjuk didn't mention it because he possibly never was a death-camp guard, and the whole German trial was just a farce to make Germany look tough on Nazi criminals in a rather pathetic way. But that is a topic for a different question. 88.8.79.204 (talk) 03:21, 19 September 2011 (UTC)[reply]
Probably he wasn't but it is more likely that he was, or do you truly believe that the US court simply decided to strip him of his US citizenship and to expel him so "that Germany may look tough on Nazi criminals in a rather pathetic way"? Jesus, I see this so often: "the German who wasn't even born before 1945 has to pay and keep paying for the Holocaust and WWII". Even in the United 93 (film)#Criticism. They had to portray the only German passenger as a hysterical coward compared to the brave and courageous American passengers and crew. Screw this pathetic self-righteous "we are holier than the Germans" attitude. Take your cheap shot at another nation, preferably your own (whatever it may be). Flamarande (talk) 03:45, 19 September 2011 (UTC)[reply]
I said he possibly was not and you reply with "it is more likely that he was"??? That's REALLY a serious reason to convict him and THAT's what I mean by pathetic. Note: I didn't watch the film, and commenting it here is also not relevant. 88.8.79.204 (talk) 03:51, 19 September 2011 (UTC)[reply]
Let's end this. You wrote that "the whole German trial was just a farce to make Germany look tough on Nazi criminals in a rather pathetic way". If this was meant in an ironic way then I'm truly sorry, but it's kind of hard to hear/read an ironic tone of voice. Flamarande (talk) 04:04, 19 September 2011 (UTC)[reply]
It's amazing how my question ended at such topic. Quest09 (talk) 13:15, 19 September 2011 (UTC)[reply]
Godwin's law, man. It's alive and kicking. Flamarande (talk) 20:27, 19 September 2011 (UTC)[reply]


September 19

Dr. without dissertation around the world

In which countries, people don't care who just put a Dr. before his name? Quest09 (talk) 01:24, 19 September 2011 (UTC)[reply]

What do you mean by "people don't care"? And are you referring to PhD.s or M.D.s? In the US at least, people will care a lot if you are pretending to be an M.D. They might not care if you pretend to be a PhD. Blueboar (talk) 02:04, 19 September 2011 (UTC)[reply]
just sign your name Dor. John SMITH. People will assume it's a foreign abbreviation for "doctor" (whereas, though you don't mention it, you've just taken up the pen name Dorothy). 82.234.207.120 (talk) 02:16, 19 September 2011 (UTC)[reply]

Vietnam, i would say. People is not going to call you with special tittle even you are a king or the best person in the world. They just call you by your name.Trongphu (talk) 02:20, 19 September 2011 (UTC)[reply]

I was not asking where do people do not care about legit degrees. It was more like: in what countries the Dr. before your name is not regulated or where nobody cares if you did your PhD or not. In what countries John Smith could simply print a card with Dr. J. Smith and get away with it. Quest09 (talk)

Well i guess the answer to your question is "all the poor countries and some developing country". It's also base on chance, in any countries in the world, you may get caught if you unlucky. In richer countries like the US as an example, there is more chance of being caught by doing illegal stuffs like claim that you have a PhD but you actually don't have one.Trongphu (talk) 02:39, 19 September 2011 (UTC)[reply]

I am not aware of any law in the United States that forbids claiming you have a PhD even though you actually don't. Now, if you induce someone to buy your services on that basis, that's different; that's fraud, which is illegal pretty much everywhere I'd think. But just lying about it, with no transaction involved? As far as I know that's perfectly legal. (Note: I am not a lawyer.) --Trovatore (talk) 02:53, 19 September 2011 (UTC)[reply]

Definitely not the United States, you get in trouble there for using "Dr" even if you have a Doctor Juris degree (JD), which is literally a doctor of law, but only the first legal degree and not the terminal one. some woman politician got in troubl for that recently... I am sorry for saying "woman politician" I wish there was some way to edit this. I should have just said someone who was a politician (a woman). 82.234.207.120 (talk) 04:43, 19 September 2011 (UTC)[reply]

"I wish there was some way to edit this"? You must be joking. ElMa-sa (talk) 08:39, 19 September 2011 (UTC)[reply]
In the UK, if you hold a doctorate in any subject, you can call yourself "Doctor" quite legitimately. Unfortunately, people tend to think that only medical doctors are entitled to use the term "Doctor". This can cause problems. Is this the sort of situation the OP is referring to? --TammyMoet (talk) 12:05, 19 September 2011 (UTC)[reply]
Nope. Quest09 (talk) 13:12, 19 September 2011 (UTC)[reply]
In the UK, anyone can call themselves anything they like. There are laws against practising medicine without a license and there are laws against fraud, but neither of those stop you using any title you like socially. --Tango (talk) 12:08, 19 September 2011 (UTC)[reply]
That is, Dr. Tango. Quest09 (talk) 13:12, 19 September 2011 (UTC)[reply]
I’m not a doctor, but I play one on TV for the "that's a bluelink?" of the day... --Jayron32 12:51, 19 September 2011 (UTC)[reply]
What do you mean by 'get in trouble'? I've looked in to the case and I'm finding no evidence there was any real legal risk. In most countries with a free media and resonable level of democracy, calling yourself by a title people feel is undeserved or unwarranted is liable to have a negative effect on your reputation which is generally a bad thing for a politician, but this is a rather different thing. There are of course some circumstances where you may get in trouble as Tango mentioned for the UK (but likely apply to the US) and Trovatore mentioned in the US but again that doesn't mean it's illegal to call yourself a Dr in all circumstances in the US if you have no (what people would call) legitimate claim to the title. Nil Einne (talk) 03:54, 20 September 2011 (UTC)[reply]
Dr. J, Dr. John, and Dr. Hook aren't breaking any laws, as far as I know. --Jayron32 04:02, 20 September 2011 (UTC)[reply]
Well, Dr. Hook probably is, but that's an issue aside from the name... --Mr.98 (talk) 17:34, 20 September 2011 (UTC)[reply]
Only at the Freaker's Ball, however...--Jayron32 19:22, 20 September 2011 (UTC)[reply]

Coal miners in gilded age at United States?

How many people were dead each year in the gilded age in the United States (after civil war to 1900 year) by working in coal mine? What are the majority cause of the death like black lung disease or accident or...? Is there any protection for coal miners during that time like equipment or something like that? What are the protection, equipment? Total death toll for coal miners in 19th century in the US? And if anyone knows any more info and statistic about coal mining in gilded age. Everything about it would be helpful. All the info should only from in period of the gilded age in the US. Thanks!Trongphu (talk) 02:35, 19 September 2011 (UTC)[reply]

Social sciences and humanities approaches to past labour safety issues have fallen under historical Industrial Relations and Labour History. These fields tend to not produce statistical compilations, due to the highly contested ideological nature of what constitutes a work place death and socially acceptable safety equipment. My understanding, from reading the abstract to This Book and from Gollan's book on the Australian coal miners, is that coal mining was paid on a piece rate system by gang, where the Boss controlled the piece rates, causing the use of safety apparatus and shoring work to be underfunded systematically (See Emile Zola's novel Germinal). Total Death Tolls for mining are hard to compute. Do pinkerton murders count? Does drinking yourself to death because you hate the work count? Does dying at 60 from exhaustion count? Black lung? As Gollan observes in relation to Australia, coal miners became class conscious at a very early stage—control over the mining industry was about naked class warfare, including day-to-day resistance. Treating this kind of territory as if it is amenable to a statistical analysis of agreed "industrial deaths" and acceptable "protection equipment" is futile. Miners wanted more shoring and more money for shoring universally. Employers wanted to (and did) test the minimal necessary shoring requirements by destructive testing. Fifelfoo (talk) 03:24, 19 September 2011 (UTC)[reply]
So there are no statistic before 20th century? I found some statistics about total death toll from beginning of 20th century forward. By death toll i mean all the deaths involve in mining (suicide because you hate the job would also count since mining caused it). There are actually no safe equipment to make the job safer? I doubt it, i believe over the past century they have developed many equipment to make it safe for mine workers.174.20.78.221 (talk) 21:42, 19 September 2011 (UTC)[reply]
In many cases, safety equipment was technologically available, but not provided by mine operators. See, for example, the Darr Mine Disaster; 239 deaths because open lamps were used rather than safety lamps, which had been developed almost a century earlier.
Regarding overall fatality statistics, it's not that they don't exist, but they're harder to calculate - there was much less official reporting of this sort of thing in the past. You might find this paper interesting; it quotes an estimate of ~3 direct deaths per thousand workers per year in the late nineteenth century. There's a fairly broad bibliography, and you'll likely find all the statistics you need there... Shimgray | talk | 21:53, 19 September 2011 (UTC)[reply]

Here we go, the link is what exactly what i needed. Thanks a lot. Thanks the person above too for some addition info.Trongphu (talk) 22:54, 19 September 2011 (UTC)[reply]

How much has black flight contributed to the population decline and urban decay of many American cities since the 1950’s when compared to white flight?

In my previous question I asked why all those American cities that are declining in population generally started doing so in the 1950s and what were the reasons behind their decline. One of the reasons given to me had to do with the concept of white flight. That is a term I've heard many times before (but I'm not too familiar with) and that many of us are probably familiar with, but when I got into the See Also section of the white flight article I came across the concept of black flight. As black flight is now a new concept for me like I mentioned above, I’m curious to know to what extent has black flight contributed to the population decline and urban decay of many American cities since the 1950’s compared to white flight. Black flight doesn't seem to be a term that comes up often. Willminator (talk) 02:38, 19 September 2011 (UTC)[reply]

I'm pretty sure white flight has bigger influence. I'm not try to being racist or something. I'm a pro equally for ALL. But well i do have to admit that white typically richer than black (simply enough because they have been superior for so long, which shouldn't be that way) so there should be more white move to suburb and have a nice house(or just a house) than black.Trongphu (talk) 02:43, 19 September 2011 (UTC)[reply]
Also because whites are simply more numerous in America. 69.171.160.78 (talk) 04:07, 19 September 2011 (UTC)[reply]
Not on topic, but interesting, I think: [5] Bus stop (talk) 04:26, 19 September 2011 (UTC)[reply]
I don't think "black flight" would have much effect on city population, since only wealthy blacks typically leave, and they are a small portion of the population. However, losing wealthy taxpayers (of any race) has a disproportionate effect on the tax-base, especially if they are business owners and take their business with them to their new location. StuRat (talk) 04:32, 19 September 2011 (UTC)[reply]
What are you looking for that the article doesn't cover. 9 million blacks moving from the inner city to the suburbs (1960-2000) certainly appears significant. Rmhermen (talk) 06:39, 19 September 2011 (UTC)[reply]
Unscrupulous lending practices by banks allowed almost anyone to get a mortgage in the mid 2000s and buy a home, leading to movements to new suburban housing. This applies to blacks as well as whites; poor blacks have been facing a lot of repossessions since the credit crash. --Colapeninsula (talk) 09:43, 19 September 2011 (UTC)[reply]
There are two books you should read if you are interested in this subject:
  • Dreier, Peter (2005) [2001]. Place Matters (2nd Revised ed.). Kansas: University Press of Kansas. ISBN 0-700-61364-1.
  • Peterson, Paul E. (1981) [1981]. City Limits. Illinois: University of Chicago Press. ISBN 0-226-66293-4.

Gx872op (talk) 15:12, 19 September 2011 (UTC)[reply]

Gx872op, I'll check out those books you recommended to me. Thank you. Willminator (talk) 17:45, 19 September 2011 (UTC)[reply]

The second paragraph of the black flight article says that “Early years of residential change accelerated in the late 1960s after passage of civil rights legislation ended segregation, and African Americans could exercise more choices in housing and jobs. Since the 1950s, there began a period of major restructuring of industries and loss of hundreds of thousands of industrial jobs in northeast and Midwest cities. Since the late 20th century, these events led to reduced density in formerly black neighborhoods in cities such as Chicago, Detroit, and Philadelphia, which have also had absolute population decreases, losing white population as well.” So, does that means that the black flight to the suburbs was indeed a significant and impactful thing, but still not nearly as significant and impactful as white flight was, and not nearly as significant and impactful to warrant significant attention in the history books? Willminator (talk) 17:45, 19 September 2011 (UTC)[reply]

The main social impact of black flight has been to separate the poor African American population of many cities from the African American middle class, many or most of whom have moved to either the suburbs or to more affluent urban neighborhoods. This has led to the development of poor African American neighborhoods that lack role models and personal networks that might lead to employment opportunities. Before black flight, most African American neighborhoods included middle-class families who attended the same churches and schools as the poorer population and whose businesses might have offered jobs to poorer neighborhood acquaintances. Sociologists such as William Julius Wilson have argued that black flight has cut off many of these avenues of opportunity and consigned poor black neighborhoods to entrenched poverty. So, the main impact of black flight has not been on urban populations in most cities but on economic conditions in cities with large poor, black populations. Marco polo (talk) 18:24, 19 September 2011 (UTC)[reply]

does anyone know any jokes like this?

does anyone know any jokes like this:

person a: Do you know any (somebody's name) jokes? person b: Yeah - (same name)!

In other words, person b is saying that that person themselves is a joke. for example: Do you know any Michele Backmann jokes? Yeah - Michele Bachmann!

anyway this is the formula and I wonder if anyone knows any jokes like that. 82.234.207.120 (talk) 04:51, 19 September 2011 (UTC)[reply]

I vote we move this to the Miscellaneous desk, but to the OP, can you tell us what on earth you actually want? How broadly are we to interpret your formula? If it has to be in the exact format you described, I could give you several, but they would all be the same, and by the way, none of them would be particularly funny. It's been emotional (talk) 06:13, 19 September 2011 (UTC)[reply]
"Yeah - jokes with that formula!". I guess though that there must be some people who are particularly common targets of this form of joke (or non-joke).  Card Zero  (talk) 08:21, 19 September 2011 (UTC)[reply]

There is currently a discussion at Wikipedia:Administrators' noticeboard/Incidents regarding posts by this user. --RA (talk) 12:18, 19 September 2011 (UTC)[reply]

See here in the names section--85.55.198.76 (talk) 20:13, 20 September 2011 (UTC)[reply]

Electricity Pylons

Could anyone tell me who owns pylons in the UK? If they are placed on public land (e.g. in/near parks), do the owners pay "rent" or any sort of fee for placing them there? And, finally, how successful are petitions to get them removed from public areas for aesthetic reasons? Thanks much in advance for any answers. ScarianCall me Pat! 12:10, 19 September 2011 (UTC)[reply]

In England and Wales, National Grid plc. In Scotland, Scottish Power and Scottish and Southern Energy. Pass on question 2. ON 3, I guess "not very", since the costs of removal are extremely high. But cheer up: there's quite a lot going on in prettifying pylons. --Tagishsimon (talk) 13:09, 19 September 2011 (UTC)[reply]
Perfect answer. Ta very much. :-) ScarianCall me Pat! 13:13, 19 September 2011 (UTC)[reply]
The basic pylon design was by Sir Giles Gilbert Scott - [6] - who also designed cathedrals. Some would say his pylons are rather more functional, and no less beautiful. Pylon fans can join the Pylon Appreciation Society. Ghmyrtle (talk) 13:20, 19 September 2011 (UTC)[reply]
For US readers, an electricity pylon is an electrical transmission tower. StuRat (talk) 15:36, 19 September 2011 (UTC)[reply]
When talking about removing them, there are two possibilities:
1) Bury the wires, instead.
2) Move them elsewhere. This is sure to bring opposition from whoever now gets to stare at them. StuRat (talk) 15:50, 19 September 2011 (UTC)[reply]
Regarding question 2, yes the power companies pay rent. See here for some details of wayleaves and easements (which are not the same thing in UK law, though Wikipedia seems to think they are). There was a big fuss a few years ago when the power companies wanted to pay farmers a lump sum instead of administrating ongoing rents. They didn't get away with it.--Shantavira|feed me 16:02, 19 September 2011 (UTC)[reply]
I seem to recall my father, as occupier of some land, though not the owner, used to receive one shilling per pylon per year back around 1970. He never did get very rich... -- Arwel Parry (talk) 17:29, 20 September 2011 (UTC)[reply]
A relative is involved in the Groton Pylon Alliance which aims to have some projected pylons rerouted past somebody else's houses. Removing existing pylons may be a taller order, although they did it for the 2012 Olympic Park. Alansplodge (talk) 16:30, 19 September 2011 (UTC)[reply]
Following on from Ghmyrtle's comment above, there is currently a UK competition (between professional firms of architects) for the design of the 'next generation' of the UK's pylons - one report with some pictures is here, others can be found by googling. What those stills don't make obvious is that most of the 6 finalist designs are shorter than (though retaining the same cable heights as) the current Scott design. {The poster formerly known as 87.81.230.195} 90.193.78.49 (talk) 18:43, 19 September 2011 (UTC)[reply]
Agreed that Ghmyrtle's link says it was designed by GGS and I thought so too, but nearly everyone else seems to think that the winner of the 1928 competition (chosen by classical architect Reginald Blomfield) was a design by the Milliken Brothers (don't tell anyone, but they were Americans shhhh...) [7][8][9][10][11][12] Maybe the confusion comes from the fact that Blomfield was on the 1924 committee that chose GGS's design for the Red telephone box. Alansplodge (talk) 20:45, 19 September 2011 (UTC)[reply]
The most successful way to get existing pylons replaced by underground ducts, is for yourselves to offer to pay for the necessary work – which is horrendously more expensive than overhead transmission. The problem with petitions is that the petitioners expect that for their unobstructed view, all the transmissions company's other customers should pay for it or other tax payers (if it comes from the public purse. --Aspro (talk) 19:05, 19 September 2011 (UTC)[reply]
To answer one of your questions, petitions on their own are unlikely to succeed. A big campaign, which would include a petition, might have some chance if it is correctly prepared and if enough people join in. The first step is to talk to the local authority planning department(s) and find out their view. Find out if the land is in a national park or an Area of Outstanding Natural Beauty. Is anything special about the landscape that means the power lines should be buried? Itsmejudith (talk) 19:51, 19 September 2011 (UTC)[reply]
A third option may soon be to use carbon nanotube based transmission cables, which should be stronger and carry more power, reducing the number of pylons needed. Wnt (talk) 10:32, 20 September 2011 (UTC)[reply]
"Here at British Plutonium, we use the latest technology to make your world only half as hideous as we made it before." :-) StuRat (talk) 19:25, 20 September 2011 (UTC) [reply]
Well, especially in a place like the Ridge and Valley province of Pennsylvania, which has something of a Riverworld-like topology, stronger cables would really matter. I think it might make the difference between having a 50-foot wide clear-cut all the way down one forested hill, across inhabited valley, and up the next hill, versus having two small cleared areas at the top of each hill linked by cables you would barely see, except for the all too inevitable lights and spheres to warn off planes. Wnt (talk) 20:17, 20 September 2011 (UTC)[reply]
There's currently a good article on The Economist website roughly on this subject. Useful factoids: underground cables are 12 - 17 times more expensive than pylons, involve a 17m to 40m wide sterilised corridor, and have to be replaced more often. :--Tagishsimon (talk) 23:20, 21 September 2011 (UTC)[reply]
I didn't see where it says they last longer on pylons, and that doesn't seem to make sense, since the buried wires are protected from the environment (assuming you bury them below the frost line). StuRat (talk) 04:15, 22 September 2011 (UTC)[reply]

Earliest certain celebrity endorsement in history?

Simple question - what is the earliest celebrity endorsement in history that historians are certain actually happened? Obviously this disqualifies conjectures that Da Vinci was on posters for Luigi's Quills & Scrolls... The Masked Booby (talk) 12:40, 19 September 2011 (UTC)[reply]

I expect claims could be made about ancient Egypt. The Pharaohs endorsed all sorts of things, depending on what you mean by "endorse". Itsmejudith (talk) 12:43, 19 September 2011 (UTC)[reply]
I would suspect there'd be something of this nature in Herodotus, but I can't think of a precise example right now. (Herodotus doesn't necessarily confer certainty, though). I think it would be celebrity endorsement for a fashion (a ruler or athlete or politician started doing or wearing something in a particular way, and others imitated), rather than for a specific branded product. Maybe that doesn't quite fit the question. --Demiurge1000 (talk) 12:57, 19 September 2011 (UTC)[reply]
Royal warrant of appointment is a centuries-old old system in several countries, if the king/queen counts as a celebrity; according to Royal Warrant of Appointment (United Kingdom) the earliest in the UK was in the year 1155 to the Weavers' Company. The articles don't have much useful information. --Colapeninsula (talk) 13:17, 19 September 2011 (UTC)[reply]
I have read that there is a graffito in Pompeii to the effect that the prostitutes of a given brothel recommended a vote for a given politician. -- Mwalcoff (talk) 22:23, 19 September 2011 (UTC)[reply]
That's true. It's in classical Latin too. HiLo48 (talk) 10:31, 20 September 2011 (UTC)[reply]
Biblical evidence for sponsorship of a motor manufacturer: Moses came down in Triumph. --Dweller (talk) 09:08, 20 September 2011 (UTC)[reply]

Animals and the New Zealand Māori

Are there any animals typically associated with the Māori of New Zealand? --CGPGrey (talk) 12:53, 19 September 2011 (UTC)[reply]

"New Zealand has many unique native fish, insects, birds, lizards and frogs. Our only native mammals are bats and marine mammals."[13]. See also Biodiversity of New Zealand. Alansplodge (talk) 13:22, 19 September 2011 (UTC)[reply]
(e/c) Not a full answer, but the extinct bird the Huia was sacred to Maoris. There weren't actually many vertebrates in New Zealand other than birds (see Fauna of New Zealand). --Colapeninsula (talk) 13:23, 19 September 2011 (UTC)[reply]
Kiwi - see kiwi and Māori. Gandalf61 (talk) 14:16, 19 September 2011 (UTC)[reply]
Or maybe whales; see Whales in Māori tradition and Maori, whales and "whaling"; an ongoing relationship. We have an article on the legend of Paikea. There was also the rather good 2002 film Whale Rider. Alansplodge (talk) 15:38, 19 September 2011 (UTC)[reply]
But the kiwi most particularly. We even call the people "Kiwis". Is there any other people on Earth who are regularly referred to by the name of an animal peculiar to their country? I doubt it. Australians are not called "kangaroos" or "koalas" or "platypuses" (but I have known a couple of Tasmanian tigers). -- Jack of Oz [your turn] 20:42, 19 September 2011 (UTC)[reply]
Frogs? ;-) Alansplodge (talk) 20:59, 19 September 2011 (UTC)[reply]
Oh, yes. But that's a little bit pejorative, whereas "kiwi" can be used without fear of offending anyone. -- Jack of Oz [your turn] 21:03, 19 September 2011 (UTC)[reply]
I know that the Papua New Guinea national rugby league team is known as the Kumuls. Not sure if the term can be applied to PNG nationals too. Our article on the bird doesn't even mention the rugby league connection. --Dweller (talk) 09:06, 20 September 2011 (UTC)[reply]
I doubt it. For some unexplained reason, around 20 years ago it became customary to name to give every separate Australian sporting team a different nickname, even different names for the male and female teams. One of our rugby teams (can't recall which one) is called the Wallabies, but that's not a nickname for Australians in general. HiLo48 (talk) 22:34, 20 September 2011 (UTC)[reply]
I should clarify that "Kiwi" applies to all New Zealanders, not just Maoris, but it certainly includes Maoris. -- Jack of Oz [your turn] 21:34, 20 September 2011 (UTC)[reply]

Survival rate of US government programs that kick in in the next administration

When some president actually gets something he's backing through Congress and signed into law, but that bundle of legislation is set to take effect after he's out (and won't be able to veto attempts to change it anymore), how often does it happen that that legislation gets reversed or modified at all before it even spends one day as active law as originally passed? For instance, in a current article talking about President Barack Obama's proposed debt plan, "[a senior administration official] added that any changes to Medicare benefits won't kick in before 2017." If such a thing passed, what are the odds (and by 'what are the odds,' I mean the odds based on past survival rates as I'm talking about) that those "changes," whatever they are, would become active law exactly as they were when they got passed (hypothetically assuming they did) by 2017? Yes, yes, past performance cannot predict future returns. I'm just asking the factual question of survival rates until implementation post-originator in the past. 20.137.18.50 (talk) 13:16, 19 September 2011 (UTC)[reply]

I'd think it would depend almost entirely on who follows the current President:
1) If a member of the opposing party is now President (and dominates Congress, too), the chances it will last are very low.
2) If a member of the same party is now President (and dominates Congress, too), the chances it will last are quite high, especially in the case where the previous President was assassinated. StuRat (talk) 15:42, 19 September 2011 (UTC)[reply]
Incidentally, the way to stop this back-and-forth legislation is to require a supermajority to pass or repeal laws. Currently, the only mechanism for this for regular laws (as opposed to Constitutional Amendments) is the rather silly filibuster and cloture process in the US Senate. If both houses needed 60% to pass a law and 60% to repeal it, then laws would be unlikely to pass and even more unlikely to be repealed (since going from 60% support to 40% is a big leap). StuRat (talk) 15:47, 19 September 2011 (UTC)[reply]
We effectively have a 60% majority requirement for anything at the moment on account of the filibuster requirement in the Senate. It has not proved very effective at actually making policy. This is in part because the current state of the filibuster does not require an actual filibuster, just the threat of one. --Mr.98 (talk) 22:25, 19 September 2011 (UTC)[reply]
I don't think filibusters can be used in all cases, though. For one, thing, they're exhausting. So, little bills get through, since nobody cares enough to filibuster them. StuRat (talk) 00:00, 20 September 2011 (UTC)[reply]
You don't actually have to get up and speak to prevent passage these days. If someone indicates that they will filibuster the bill to prevent passage, the Senate will move onto other business. See Filibuster in the United States Senate. Personally, I wouldn't mind seeing Reid pulling an LBJ at some point and making an opponent actually get up on their feet and talk for a while, but that's not likely to happen. Buddy431 (talk) 06:56, 20 September 2011 (UTC)[reply]
What Buddy431 says (and I said) is correct. Filibusters are not exhausting anymore. They are used routinely. Since the Republicans became a minority in the Senate in 2007, there were well over 100 filibuster threats (note that the graph there are for cloture motions, which attempt to undo/avoid filibusters — so there are likely more filibuster threats than indicated on that chart), over a doubling of the previous period (which itself showed a steady increasing of filibuster threats from previous congresses). They've relied on the filibuster as their main "negotiating tactic" because it's very hard to get a 3/5ths majority and because they don't actually have to do any filibustering. One of the major reforms being suggested for reducing this abuse of the filibuster is to require them to actually filibuster again — if they are going to require 60 votes, they ought to have to work for it, not just say, "we're filibustering" and go home. --Mr.98 (talk) 15:08, 20 September 2011 (UTC)[reply]

President's Voting for Candidates

Can you make a Wiki Page of all the Former Presidents of the United States and can you list which candidate they voted for President throughout there lifetime? — Preceding unsigned comment added by 97.102.130.241 (talk) 23:06, 19 September 2011 (UTC)[reply]

Very unlikely. We have a secret ballot. So at best we could report whom the former presidents claimed to have voted for. I doubt most of them ever even made a firm statement about that; it's generally just assumed that they voted for their own party, but you never know.
(There was a claim made that Reagan voted for Clinton in '92, but there's no way of checking it that I can see.) --Trovatore (talk) 23:22, 19 September 2011 (UTC)[reply]
By 1992 Reagan probably thought he was voting for waffles. --Sean 18:35, 22 September 2011 (UTC)[reply]
(Edit Conflict) The first part of your request is I think met by the existing article List of Presidents of the United States. The second part might be difficult: my understanding is that in the USA, as in most other countries (legally if not always in practice) everyone's vote is a secret ballot, so one could only know who they voted for in any election if they had come out and said so. Although some future or past Presidents might sometimes have done this (though who knows if all of them were always honest?), it seems unlikely that most of them did so for most of the times they voted. {The poster formerly known as 87.81.230.195} 90.197.66.241 (talk) 23:30, 19 September 2011 (UTC)[reply]
There used to be a very different attitude, where it was thought that anyone who campaigned for himself and voted for himself was too immodest. Back then, I wouldn't be surprised if they would have voted for another (or at least claimed to), so they would seem properly meek, and thus inherit the Earth, or at least the Presidency. StuRat (talk) 23:35, 19 September 2011 (UTC)[reply]
Maybe for president of the sixth grade. --Trovatore (talk) 23:46, 19 September 2011 (UTC)[reply]

Our article on the secret ballot suggests that this is possible for presidents who died prior to 1884 "In the United States, most states had moved to secret ballots soon after the presidential election of 1884." However, I'd suggest that the partisan nature of newspapers in that age, and the quality of recording Joe Q Public's voting for 40+ years prior to their presidency, means that no such data sets will exist. Fifelfoo (talk) 23:54, 20 September 2011 (UTC)[reply]


September 20

Bill O'Reilly and Vietnam,

69.86.208.28 (talk) 00:56, 20 September 2011 (UTC) Bill O'Reilly was born in 1949, which would make him 18 years old 1967. How did he stay out of the draft for the Vietnam War? Was it a high draft/lottery number? There's nothing on his wikipedia page about his draft status, and there probably should be, given his stature within political commentary and his interest in Vietnam:[reply]

"After the taping of his broadcast one recent evening, O’Reilly opined on the current uses of historical narrative. His large corner office on the 17th floor of the News Corp. building in New York, with a broad Sixth Avenue view, is itself a kind of history lesson, its walls filled with rare, signed presidential letters, photographs, and lithographs, hung alongside a homemade Viet Cong banner and the last flag of the Republic of South Vietnam to fly at the American Embassy in Saigon."

http://www.thedailybeast.com/newsweek/2011/09/18/bill-o-reilly-makes-history.html?obref=obinsite

Wouldn't he have had a college deferment? -- Mwalcoff (talk) 01:18, 20 September 2011 (UTC)[reply]
Like Bill Clinton, Dick Cheney, Dan Quayle, and my own father, O'Reilly managed to avoid the draft with his college deferment.[14] --Mr.98 (talk) 01:45, 20 September 2011 (UTC)[reply]
We don't have an article on college deferment - what does it mean? In my understanding, a deferment means you agree to do something later (defer it) but this seems to be an exemption? --Dweller (talk) 08:56, 20 September 2011 (UTC)[reply]
Conscription_in_the_United_States#Perception_of_the_draft_as_unfair says that if your parents could afford to send you to college you escaped the draft while you were there. This paper provides more information on college deferment. The draft was very dependent on when you were born. The draft picked up in mid 1965, and until 1970, the order of the draft was (1) "delinquents" (2) volunteers (3) people 18-25 in order of age from 25 down to 18 without a deferment (4) other groups, such as deferred graduates; and generally the first 3 groups provided more than enough people (so e.g. if you could last till you were 25 you were unlikely to be drafted). O'Reilly being 18 in 1967 would be low-priority at first; Clinton being a couple of years older was at greater risk. In 1970, this system was replaced by a lottery, but college deferments kept you out of that until September 1971. After mid-1971 the rate of induction was in any case vastly reduced, and the draft was suspended in 1973, so if you could escape from 1965-71 you'd be fairly safe. --Colapeninsula (talk) 09:38, 20 September 2011 (UTC)[reply]
Why was it called a "deferment", when it wasn't deferred? Or are you saying it was a deferment, they just never got round to calling some/most of those who deferred, because the draft came to an end? --Dweller (talk) 09:57, 20 September 2011 (UTC)[reply]
Draft deferments termed such because they are technically temporary. After finishing undergraduate study, you could be drafted again. In practice this was rare. Those who finished college during the draft period and who were still younger than 25 (the cut-off age for the draft) could apply for graduate deferment early on in the war, and later for occupational or dependent deferment. (From the same paper cited above.) The temporary nature of deferments was more clear in World War II, in my research experience. I have seen many records of people coming up two, three times before draft boards, deferring because of their connection to "vital war research". During WWII the draft boards were pretty aggressive and required very strong statements about irreplaceability to grant a war-research-related deferment. --Mr.98 (talk) 12:16, 20 September 2011 (UTC)[reply]
Thanks for explaining that. Thanks also for answering the IP's question, as they seem to have forgotten to return. --Dweller (talk) 13:36, 21 September 2011 (UTC)[reply]

The UC Irvine Muslim students

"The students face misdemeanor charges of conspiring to disrupt a meeting and disrupting a meeting." Is this kind of law common in the U.S.; i.e., would you assume that it is the case in any given jurisdiction? Thanks. 66.108.223.179 (talk) 03:47, 20 September 2011 (UTC)[reply]

There are lots of laws which are somewhat vague and allow the police to break up disturbances. The classic is Disturbing the peace which is likely (or a variation of it) the actual crime someone you describe would be charged with. There may be variations of the law in California dealing with various ways the peace can be disturbed, so the specific wording (disruption of a meeting) is likely one of them. --Jayron32 03:50, 20 September 2011 (UTC)[reply]
The OP seems to be referring to this story from the AP news wire. To answer the question though, local laws vary. It may be a local law that interrupting a gov't meeting is considered disruptive to the community and therefore an offense. Dismas|(talk) 03:53, 20 September 2011 (UTC)[reply]
Ohio has a similar law (ORC 2917.12, "Disturbing a lawful meeting"). -- Mwalcoff (talk) 22:22, 20 September 2011 (UTC)[reply]
See the article on the "heckler's veto." Neutralitytalk 06:50, 23 September 2011 (UTC)[reply]

Jean-Bedel Bokassa

So apparently there was this former dictator in Africa named Jean-Bedel Bokassa who became notorious for giving diamonds to the former President of France and for ordering the killing of several schoolchilden, some of whom he reportedly killed personally, because they refused to wear a ridiculously expensive school uniform with his face printed on it. When he was crowned "Emperor", his Empress was Catherine, who was the favorite of his nine wives. But he was a Catholic (except for a brief period in 1976 when he converted to Islam). So how did he manage to have nine wives? Narutolovehinata5 tccsdnew 05:55, 20 September 2011 (UTC)[reply]

Well, one of the key points of being a despot is that you make your own rules. Arguably, Bokassa violated various core principles of both Catholicism and Islam, but who would have challenged him on any of this? --Soman (talk) 07:11, 20 September 2011 (UTC)[reply]
Basically, he claimed to be the inheritor of various traditions, including African traditions by which a chief or any other wealthy or powerful individual can practice polygamy. He obeyed whatever rules or traditions suited him best at the time, and made up his own when necessary. His whole coronation as "Emperor" was a mix of Napoleonian and African practices. --Xuxl (talk) 08:00, 20 September 2011 (UTC)[reply]
The Catholic Church may have preferred to have an imperfect Catholic than an atheist communist as ruler of the CAR. Not that they could have done much about him. --Colapeninsula (talk) 09:48, 20 September 2011 (UTC)[reply]
From a Catholic point of view, nothing you do means you aren't a Catholic any more (as they say, "once a Catholic, always a Catholic") because Baptism is believed to leave a permanent mark on the soul which cannot be removed. However, any one of the actions you describe above are enough to automatically excommunicate him, which means no declaration of excommunication is necessary: he has excommunicated himself, and should be aware of that. This means he is out of communion with the Church, and remains so until he has fully repented in a way that involves ceasing to commit the sin, and sought Confession, and almost certainly he would need the excommunication to be lifted by a bishop or archbishop, maybe even the Pope. Since his sins were notorious, all Catholic priests would be expected to deny him Communion if he presented himself at the altar and they recognised him. I would be interested in any articles based on fact which discussed how (if at all) he practiced his Catholicism after becoming a dictator, and whether this involved pressure on certain priests. 86.164.76.231 (talk) 10:02, 20 September 2011 (UTC)[reply]
It seems that Joachim N'Dayen was the Catholic Archbishop in his country. I wonder how they got on? Itsmejudith (talk) 10:34, 20 September 2011 (UTC)[reply]
Apparently during the "coronation", N'Dayern read greetings from the pope. Other than that, I'm not so sure. He may have been religious as he wanted a coronation in a cathedral in Bangui and apparently during his trial he wore a white cassock and a cross given to him by the pope (at least according to a book that I read). Narutolovehinata5 tccsdnew 14:01, 20 September 2011 (UTC)[reply]

According to his Wikipedia page, by the time he died, he had 17 wives. That's a lot of wives. The fact that he was able to practice polygyny even if he was a Roman Catholic astonishes me. My point is, how did he manage to have this many wives? Narutolovehinata5 tccsdnew 14:06, 20 September 2011 (UTC)[reply]

If you valued your life in CAR at that time, you didn't say or do anything against the president. He basically just did what he likes. As to what his religion had to do with it, you'll have to ask his God what He has planned for him in the afterlife! --TammyMoet (talk) 14:26, 20 September 2011 (UTC)[reply]
I'm still not sure I see what you're asking. From the point of view of the Catholic Church he wouldn't have been a Catholic with 17 wives, he would have been a Catholic married to his first wife and committing adultery with 16 other women. 86.164.60.149 (talk) 16:56, 21 September 2011 (UTC)[reply]

What does "unrewarded genius is almost a proverb" mean?

I read this is a quote by Calvin Coolidge: "Nothing in the world can take the place of persistence. Talent will not; nothing is more common than unsuccessful men with talent. Genius will not; unrewarded genius is almost a proverb. Education will not; the world is full of educated derelicts. Persistence and determination alone are omnipotent. The slogan Press On! has solved and always will solve the problems of the human race."

What does "unrewarded genius is almost a proverb" mean in the context of persistence? --LijoJames (talk) 06:50, 20 September 2011 (UTC)[reply]

To paraphrase, "Genius allowed to go to waste is extremely common", in contrast with genius that achieves great things because it is combined with hard work and perseverence, which is much rarer. --Xuxl (talk) 08:03, 20 September 2011 (UTC)[reply]
"Unrewarded genius is almost a proverb" means that people say "genius goes unrewarded" it so often that it has almost become a proverb. Like "Least said soonest mended" or "lightning never strikes twice in the same place" which are traditional English proverbs and just as difficult to prove. Itsmejudith (talk) 08:07, 20 September 2011 (UTC)[reply]
It is literary shorthand that is generally acceptable in its use. One gets the gist from what is said because of the similarity between the phrase "unrewarded genius" and proverbs. The similarity is that proverbs are repeated interminably and are generally accepted as being factual beyond reproach. I think "unrewarded genius" is in fact not "almost a proverb" because proverbs are presented in the form of full sentences. By contrast "unrewarded genius" is only a phrase, and that is why I am comparing it to "shorthand": I think we understand that the phrase is similar to a proverb in that it is often repeated and seldom questioned. Bus stop (talk) 12:29, 20 September 2011 (UTC)[reply]
Much better quote, by Jim Croce:
"No matter how smooth I talked, they wouldn't listen to the fact that I was a genius. The man said 'We got all that we can use'. I've got those steadily depressin', low-down messin', workin' at the car wash blues."
Genius. --Jayron32 22:16, 20 September 2011 (UTC)[reply]
The infobox at Thomas Edison has a famous quote with a similar sentiment to the full quote by Coolidge: "Genius is 1 percent inspiration, 99 percent perspiration." wikiquote:Thomas Edison has some longer variants. PrimeHunter (talk) 02:35, 21 September 2011 (UTC)[reply]
I'm not sure of the nature of the original question, though I tried to answer it above. The question as posed was: "What does 'unrewarded genius is almost a proverb' mean in the context of persistence?"
I am puzzled because the quote from Calvin Coolidge above says that "Nothing in the world can take the place of persistence" and "Persistence and determination alone are omnipotent."
It seems obvious that the quote is saying that "genius" cannot replace "persistence". Is there a question as to how this can be a proverb? Is there an implied searching for proverbs that embody this notion? Bus stop (talk) 11:21, 21 September 2011 (UTC)[reply]
I think you are trying to over-analyze the formulation of the question. The heading was only What does "unrewarded genius is almost a proverb" mean? The meaning may seem obvious to you but people are different. LijoJames was probably just trying to understand the Coolidge quote. PrimeHunter (talk) 16:14, 21 September 2011 (UTC)[reply]

History of pornography

Can anyone please provide some online references on the history of pornography? --DinoXYZ (talk) 08:44, 20 September 2011 (UTC)[reply]

See History of erotic depictions. Dismas|(talk) 08:50, 20 September 2011 (UTC)[reply]
BTW to DinoXYZ, history of pornography redirects to the above linked article Nil Einne (talk) 00:29, 21 September 2011 (UTC)[reply]

Secret US intelligence budget

How is it possible that the United States intelligence budget is secret? Since the rest of the budget is unclassified it should be possible to work out as a residual. 131.111.195.11 (talk) 16:05, 20 September 2011 (UTC)[reply]

Not really, because its a budget and not a check register. That is, a budget is something you do before you spend money. Its a plan of how much money you expect to take in, and what you plan to spend it on. The actual amount of revenue and outlay is quite likely to be somewhat different than the budget. Since the budget is the prospective spending, the intelligence community can keep it perfectly secret, and the actual spending can be covered up pretty easily as well. --Jayron32 16:37, 20 September 2011 (UTC)[reply]
I think the question is more, couldn't you take the full budget figure, subtract everything that is listed publicly, and end up with the intelligence budget figure? It depends on whether the defense budget figure is included in the full budget figure, obviously. Presumably it is not, or is hidden into gray categories (e.g. discretionary spending given to the CIA or military). Usually when budgets are "hidden" in this way it is because they fall into vague categories that don't respond well to probing. --Mr.98 (talk) 17:32, 20 September 2011 (UTC)[reply]
I'm not sure, but in the UK I believe the total spend of the intelligence services is public knowledge, but not much beneath that (overall number of employees, I think). It wouldn't help sabotage it, really. Grandiose (me, talk, contribs) 16:57, 20 September 2011 (UTC)[reply]
The secrecy doesn't prevent sabotage in the US, either, unless you consider "public oversight" to be a form of sabotage. The only plausible argument is that a nation like Russia or China would be able to track the budget's change over time and somehow infer programmatic decisions from it. But it's such a vague measure of that. You'd get more insight into programmatic information from reading the New York Times than from that number. --Mr.98 (talk) 17:02, 20 September 2011 (UTC)[reply]
Actually, the US intelligence budget has been disclosed for the next year, for the first time in a long while. See here. $55 billion. Ooga booga. Don't tell any terrorists! --Mr.98 (talk) 17:02, 20 September 2011 (UTC)[reply]
Be careful, that figure only covers half of the US intelligence agencies. Anonymous.translator (talk) 20:36, 20 September 2011 (UTC)[reply]
That's true — I overlooked that fact. The MIP is still (pointlessly) classified, for now. --Mr.98 (talk) 22:14, 20 September 2011 (UTC)[reply]
I think that what is secret about it is the specific allocation of that amount among various programs, or even the names or existence of some such programs. Marco polo (talk) 18:33, 20 September 2011 (UTC)[reply]
That is definitely secret. But it used to be the case, until quite recently, that even the bulk sum was classified. Starting in the 2000s they started to release those for the first time. And just recently they've released budget proposals in advance of actually having them approved. As the Secrecy News post discusses, this has been a major bone of contention regarding classification for awhile now. --Mr.98 (talk) 18:59, 20 September 2011 (UTC)[reply]
I can see why, if the overall budget shoots up with no apparent reason, the public would question what was going on. For example, it might be a precursor to an invasion. StuRat (talk) 19:22, 20 September 2011 (UTC)[reply]
It wouldn't actually impart any information, other than the fact that there was increased spending. You couldn't distinguish one crazy explanation from another based on that number alone (an invasion, a new airplane, a huge slush fund for retired officers, whatever). Anyway, apparently they've decided it's no longer a big deal, as I've indicated, and are treating it like regular defense spending to a large degree. Note that the real question here is why an intelligence budget should alone be treated this way — we don't treat our other military budgets this way, we don't treat any non-military budgets this way. --Mr.98 (talk) 22:14, 20 September 2011 (UTC)[reply]

It seems likely that the budget is kept secret by including it in some other, larger, budget item. Which? Jacob Lundberg (talk) 20:50, 20 September 2011 (UTC) (I'm the IP address above)[reply]

It could be any of them. Increment the expected cost of Bic pens by a nickel, of bullets by a dollar, of toilet paper by a few cents a roll, and viola; you have now burried a $55 billion intelligence budget. --Jayron32 22:12, 20 September 2011 (UTC)[reply]
I've never heard of them hiding funds in that way. They aren't usually that sneaky about it. --Mr.98 (talk) 22:16, 20 September 2011 (UTC)[reply]
There was just a report on NPR tonight [15] about last year's Department of Justice expenditures for food items at conferences and the like; they were paying a going rate of $16 per muffin and a snack of cracker jacks and candy bars cost $32 per person. You tell me where that money is going, cuz Otis Spunkmeyer isn't getting $16 per muffin. --Jayron32 00:51, 21 September 2011 (UTC)[reply]
No, but the contractor who manages the procurements and logistics probably is. Hooray for no-bid contracts and the like. I don't find it likely that that's where they'd be "hiding" funds — it's actually too far out in the open, hiding it as "waste". What you want is to hide funds within bigger, vaguer pools. Anyway, I've never heard of them doing that sort of thing. There's actually a pretty big literature out there on government secrecy, black budgets, etc., over the years. As I said, it's not that sneaky. It's bland but effective at hiding things. --Mr.98 (talk) 12:09, 22 September 2011 (UTC)[reply]
The DOD has "discretionary" funds which are not audited publicly (but still have to be justified in executive committee sessions, to some degree). I'm not sure where the non-military stuff would be kept, but there are lots of places. During WWII the atomic bomb project was initially funded out of discretionary Presidential funds, later out of discretionary War Department funds. Wherever it is, it is likely in a big heap of other things with a bland, empty name, like "discretionary funding" or "special projects" or something like that. --Mr.98 (talk) 22:14, 20 September 2011 (UTC)[reply]

human "breeds"

Are people really subject to traits they have inherited (like violent tempers) that cannot be removed by surgery or drugs and if so could relief be found though messing with their DNA by inserting or removing fragments? --DeeperQA (talk) 17:43, 20 September 2011 (UTC)[reply]

To put your question in terms commonly used: If nature influences behaviour as opposed to nurture (see nature versus nurture), will gene therapy change behavior? Well, as you can see in the nature vs. nurture argument, there is no clear indicator that nature strongly influences behavior. Of course, this can be taken to an extreme. If a person has a violent temper and you use gene therapy to turn the person into a vegetable laying on a bed without the capability to move or talk, was the violent temper removed? -- kainaw 17:47, 20 September 2011 (UTC)[reply]
Which is pretty much what a lobotomy did. StuRat (talk) 18:06, 20 September 2011 (UTC)[reply]
Behavior patterns, in adults, are mostly hard-wired in the brain, so difficult to change. Genetic changes might be more successful in controlling how the brain develops, if performed on children. As for drugs, a testosterone blocker should decrease aggression, but also has some side effects, like lowered sex drive. StuRat (talk) 18:06, 20 September 2011 (UTC)[reply]
To answer your first (embedded) question, the jury is still out on whether personality is mainly genetic or mainly a result of fetal and childhood environment and experience. No doubt, it develops from an interplay between both genetics and experience (or environment). As StuRat says, to the extent that personality is genetic, a genetic intervention would have to take place very early, perhaps before birth, since the neural networks and other structures controlled by genes are formed during fetal and early childhood development. Changing an adult's genes would have no more effect on their behavior than deleting a software installer after you have already installed the software. Marco polo (talk) 18:31, 20 September 2011 (UTC)[reply]
If the bad temper is caused by a hormonal imbalance, then you might not even need gene therapy, a hormonal supplement might suffice. Googlemeister (talk) 19:32, 20 September 2011 (UTC)[reply]
In general, control over a violent temper requires sophisticated mental processing, i.e. frontal lobe function. Damage to the frontal lobe - even lobotomy - could degrade that and leave people more prone to problems (that article describes "irresponsibility", "lack of discipline", etc.) So I am skeptical of any easy, general methods to reduce violence unless they remove the capability for violence in general, even in situations where it is accepted like resisting a hijacking. (For that you could, say, cut the Achilles tendon or make a meal of the bicep muscle, or install a remote electric shock shackle, or some neurological equivalent action) Now it should be clear though that it is possible to get people to exercise that frontal lobe and be ready to use it, whether by conventional discipline or some new teaching. And it should also be clear that in individual cases, various interventions will reduce violence. If someone is a cocaine addict, cure the addiction; if their neighbor's kid is an aspiring tuba player, gift the child an XBox. Perhaps sometimes even hormones will work, if that is what is keeping normal self-control from working. Wnt (talk) 20:33, 20 September 2011 (UTC)[reply]
Doesn't this question belong on the Science desk? Imagine Reason (talk) 02:15, 23 September 2011 (UTC)[reply]

Most incumbents ever

What political office has had the most people ever hold it? - Presidentman talk·contribs Random Picture of the Day (Talkback) 23:09, 20 September 2011 (UTC)[reply]

Given the literally millions of offices throughout history, it may be difficult to nail down. Many Roman Republic/Empire offices were restricted to one-year terms, so they may have had hundreds of incumbants. One candidate may be the Pope of which there have been around 265ish (including Antipopes). The position of Roman Consul had a roughly continuous history for something over 1000 years, so you can find LOTS of people to fill that office (though its role changed drastically over time). See List of Roman consuls. --Jayron32 00:48, 21 September 2011 (UTC)[reply]
I'd be more inclined toward the office of Member of the House of Commons in the UK — the UK is considered the legal successor of the Kingdom of Great Britain, which is considered the legal successor of the Kingdom of England, so the House of Commons has existed since 1341. Given its often large size (650 members right now) and its history of more than two-thirds of a millennium, the office of membership in it is a good candidate. Nyttend (talk) 02:24, 21 September 2011 (UTC)[reply]
Yes, but how do you count an "office"? Wouldn't a single constituency count as a single "office"? --Jayron32 02:57, 21 September 2011 (UTC)[reply]
I was confused about that as well. It really depends on what Presidentman meant by "political office". If multiple holders are allowed concurrently then it's most likely the Roman senators. Anonymous.translator (talk) 03:04, 21 September 2011 (UTC)[reply]
Captain Regent or Vice Captain Regent of San Marino is another candidate. Their term length is only 6 months. Anonymous.translator (talk) 03:00, 21 September 2011 (UTC)[reply]
Or if you're asking about "What political office has the most incumbents concurrently?" Then it would be the Chinese National Congress Delegates at 2,217. Anonymous.translator (talk) 03:07, 21 September 2011 (UTC)[reply]
There's currently more than 20000 local councillors in the UK.[16] It depends what you mean by a distinct political office. --Colapeninsula (talk) 09:45, 21 September 2011 (UTC)[reply]
They all serve on different councils, though. If you count that as the same office, you might as well count members of the UK parliament and members of the Australian parliament as being in the same office. --Tango (talk) 17:30, 21 September 2011 (UTC)[reply]
There's one difference: because each local council is different from the rest, a former officeholder is retired from being a member of the ___ Local Council, but because there's only one House of Commons, all its former members are retired from being in the same body. Nyttend (talk) 21:26, 21 September 2011 (UTC)[reply]
The office of Lord Mayor of the City of London has been going since 1189. A full list here. Alansplodge (talk) 11:44, 22 September 2011 (UTC)[reply]
Some of the High Shrievalities are even older. For example, the names of most High Sheriffs of Kent since 1040 are known, and most since the 1230s have served for a single year. Warofdreams talk 11:58, 22 September 2011 (UTC)[reply]

September 21

John Kalymon

The question above about ESTA made me curious about the deportation of Nazis from the US, specially when I read today in the news that John Kalymon will probably be deported (he is in the List of denaturalized former citizens of the United States). So, from a legal perspective, he is being accused of killing at least one Jew, in a region which was Poland, but is now Ukraine, and that was occupied by Germany then. How do they decide now where to deport him to? Israel, Poland, Ukraine, Germany, Hague? Couldn't the US claim something like a universal jurisdiction and judge him? Wikiweek (talk) 07:39, 21 September 2011 (UTC)[reply]

Well it depends who is asking for him to be deported. They don't just "deport", they have to have a request from a foreign government first. (Except if they want to simply get rid of someone that is embarrassing, then they just try to find a government that will accept a new resident, but usually, there is a request first). --Lgriot (talk) 08:16, 21 September 2011 (UTC)[reply]
I'm pretty sure this is the same case as John Demjanjuk: Germany is pursuing some former real or imaginary minor collaborators of the Nazi Regime. Surprisingly, Eastern Europeans get pursued much more often than former German Nazis (although the latter group is much bigger and easier to persecute, go guess). Not surprisingly, the courts try to proceed as fast as possible, to bring the proceedings to a close before the accused dies. Obviously, there are just a handful of such proceedings. When you wait 60 years to get tough on Nazis, as Germany did, there are not many Nazis left, and even Israel doesn't get interested on it. 88.8.79.204 (talk) 14:31, 21 September 2011 (UTC)[reply]
I'm pretty sure that you're predujiced against Germany and/or Germans in general. Take your cheap shot at your own nation (whatever it may be). Flamarande (talk) 20:46, 21 September 2011 (UTC)[reply]
I don't see how you jumped to the conclusion that I could be predujiced or even prejudiced against Germany or Germans in general. I was just commenting a pretty concrete procedure in the same way many could criticize Guantanamo without judging the US or any similar case. 88.8.79.204 (talk) 22:05, 21 September 2011 (UTC)[reply]
I dunno, 88.8. Maybe it's we remember your last comments on this subject a couple of days ago [17] [18] which had the same sort of tone. "he possibly never was a death-camp guard, and the whole German trial was just a farce to make Germany look tough on Nazi criminals in a rather pathetic way." Your words, but we're free to draw conclusions. --Tagishsimon (talk) 22:12, 21 September 2011 (UTC)[reply]
Yes, I made both comments, and still hold the same position: the whole thing is pathetic. However, my comments do not refer to the whole of the German society. In the same way that you are not anti-semitic if your criticize some action of the Israeli military or anti-American when criticizing Guantanamo. 88.8.79.204 (talk) 22:19, 21 September 2011 (UTC)[reply]

Who Funded the 'No to AV Yes to PR' Campaign in the UK?

I would like to know if there is any information about the groups and / or parties that funded the No to AV Yes to PR campaign in the UK. I seem to remember some news stories saying that the conservative and labour parties had donated money, but some initial googleing has provided nothing definitive. --CGPGrey (talk) 08:59, 21 September 2011 (UTC)[reply]

Don't know who funded it, but it was reportedly organised by Lee Rotherham [19] of the Taxpayers' Alliance, a right-wing pressure group, and Piotr Brzezinski who works here at the Policy Exchange, a right-wing "think tank". Ghmyrtle (talk) 09:32, 21 September 2011 (UTC)[reply]
Their financial return is here. They report £21,149.67 of donations, but if you read down, all of that came in the form of non-cash notional spending on their behalf by the No Campaign. There may be more detail in the No Campaign's own return, which is not due until 5 November. Check back on the Electoral Commission website when it comes in. Sam Blacketer (talk) 12:51, 21 September 2011 (UTC)[reply]
Forgive my ignorance, but what exactly is 'non-cash notional spending'? --CGPGrey (talk) 14:47, 21 September 2011 (UTC)[reply]
Goods or services provided free or at a discount.[20][21]. --Colapeninsula (talk) 14:56, 21 September 2011 (UTC)[reply]
So does that mean that, in effect, the No campaign paid for all the 'yes to PR' ads? --CGPGrey (talk) 19:50, 21 September 2011 (UTC)[reply]
No to AV campaign neutrality under spotlight over Tory party funding - this grauniad story points to a "datablog" of funders, apparently. --Tagishsimon (talk) 20:17, 21 September 2011 (UTC)[reply]

MAG or MUG

Origin Africa, particularly Uganda. A drinking cup in the form of a man made out of metal and painted. Any ideas? Kittybrewster 11:11, 21 September 2011 (UTC)[reply]

Is there a picture or a reference to something that would help zero in on it? Bus stop (talk) 11:30, 21 September 2011 (UTC)[reply]
No. Kittybrewster 12:37, 21 September 2011 (UTC)[reply]

Israel

Why is Israel racist against Muslims and especially Palestinians? --75.6.5.122 (talk) 12:04, 21 September 2011 (UTC)[reply]

I don't think racism correctly characterizes the conflict you refer to. You seem to be assuming that it does. Can you tell me how you see the referred-to conflict as being racially defined? Bus stop (talk) 12:08, 21 September 2011 (UTC)[reply]
75.6.5.122 -- There are well over a million Arabs (mostly Muslims) with Israeli citizenship within the borders of 1949 Israel, and less than 10,000 Jews remaining within all the 20 or so Arab-ruled countries (over half of them in Morocco, which is the only Arab country which has consistently protected a whole Jewish community -- not just a pitiful remnant of a few old people -- over the last half-century or more). So based solely on hard factual numbers (i.e. statistical-demographical grounds) it would seem to be more fair to ask why Arabs are racist against Jews. For example, in 1967 Libya (under the king, not Qaddhafi) conducted a classic old-style pogrom which reduced the Jewish population from thousands to effectively zero in about a week, despite the fact that there had been Jews in Libya since before there were any Arabs in Libya, and few of the Jews had much connection with Israel... AnonMoos (talk) 12:42, 21 September 2011 (UTC)[reply]
Wikipedia has a vast amount of information on the Arab-Israeli conflict. For claims that Israel is racist, Israel and the apartheid analogy is a good article to start with. You should be aware that Israel's attitude to Muslims and the Arab world involves three interrelated topics: the status of Arab citizens of Israel; Israeli treatment of Palestinians in the occupied territory; and Israel's relationship with other Arab and Muslim nations. The following Wikipedia articles are relevant to these topics: History of the Arab–Israeli conflict, Human rights in Israel, International law and the Arab–Israeli conflict, Political status of the West Bank and Gaza Strip, Arab citizens of Israel, Palestinian territories, Status of territories captured by Israel, Foreign relations of Israel. --Colapeninsula (talk) 12:55, 21 September 2011 (UTC)[reply]

I'd also suggest you read propaganda. Seeing any of the world's many complex and long-lasting disputes in simple terms is usually a result of propaganda from biased newspapers, TV etc, and general ignorance of the detail of these problems.

Your question can be turned around. You might easily ask (to take one of the muslim groups) why is Hamas racist against Jews and expecially Israelis?. Your question and the one I just posed both have elements of truth in them... and huge great lumps of untruth.

There are racist Israelis and there are racist Palestinians. But there are also many who are not, just like the people who live in the rest of the world. Nearly all the Israelis and Palestinians I've ever met have just wanted peace and an end to the killings and none of them have come across as racist.

And in the same way, you can interpret the statements and actions on both sides in many different ways. Racist or self-protecting? Aggressive or defensive? How you see these things, ultimately, will sadly often be affected by the propaganda you consume. See also historiography. --Dweller (talk) 13:34, 21 September 2011 (UTC)[reply]

Additional relevant articles are Zionism and Racism in Israel. Marco polo (talk) 16:45, 21 September 2011 (UTC)[reply]

Overcoming the effects of propaganda isn't easy, but one solution is reading the other side's point of view. Try this piece on the BBC new site? --Dweller (talk) 10:17, 22 September 2011 (UTC)[reply]

"Yossi Klein Halevi, an Israeli author and political commentator, argues that all the Palestinians need to do to get a state is to convince Israelis that this state does not represent a threat." - I've been waiting a long time to see words like this published. I like this author already, smart. Sir William Matthew Flinders Petrie | Say Shalom! 23:51, 22 September 2011 (UTC)[reply]
When smart=voices my opinion, you know you're living in a bubble, Flinders, old chap. I suggest a little more Fisk might help. --Tagishsimon (talk) 00:00, 23 September 2011 (UTC)[reply]
I would suggest getting to know many more Israelis mate. I'm pretty sure they know what they want and it's much better than getting it secondhand from a journalist or writer. Sir William Matthew Flinders Petrie | Say Shalom! 00:22, 23 September 2011 (UTC)[reply]
By the same token, you could get some new pals? --Tagishsimon (talk) 00:18, 23 September 2011 (UTC)[reply]
Oh my no; I've many already from all over the globe. One of the benefits of living in New York City. Sir William Matthew Flinders Petrie | Say Shalom! 00:22, 23 September 2011 (UTC)[reply]

reductio ad absurdum, a logical fallacy?

How can a legitimate form of argument be a logical fallacy as seen here --Dondrodger (talk) 12:53, 21 September 2011 (UTC)[reply]

Did you read the article you linked to? It says

In formal logic, the reductio ad absurdum is a legitimate argument. It follows the form that if the premises are assumed to be true it necessarily leads to an absurd (false) conclusion and therefore one or more premises must be false. The term is now often used to refer to the abuse of this style of argument, by stretching the logic in order to force an absurd conclusion. For example a UFO enthusiast once argued that if I am skeptical about the existence of alien visitors, I must also be skeptical of the existence of the Great Wall of China, since I have not personally seen either. This is a false reductio ad absurdum because he is ignoring evidence other than personal eyewitness evidence, and also logical inference. In short, being skeptical of UFO’s does not require rejecting the existence of the Great Wall.

Reductio ad absurdum is a legitimate style of argument, but it can be easily misused by nonsensical exaggeration. --Colapeninsula (talk) 13:07, 21 September 2011 (UTC)[reply]
(EC) Legitimate form of argument according to whom? There is no official arbiter. Not only reductio ad absurdum, I have noticed most of the popular logical fallacies being used by reputable people and institutions under the impression that they are legitimate forms of argument; the point about listing them as logical fallacies is precisely because they are used as legitimate forms of argument. In the case of reductio ad absurdum, the essence of the logical fallacy comes in the leap between what is actually proposed and the version of it when its opponent has reduced it to the absurd. If the opponent does not leap the gap but instead makes a reasonable argument that the proposal would actually reduce to the absurd proposition, then that is not a logical fallacy. Sam Blacketer (talk) 13:09, 21 September 2011 (UTC)[reply]

In mathematics, Proof by contradiction is considered a perfectly-valid form of proof... AnonMoos (talk) 13:31, 21 September 2011 (UTC)[reply]

Many so-called logical fallacies can be legitimate (though not always conclusive), e.g. argument from authority should be persuasive if the people speaking are generally recognised as authorities on the subject under discussion, but if the Pope tells you something about biology that's less persuasive because the Pope isn't a biologist. --Colapeninsula (talk) 15:07, 21 September 2011 (UTC)[reply]

If you want to argue from authority, it helps to talk about the credibility of an proposition rather than its truth or falacy. Since we know people aren't infallible, the best you can say is that something is probably true if an expert says it is. You can't say it is definitely true. If you can come up with a logical argument from accepted premises, then you can make a definite assertion of truth (conditional on your rules of inference being sound, I suppose, but you have to start somewhere!). --Tango (talk) 17:36, 21 September 2011 (UTC)[reply]

Reductio ad Absurdum is a narrow step away from the slippery slope fallacy. In short: if there is a direct and logical link from one action to the next then the argument is valid. If there is NO logical link from one to the other, then it is a fallacy. As an example, "if you outlaw religion A because it's anti-national why not religion B, or C? soon we might ban religion altogether!" is a valid attack on anticlerical fascism- there is a link between religion A, B and C, for instance they are all christian denominations with similar beliefs. On the other hand "If we ban rocked launchers what's next? Pistols? hunting shotguns? before you know it we'll have to register our kitchen knives!" is a slippery slope fallacy because there is an essential difference between a military antitank weapon, a sporting weapon and kitchen tools. HominidMachinae (talk) 05:14, 22 September 2011 (UTC)[reply]

Right wing Irish republicanism and left wing Ulster unionism

Since apparently there aren't any major political parties that take such a stand, who do more left wing leaning unionists and right wing republicans vote for? Also, why is that part of the political spectrum void of major parties in Northern Ireland? --Belchman (talk) 13:01, 21 September 2011 (UTC)[reply]

There are left-wing Ulster Unionists, including most notably Chris McGimpsey who frequently spoke at Labour Party conferences in the 1980s and 1990s. The Democratic Unionist Party originally identified itself with the left-wing in economic policies, although this is less so nowadays. Back in 1918 the Ulster Unionists set up the Ulster Unionist Labour Association to attract the support of working-class voters. Two successive Unionist MPs for North Down have been or become Independent MPs who normally align with the British Labour Party: Robert McCartney and Sylvia Hermon. As far as right-wing republicans, many have conservative views on social policies. Sam Blacketer (talk) 13:09, 21 September 2011 (UTC)[reply]
There have been a few historical occasions when some working-class Irish Protestants and Catholics have found common cause in economic grievances against the English ruling classes, but the Anglican ascendancy generally found this to be far more threatening than Catholic-only disaffection, and took steps to effectively break up such alliances. The main Catholic/Nationalist-aligned party which could be considered somewhat "moderate" (though hardly right-wing!) is the Social Democratic and Labour Party... AnonMoos (talk) 13:15, 21 September 2011 (UTC)[reply]
The Progressive Unionist Party is left-wing, though it's smaller than the UUP or DUP. The Alliance Party of Northern Ireland is described as "liberal", and gets support from both unionist Catholics and Protestants. Catholic Unionist covers some boundary crossing, but suggests it's rare. In first-past-the-post elections different considerations are involved, and unionists may vote SDLP to keep Sinn Fein out.[22] --Colapeninsula (talk) 13:16, 21 September 2011 (UTC)[reply]
There was previous discussion of left-wing unionism at Wikipedia:Reference desk/Archives/Humanities/2009 October 3#Left-wing unionism. Historically, large sections of Sinn Féin and the Nationalist Party were conservative, and despite SF and the SDLP both being nominally on the left, significant minorities within the parties support conservative social and economic policies. Fianna Fáil, which is broadly conservative and historically republican, had planned to stand candidates in Northern Ireland before its electoral woes set in; it attracted interest from some SDLP members, and Gerry McHugh defected from SF.
As to why no major parties are left-wing unionists or right-wing republicans, I know of no good evidence, but I would hypothesise that republicans saw many of their supporters discriminated against, naturally desired social change to rectify this; meanwhile, unionists in defending the union with Britain have tended to see this as good reason to defend existing social conditions, or have romanticised those from the period before the Troubles. Warofdreams talk 15:36, 21 September 2011 (UTC)[reply]
As regards PUP and their left-wing tendencies, LVF founder Billy Wright (loyalist) described the Belfast UVF as "communists". PUP is comprised of former UVF members.--Jeanne Boleyn (talk) 15:46, 21 September 2011 (UTC)[reply]
So in other words, nationalism and populism are generally orthogonal. 70.91.171.54 (talk) 21:20, 21 September 2011 (UTC)[reply]
Nobody has mentioned populism, so that's not a useful summary of the discussion above. Warofdreams talk 09:45, 22 September 2011 (UTC)[reply]
A couple of other points: prior to Irish independence, the Conservative Party generally opposed Irish Home rule and the more left-wing Liberal Party supported it. Many of the leading figures in Irish nationalism, such as James Connolly, were socialist. In the west of Scotland there was for most of the 20th century a strong, almost tribalistic, connection between on one side the Conservative party and Protestantism, and on the other the Labour party and Catholicism. The principal parties of Scottish and Welsh nationalism, Plaid Cymru and the Scottish National Party are both leftwing (elsewhere regionalist parties like the Northern League in Italy may be on the right - possibly because northern separatism in Italy is based on the rich north being reluctant to subsidise the poor and often corrupt south, while Ireland, Wales, Scotland traditionally feel poor and hard-done-by). Hence in Britain there's a strong link between unionism and rightwing politics, and between the desire for regional independence/devolution and leftwing politics. It's often the case that political parties hold multiple orthogonal viewpoints, e.g. social liberalism/conservativism and leftwing/rightwing economic policies, or the divisions in British politics over European integration. --Colapeninsula (talk) 12:42, 22 September 2011 (UTC)[reply]

MMP & Overhang Seats

I'm having a hard time understanding this section on the Overhang seat article as it relates to Mixed-member proportional representation:

Take the number of additional list seats off from the other parties' proportional entitlement — A party is allowed to keep any overhang seats it wins, and the corresponding number of list seats allocated to other parties is eliminated to maintain the number of assembly seats. This means that a party with overhang seats has more seats than its entitlement, and other parties have fewer.

It's unclear to me how a party that is already over-represented ends up with an extra seat and makes the parliament bigger.

Lets say there is a ten-member parliament with 5 constituency seats and 5 list seats. There are three parties who win the results in all the constituencies as follows: A 34% B 33% and C 33%

Under this result, A would be over represented because it gets all of the constituency seats giving it 50% of the parliament. How are the rest of the seats determined and where do the overhand seats come from? --CGPGrey (talk) 15:22, 21 September 2011 (UTC)[reply]

There are numerous ways of doing this. For an example, the Scottish Parliament uses the D'Hondt system. To allocate the first seat, each party's total votes are divided by a number one more than the number of seats they already won. Whichever party has the highest total is awarded a seat. This is then repeated until all seats are filled. In your example, assuming that party B had one more vote than party C, of your five list seats, three would go to B and two to C. There is more detail on the Scottish system in this factsheet, but bear in mind that this is only one possible way of apportioning the seats.
I think that the section in the article is confusingly worded, and I'm unconvinced that it is appropriate to call additional constituency seats "overhang seats" in these circumstances where the size of the parliament is not increased. Warofdreams talk 15:50, 21 September 2011 (UTC)[reply]
Let me give an example : in the 2008 New Zealand election the Maori party got more constituency seats than it should have, and the size of the parliament was increased by one. Where did this extra seat come from? --CGPGrey (talk) 16:19, 21 September 2011 (UTC)[reply]
That's the first example in the overhang article - "Allow the overhang", not the second one which you asked about above. In this case, the extra list seat was granted in order to ensure that each party still got the number of list seats to which they were entitled - it didn't come "from" anywhere. Is the explanation about this in the mixed-member proportional representation article any clearer for you? Warofdreams talk 17:05, 21 September 2011 (UTC)[reply]
OK, I think I'm getting muddled up here. Let me see if I understand, in general, how overhang seats happen.
Lets say there is a ten-member parliament with 5 constituency seats and 5 list seats. There are three parties who win the results in all the constituencies as follows: A 34% B 33% and C 33%
So party A gets all 5 constituency seats.
According to the proportional results, of the seats, Party A should get 4 and Parties B & C should get 3 each. However, because party A already has five seats, it gets none of the proportional seats. Now, at this point B gets three list seats and C gets three list seats for a total of 11. Is this correct? --CGPGrey (talk) 19:35, 21 September 2011 (UTC)[reply]
That is correct for the "allow the overhang" approach. Warofdreams talk 09:46, 22 September 2011 (UTC)[reply]

Destinations per airport

Hello,

since traffic seems to fall under economics (by the way, I feel the helpdesk categories are a bit in "questions about giraffes/questions about non-giraffes" territory) I'm asking this here:

Which airport currently has the highest number of destinations served with scheduled passenger flights, domestic and international combined?

Cheers, -M.

--93.220.120.122 (talk) 15:50, 21 September 2011 (UTC)[reply]

World's busiest airport says that Frankfurt has the most international destinations, which doesn't answer your question but might give you some leads. AndrewWTaylor (talk) 15:58, 21 September 2011 (UTC)[reply]

Domestic connections from FRA are limited by the size of the country. My personal hunch is the boring choice ATL. Just wanted to entrust myself to the Wikimind's better wisdom!

--93.220.120.122 (talk) 16:10, 21 September 2011 (UTC)[reply]

Contributors to this forum did some research and found that Frankfurt has the most non-stop destinations overall, including both domestic and international destinations. Neither Atlanta nor O'Hare really came close. It's not so surprising, really. Frankfurt is THE hub for a nation of 80 million people in a part of the world with a greater density of cities than North America, and it serves in addition as a transfer point for people traveling to and from every continent except Antarctica. Marco polo (talk) 18:51, 21 September 2011 (UTC)[reply]

I'd seen that discussion, but it was from 2001. Still, thank you for the effort! I think it's the closest we're going to get.

-M.

--93.220.120.122 (talk) 19:19, 21 September 2011 (UTC)[reply]

Questions on Non-Disclosure Agreements

I have some questions regarding non disclosure agreemenent. They are contracts, correct? If that is so, then do the standard requirements of consideration apply to them? Also, what are the remedies for its violation? Breach of Contract? What sorts of equitable remedies are there? — Preceding unsigned comment added by Rabuve (talkcontribs) 16:21, 21 September 2011 (UTC)[reply]

You may want to read the article titled Non-disclosure agreement. --Jayron32 17:34, 21 September 2011 (UTC)[reply]
And, if you are a party to a specific NDA and want to know what that means for you, then you should consult a lawyer. --Tango (talk) 17:37, 21 September 2011 (UTC)[reply]
Sadly, the article doesn't seem to answer any of the questions raised by Rabuve. Sigh. --Tagishsimon (talk) 17:51, 21 September 2011 (UTC)[reply]
Yes, but there are also external links and references which can be followed. Perhaps the answer lies there. Wikipedia is a starting point, not a destination, for research. --Jayron32 17:52, 21 September 2011 (UTC)[reply]
Hmm. --Tagishsimon (talk) 17:59, 21 September 2011 (UTC)[reply]
But this page is. --93.220.120.122 (talk) 19:20, 21 September 2011 (UTC)[reply]
The consideration is the "valuable" confidential information disclosed, and injunctive relief is always available, with most courts very willing to issue preliminary injunctions. Equitable relief is slightly less common, beyond attorneys fees, and may be up to the judge or jury or both depending on the jurisdiction, if I remember right. But I'm not an attorney so don't believe me without a corroborating lawyer's opinion in the pertinent jurisdiction. 70.91.171.54 (talk) 20:25, 21 September 2011 (UTC)[reply]
Consideration does have to be shown. It's going to vary with the particular agreement and circumstances, but in general, the consideration is the communication of the confidential information, just as 70.91.171.54 says. Typically the owner of the information will seek to be able to obtain an injunction forbidding disclosure of the information; this is a kind of equitable remedy. The owner may also be able to receive cash damages, which is a legal rather than an equitable remedy. John M Baker (talk) 22:54, 21 September 2011 (UTC)[reply]
That's for contractual non-disclosure agreements. With an agreement under statute -- one of the various "Official Secrets Acts", for example -- owner of the information could put you in jail (or gaol, if you prefer) for disclosure. If it's on the statute books, you could probably put away for threatening to disclose information or other inchoate offences. Or are we talking strictly contacts here? --Shirt58 (talk) 03:58, 22 September 2011 (UTC)[reply]
The Official Secrets Act, at least in the UK, isn't an agreement. It's a law. While people talk about "signing the Official Secrets Act", you are subject to it whether you've signed anything or not, just like any other law. The act of signing it (or, rather, a declaration about it) is just a way to make sure people are aware of their obligations. It isn't a requirement for the law to be enforced. --Tango (talk) 14:25, 22 September 2011 (UTC)[reply]
Equitable remedies are only available where there is no adequate remedy at law. That is money is not enough. The equitable remedy article has this ancient legal doctrine burried near the end. The Supreme Court of the United States has ruled that it is a violation of due process to deny a defendant a jury trial when a plaintiff pursues equitable relief when monetary damages are sufficient. I forget the name of the case, but it is from the 19th century. Do a google scholar search for "no adequate remedy at law" to learn more. Non-disclosure agreements could be the subject of a number of causes of action. Breach of contract, invasion of privacy (as was the non-disclosure agreement between Jennifer Lopez and her former husband), interference with advantageous relations or other business torts, false light or other defamation claims, copyright or patent infringement, misappropriation of trade secrets or others. The agreement could be used as the basis of the claim or it could be used as evidence of intent or maliciousness. The agreement must follow contract law, so consideration is necessary. Often a severence package, a paycheck, or continued employment is sufficient consideration. If you were already fired, the non-disclosure agreement may not be enforceable. It is going to depend on the facts and the jurisdiction. Gx872op (talk) 13:49, 22 September 2011 (UTC)[reply]
I think the statements: "We can't really answer your question, as the answer will change depending on the specifics of the case and the legal jurisdictions involved" and "Consult a lawyer if this is more than just curiosity" are the best answers to any legal question. Blueboar (talk) 14:08, 22 September 2011 (UTC)[reply]
The NDAs that I have seen say that the consideration is the receipt of the confidential information in order to evaluate a possible business relationship. Riteota (talk) 15:39, 22 September 2011 (UTC)[reply]

VP of the US

Does the VP of the US have any authority to give orders to persons in the US military? Presumably, POTUS could as the commander in chief but I don't recall that the VP has any official military capacity. Googlemeister (talk) 18:17, 21 September 2011 (UTC)[reply]

Cheney gave orders to the military to shoot down hijacked airliners late in the sequence of events on September 11, 2001. It is not clear either that he had the authority to do so. It is not clear that the military would have carried out his order. Edison (talk) 18:23, 21 September 2011 (UTC)[reply]
The vice president has exactly two constitutional powers: 1) He takes the powers of the presidency if the president dies or is incapacitated, and 2) In the event of a tied vote in the Senate, the vice president can cast a vote to break the tie. Unless the president has died or is incapacitated, the vice president has no more constitutional authority than a street vendor in Cairo to command the U.S. military. Marco polo (talk) 18:40, 21 September 2011 (UTC)[reply]
As a de facto matter, the VP can probably credibly claim that the President's travel during a rapidly unfolding series of events is sufficient "incapacitation" to make his orders authoritative. If I recall correctly, nobody including the President ever objected to or refused the authority of Cheney's orders to shoot down the planes on 9/11 during or after the crisis. There's probably a law review or similar article about it somewhere by now. But I hope to goodness that the military wouldn't let a VP launch an attack against foreign soil or forces without much clearer presidential incapacitation. 70.91.171.54 (talk) 19:06, 21 September 2011 (UTC)[reply]
(edit conflict) Hence John Nance Garner's pithy quote that the Vice Presidency wasn't "worth a bucket of warm piss". In the modern age, the VP has a lot of symbolic power; they are a visible representative on their party, much like the president; they can be given special powers by the president, such as heading a special commission, or appearing in the President's stead to represent the interests of the U.S.. The president can also use the VP as a trusted advisor, to influence executive decisions based on the VPs advice. Not being codified in the constitution doesn't mean there isn't any real power. There's lots of ways power can manifest itself. Lobbyists have no legal standing by the constitution, but they have real power... --Jayron32 19:10, 21 September 2011 (UTC)[reply]
Cheney never had much regard for constitutional legal limits to his power. Sadly, he is not the only government official who has frankly violated constitutional limits with impunity. Marco polo (talk) 20:35, 21 September 2011 (UTC)[reply]
You know, that comment would be good for more than just a contemptuous chuckle if you would point out an actual concrete example of Cheney having done something unconstitutional. μηδείς (talk) 01:34, 22 September 2011 (UTC)[reply]
Apart from the possible theft of two elections and the attempt to command the U.S. military during the hysterical reaction to 9/11, you are right that there are no clear cases of constitutional violations, so I have edited my statement. However, there is plenty of evidence pointing to Cheney's involvement in illegal actions, such as the Plame affair, lying to Congress and the public (as discused in Plan of Attack), his support for "enhanced interrogation techniques" violating the Geneva Convention, and so on. In effect, he is probably guilty of war crime. Cheney himself fears prosecution for war crimes. Marco polo (talk) 16:14, 22 September 2011 (UTC)[reply]
Remember also that the U.S. was under a state of emergency and operating under the (mostly secret, even from Congress) Continuity of Operations Plan plan, not entirely the Constitution. Rmhermen (talk) 02:55, 22 September 2011 (UTC)[reply]
Both Bush and Cheney violated the Constitution by swearing the Oath of Office for offices they weren't legitimately elected to. Or is electoral fraud merely illegal but not unconstitutional? Pais (talk) 07:56, 22 September 2011 (UTC)[reply]
That is far from a "concrete example". I'm assuming you're referring to the 2004 United States election voting controversies? —Akrabbimtalk 13:28, 22 September 2011 (UTC)[reply]
More likely he is thinking of the United States presidential election in Florida, 2000 situation. Googlemeister (talk) 13:42, 22 September 2011 (UTC)[reply]
Can the US president delegate some of those powers to the VP? Separately, I remember being told that in the last days of the Richard Nixon administration, Henry Kissinger quietly told the US generals to check with him (Kissinger) before doing anything drastic that the president might order. He seemed to have been concerned that Nixon would torch the globe in a fit of pique. Is there any evidence this occurred? Riteota (talk) 15:38, 22 September 2011 (UTC)[reply]
I'm not sure he can. There are things the President can delegate to the VP (like "Go represent me at this summit meeting of world leaders"), but the enumerated powers of the President are likely his solely, unless he declares himself incapacitated under the terms of section 3 of the Twenty-fifth Amendment to the United States Constitution, then he can't willy-nilly abrogate his own personal responsibility and authority merely by designating the VP to make the decision. Clearly, he may be within his right to take the advice of the VP, even in an automatic and unquestioning way (Bush was often accused of this sort of relationship with Cheney, especially with regards to matters of national security), but ultimately the President, and not the VP, is ultimately held responsible, in a legal way, for the execution of his constitutional powers, especially including his role as Commander-in-Chief. --Jayron32 16:43, 22 September 2011 (UTC)[reply]
The concept that I've heard is that the White House business model is more or less reinvented with every new administration. My guess is that the President can delegate whatever he (not she, yet, but I have a feeling that will happen sooner or later) wants, but it's still his authority and his neck on the line if something goes wrong; i.e. the President screwed up by letting the VP handle it when he should have found someone more competent/reliable/etc... The only thing the VP is legally responsible for is his rare opportunity at a vote in the Senate, everything else is the President's authority. SDY (talk) 16:55, 22 September 2011 (UTC)[reply]

Railroad workers in gilded age in US

Does anyone know any statistic about railroad workers in gilded age? Anything like wages, hour of work a day, total death toll (anything death that related to railroad work including suicide if caused by railroad), % of death out of total workers, any other statistic would be helpful. Thanks.Trongphu (talk) 18:28, 21 September 2011 (UTC) And statistic on injured workers. Total and %?Trongphu (talk) 18:30, 21 September 2011 (UTC)[reply]

Well during construction of the first US transcontinental railroad, our article states that Central Pacifc laborers worked 8 hour shifts and foremen worked 12 hour shifts. According to http://books.google.com/books?id=xA0XAAAAYAAJ&pg=PA233&lpg=PA233&dq=railroad+construction+wages+in+1860&source=bl&ots=d9Kc8wG7BG&sig=89GlAEZv-Rw7-gMFozduXurYCuo&hl=en&ei=-TB6TuOHJsqAsgL9wYihAw&sa=X&oi=book_result&ct=result&resnum=4&ved=0CCkQ6AEwAw#v=onepage&q&f=false wages averaged $11-12 per week in 1860s Massachusetts anyway. Googlemeister (talk) 18:47, 21 September 2011 (UTC)[reply]

Is there any accident related to railroad that the cause is workers didn't get proper equipment.Trongphu (talk) 19:17, 21 September 2011 (UTC)[reply]

Trongphu, "gilded age" is not a common expression in English. "Golden age" is a familiar expression, but the limits of the period it designates are rather vague. It might be helpful if you delineated the period you are interested in more clearly. --ColinFine (talk) 21:02, 21 September 2011 (UTC)[reply]
Golden age is totally off compare to what i'm talking about. Here, Gilded Age, this is what i'm talking about.Trongphu (talk) 21:06, 21 September 2011 (UTC)[reply]
My apologies. I had never met the expression before, and thought it was a language problem. --ColinFine (talk) 18:56, 22 September 2011 (UTC)[reply]
No problem man, everyone made mistakes all the time so feel free to say anything.Trongphu (talk) 22:28, 22 September 2011 (UTC)[reply]

In the late 19th century, railyard workers in the US switching freight cars had to stand between the cars and drop in a pin at the right instant as the parts of the 2 cars came together. If it failed to drop, they were apt to be crushed between the cars. There was about a death a week in the Chicago switchyards alone. Passenger cars used automatic couplers, which cost more and eliminated the hazard. 50 unnecessary deaths a year in the second largest city, from one cause. Here is a description of one such death, at another location. The loads from the flatcars extended closely enough together that his head was crushed, even though he successfully coupled the cars. There was only 4 feet or so from the ties/ballast to the load, and only 5 inches between the loads of the 2 cars. An appeals court said there was no negligence on the part of the railroad, and the man knew the risk when he took the job. They said he should have crouched under the load between the cars and only raised his hand above the load to drop the pin. Another worker had lost his hand doing such an operation, and again it was not the railroad's fault. See [23] which discussed national legislation in 1891 to require railroads to adopt automatic couplers. The railroad owners preferred to save money on couplers at the cost of limbs and lives lost unnecessarily by workmen. See also [24]. See [25] for discussion of the economics of maiming and killing workmen to keep equipment costs low. In 1889, 300 men were killed and 6757 injured coupling cars. It took years to get the auto couplers installed, standardized, and perfected, so that by 1903, with link and pin couplers almost gone, there were still 253 fatalities and 2788 injuries, since workers had to go between cars to fix problems with the automatic couplers, often with the cars still moving, due to lack of regulations requiring the train to stop before going between cars. Edison (talk) 00:07, 22 September 2011 (UTC)[reply]

As for wages, note that they would have likely paid less to "Coolie laborers" (mostly recent Chinese immigrants). StuRat (talk) 04:05, 22 September 2011 (UTC)[reply]
There are deep methodological problems with uncritical wage comparisons, especially over time, space, gender and race. Outside of the social contexts of gender and race, there are methodological problems in computing different wages across different spaces (cost of living, expectations, what parts of the social income were "commodified" and reduced to wage labour, access to commons, etc), and different times (inflation, declining proportion of social return to labour, etc.) It seems like Trongphu really needs to start reading through the table of contents of labour history and industrial relations journals—there are problems in reading historical death statistics without the context supplied by deep reading. Also, as I've previously noted, most of the studies I'm familiar with seat industrial death and workplace control within the framework of class or industrial conflict, rather than an impartial and universal standard of acceptable industrial death—even social democrats, progressives and liberals note the conflict between labour and management over "management's right to manage" workers into shallow graves. Fifelfoo (talk) 04:46, 22 September 2011 (UTC)[reply]
This [26] paper may be of interest, and a book by the same author is cited. This was a period when the authorities were only just starting to collect systematic statistics. Do heed Fifelfoo's warnings about hasty comparison. Itsmejudith (talk) 14:04, 22 September 2011 (UTC)[reply]

Alright, thank you everyone for useful information.Trongphu (talk) 22:28, 22 September 2011 (UTC)[reply]

Chiropractic in Medicare

When did the US government first include chiropractic coverage in Medicare? Chiropractic history (which is currently up for moving; it might help if you'd offer an opinion about that proposal) and Chiropractic controversy and criticism mention Medicare, but only in passing, and Medicare (United States) doesn't mention chiropractic at all. I'm working with congressional papers from 1971 and 1972, and I've come upon tons of letters from people to their senator, urging him to vote for a bill that would include chiropractic coverage in Medicare for the first time. I don't know whether the bill passed, since I'm not concentrating on this subject. Nyttend (talk) 21:32, 21 September 2011 (UTC)[reply]

42 U.S.C. § 1395X (the Social Security Act) §§ 1861(r)(5) seems to have been enacted in 1965, but someone please double check that it was in the original bill, because we both have reason to believe it might not have been. You might need a law library for the history of the statute, or maybe just more time to google around for it than I have at the moment. 70.91.171.54 (talk) 22:54, 21 September 2011 (UTC)[reply]
Everybody that wrote on the issue (average people, medical doctors, Readers' Digest, and chiropractors) agreed that chiropractic wasn't included in Medicare; while the medical doctors and the magazine may not be as significant, you'd think that significant numbers of would-be chiropractic patients would know that they couldn't get compensated for their chiropractic visits, and I can't imagine 100% of any significantly-sized group of chiropractors incorrectly believing that they couldn't be compensated for their work. Nyttend (talk) 03:30, 23 September 2011 (UTC)[reply]

Outcry stopping an execution in the US?

Has the popular and media outcry, Pope and EU intervention, ever stopped an execution in the US? Quest09 (talk) 22:46, 21 September 2011 (UTC)[reply]

Yes, about 268 times since 1976 but some of those commutations involved more or less public pressure than others. 70.91.171.54 (talk) 22:52, 21 September 2011 (UTC)[reply]
That's a surprise answer. I'd expected something like none. Quest09 (talk) 23:08, 21 September 2011 (UTC)[reply]
Given that many of those with power to stop executions depend on public support to be elected, I would expect plenty. HiLo48 (talk) 00:12, 22 September 2011 (UTC)[reply]
Well clemency has never been politically popular with the silent majority. Governor George Ryan granted clemency to all of Illinois' Death Row prisoners on his second-last day in office because of all of the controversy over the death penalty, but he couldn't have gotten away with that had he been running for re-election. -- Mwalcoff (talk) 01:30, 22 September 2011 (UTC)[reply]
Well, they do have other options, like delaying the execution or commuting to life imprisonment (which could also lead to a new trial and possible release, if new evidence appears). StuRat (talk) 04:01, 22 September 2011 (UTC)[reply]

Would a 6'3", relatively strong, 252 lb and aggressive white male in his mid-twenties be likely to be raped in an American prison?

Just asking. Are there any sources you can draw on for this.--Sharon and girls (talk) 23:09, 21 September 2011 (UTC)[reply]

Depends on how much respects he gets and how many dudes are bigger. If he doesn't play his cards right, he might get shanked or gang-raped in the shower, never know. Sir William Matthew Flinders Petrie | Say Shalom! 23:17, 21 September 2011 (UTC)[reply]
Prison rape is much less common than fiction might make us believe. And it's even less common among those not serving a life sentence and those who are not effeminate.88.8.79.204 (talk) 23:29, 21 September 2011 (UTC)[reply]
Prison rape in the United States suggests the prior probability is about 2%, and presumably being above average sized would tend to decrease that probability, but the particular prison, cell block, sentence, crime, and the inmate's predisposition along a number of psychological dimensions might make a bigger difference. 70.91.171.54 (talk) 23:32, 21 September 2011 (UTC)[reply]
I might just point out that that number seems to be reported rapes. That's necessarily going to be smaller than those which have actually occurred. The difference between those two numbers may be quite amplified in a prison situation. Even in the non-prison world, rape is one of those crimes that is heavily underreported. So I'm not sure anyone knows what the actual incidence is. It's probably a lot less than 10%, but probably a bit more than 2%. --Mr.98 (talk) 12:07, 22 September 2011 (UTC)[reply]
A prison inmate who was brought to a criminal psychology class told us "You WILL be tested to see what they can do to you. It is not a question of IF." It's confusing when 88.8.79.204 implies that if you are in for life (murder, repeated felonies, serious offenses) that you are more likely to be raped than if you are doing 2 years for check fraud. Gang membership is said to be somewhat protective, at a price. The tall "relatively strong" fat guy would seem less at risk than a willowy and effeminate little guy. Edison (talk) 23:55, 21 September 2011 (UTC)[reply]
Oh it's not a case of being "relatively strong". You see, I am not even American. And if you think I am fat I am slightly fat but I have big muscles which I have gained over the years from work and the gym and that has been what has given me my weight for the most part.--Sharon and girls (talk) 00:33, 22 September 2011 (UTC)[reply]
Well Sharon, what gaol were you thinking of being locked up in specifically? Sir William Matthew Flinders Petrie | Say Shalom! 00:47, 22 September 2011 (UTC)[reply]

No, if he gave in first he would not be raped. μηδείς (talk) 00:41, 22 September 2011 (UTC)[reply]

I don't think that is logically true. Wouldn't it still be rape if beyond a certain point—of being beaten up—that one "gave in"? There are physical forces beyond anyone's ability to withstand. There are always stronger people than anyone, and additionally one can be outnumbered by multiple adversaries. Bus stop (talk) 00:57, 22 September 2011 (UTC)[reply]
The OP's inquiry raises a lot of questions, the first of which, to me, is, well, does he want to be raped, or doesn't he? I have heard plenty of stories about murder in jail and plenty of stories about homosexual sex, of the eager and desperate and situational versions, directly from inmates of Rikers and Sing Sing with whom I have had personal relations and elsewhere. But I have never heard an actual prison rape story. Those seem mostly to be the fantasies of fantisizers and fantasists. The way this question is asked makes me suspect the desired answer to will he be raped is...yes. μηδείς (talk) 01:46, 22 September 2011 (UTC)[reply]
Without knowing much about your friends, I would guess many people are far more willing to discuss how they had consensual sex, even if it was with someone of the same sex and they don't consider themselves gay, then they are willing to discuss how they were raped. Some rapists may enjoy boasting about their rapes, others may not, particularly if they're partially reformed and may prefer or even had deluded themselves in to thinking their rapes were entirely consensual sex. As for murders, again I don't know the context for these. But a murder does tend to be more public then a rape so it's likely more people would be aware of these even if the number that occured was equal. Also again without knowing the mentality of the people you've had discussions with, I think people often prefer to turn a blind eye to things which make them uncomfortable and in many cases this applies to rape more then murder. Also it's generally far harder to presume a murder was an accident or suicide then it is to presume a rape which you hear about or witness was actually consensual sex. More generally, it's commonly presented on TV and in movies that some people will semi-consent to sex with one person, with the desire to gain their protection and ensure they aren't raped by anyone else (and possibly the person they 'consented' to). How often this occurs in real life I have no idea. But while I don't know what you should call these, I don't know if they should be called consensual either. Nil Einne (talk) 06:26, 22 September 2011 (UTC)[reply]
In addition to the above excellent responses, I think it's important to emphasize that the person in question should, if he can, avoid going to jail, and so avoid this risk altogether. Jail has many detriments beyond the risk of prison rape, such as a complete loss of personal freedom. Dcoetzee 07:31, 22 September 2011 (UTC)[reply]
Here's a 2006 study, refuting the media's view on the prevalence of actual prison rape, but confirming that homosexual relationships do exist, though it is regarded as situational and understandably kept quiet when they get free.
Prison rape does exist though, but I agree that it would probably be quite rare. Even though being the unwilling victim of one and thus keeping quiet out of shame shouldn't be discounted either. Crazily enough, the victim is almost always regarded as the gay one, and the rapist as the "real" man merely giving in to his sexual urges. And then there's the propensity for legislators to regard any kind of homosexual contact in prisons with suspicion, and view it as rape. Some previous studies included inmates who actually prostituted themselves along with the victims of actual rape.
So no, an inmate most probably won't get raped, but depending on how long he stays there or how lonely he gets, it's likely he'll seek some sexual outlet that involves another person. Be it prison staff of both sexes or another male inmate. But it is not necessarily rape.
Here's a bibliography (with links) if you want to know more about the different studies on it. This paper also discusses the possibility of the reports of rape merely being attempts to reassert heterosexual identity, thus painting what may actually be consensual sex as a violent attack to distance themselves in postprison life.-- Obsidin Soul 08:27, 22 September 2011 (UTC)[reply]
An article I read but do not have a link to said that when youthful offenders are put in jail for minor crimes and commit suicide (as by hanging themselves in the cell) it is often because they have been raped and fear repeated rapes. I'm not sure how to search for the article without turning up tons of sites I would just as soon not see. In the US, it seems odd that "strict law and order" figures in fiction often seem glad that straight offenders will get gang raped in prison, despite the authority figures disliking homosexuals. In this schema, the prison is a place where gang lords and the most vicious criminals are rewarded for their incarceration with victims for their lust. In real interrogations police supposedly threaten suspects with the prospect of some horrific cellmate raping them, to get information or confessions, in a way reminiscent of medieval inquisotors "exhibiting the instruments of torture" to get confessions or information on others. Edison (talk) 17:52, 22 September 2011 (UTC)[reply]
To wander off topic onto the subject of youthful offenders, some statistics are in Beck, Allen. J; Harrison, Paige M.; Guerino, Paul (January 2010), Sexual Victimization in Juvenile Facilities Reported by Youth, 2008-2009, U.S. Department of Justice. (Not sure why I don't have a link to that, I'm sure it's available online.) I think it describes roughly 10% of inmates as reporting unwanted sexual contact from a staff member within the last year involving use of force, and roughly another 10% without use of force. Reports of unwanted sexual contact from other inmates were much lower. Very strange figures, perhaps I misunderstood them in some way. --Demiurge1000 (talk) 18:41, 22 September 2011 (UTC)[reply]

Where to find companies whose stocks trade on more than one stock exchanges ?

Hi, All:

where to find companies whose stocks trade on more than one exchange ? For example, some companies, whose stock maybe traded both on European (London) stock exchange and Nasdaq.

I am particularly interested in finding out these Asian and African companies whose stocks are traded on more than one stock exchanges.

Thank you. — Preceding unsigned comment added by 199.33.32.40 (talk) 23:28, 21 September 2011 (UTC)[reply]

So that you can buy low on one and sell high on another? It would seem that were that possible someone would already have thunk it.μηδείς (talk) 01:50, 22 September 2011 (UTC)[reply]
Not only has someone thought of it, they have a word to describe the practice. See arbitrage. --Jayron32 02:08, 22 September 2011 (UTC)[reply]
Cool. I remember that word from the 80's. See: http://en.wikipedia.org/wiki/Arbitrage#Dual-listed_companies μηδείς (talk) 03:13, 22 September 2011 (UTC)[reply]
I'm just attempting to address the question, don't mind me. (They have a word for 'arbitrage' because it exists and is practiced, across asset classes, actually.)
The original poster should read American Depositary Receipt and look at the external links. For example, this link to Deutsche BAnk's Depository Receipt Services, which has a detailed search tool. Riggr Mortis (talk) 03:34, 22 September 2011 (UTC)[reply]

September 22

British Empire & Colonies

Obviously there were many reasons for the British Empire to have colonies around the world, defence, trade etc

My question is - Did colonies pay taxes to Britain or did Britain subsidise these colonies?

And if taxes were paid, when did this process end? - I find it very unlikely that Hong Kong would have still been paying up until the hand-over in 1997!

I have read the articles on the History of the British Empire, Hong Kong & Singapore, but I can find no information on whether these colonies paid taxation to Britain.

And just exactly how much control did Britain have over these colonies? - in 1983, when the Hong Kong dollar was pegged to the US dollar, why wouldn't Britain have stipulated it should be pegged to Sterling (like other colonies, such as Gibraltar etc) — Preceding unsigned comment added by Jaseywasey (talkcontribs) 08:33, 22 September 2011 (UTC)[reply]

I can’t speak for other colonies, but Hong Kong has not paid taxes to the UK ever, to the best of my knowledge. In fact, it was the other way around: military forces in HK were paid for by the UK.
How much control did the UK have? Lots. The HK Governors, like the Chief Executive today, has a much larger degree of power than prime ministers or presidents in most democracies, and because they are appointed by the UK, they generally had to give at least outward support to British policy preferences (but, not always: see John James Cowperthwaite for an example of a financial secretary who wasn’t about to introduce the welfare state to Hong Kong).
Finally, the peg. Fixed exchange rates, particularly for small, open economies, are best fixed to the currency in which the largest share of trade is conducted. That has been the US dollar for many decades. DOR (HK) (talk) 09:34, 22 September 2011 (UTC)[reply]
It varies a lot. The North American colonies famously paid taxes. Imperial India paid tax, and was a market for British goods, while its products and riches were taken under less than scrupulous conditions. Other colonies are still kept for their military value, and therefore will be subsidised if necessary (South Atlantic territories like Tristan da Cunha are obvious examples, although today there is the possibility of oil too - and Economy of the Falkland Islands says the Falklands do quite well if you discount defence spending). --Colapeninsula (talk) 13:14, 22 September 2011 (UTC)[reply]
Yes, many British colonies did pay tax - they were British as far as the Crown was concerned. In America and Australia, security and police forces, etc., were all provided by the UK (at first), so it only stands to reason that they should be paid for, and paid to the UK. India was a different story - that was merely occupation and force-feeding of British trade goods at high prices, whilst taking everything India had to offer, and then charging tax. As we all know, Britannia waives the rules..... KägeTorä - (影虎) (TALK) 13:29, 22 September 2011 (UTC)[reply]

Of course there was No Taxation Without Representation - however I was thinking more of the colonies that achieved independence during the 20th Century - Before WW2, these smaller countries would have had the benefit of a super-power defence force, which may have kept ambitious neighbours in check. However after 1945, Britain was virtually bankrupt and poor third behind the USA & USSR in the military "pecking order".

It seems to me that, there WAS a benefit for countries such as Jamaica, (in fact any Caribbean nation), Malta, Singapore etc. - (HK is a slightly different case due to the 99-year lease). These nations had another country responsible for their military defence, but didn't have to pay any taxes?

Whilst I appreciate that most countries would prefer self-determination, I can see the logic for Britain happily allowing this to occur (one a stable, non-communist government was in-place). In fact I'm surprised that the UK didn't encourage independence for any non-strategic nations, with limited natural resources. Jaseywasey (talk) 15:03, 22 September 2011 (UTC)[reply]

Don't kid yourself. There are plenty of ways to bleed a colony dry without having to ask it to pay taxes. In fact, being merely taxed would have been a blessing in some cases. It was only when imperialism started getting bad rap that colonial powers started to put a little bit more effort into giving something back for their colonies. And even then, it was for the benefit of the colonial government, not the locals. Those defense forces you mentioned were not defense forces, but occupying forces meant to keep the locals in check, and yes they are expensive. In the WW2 example, colonial powers left their colonies to the mercy of the Japanese without much hesitation. The only real danger of invasion came from imperialists in the first place. -- Obsidin Soul 15:24, 22 September 2011 (UTC)[reply]
"There are plenty of ways to bleed a colony dry without having to ask it to pay taxes" - like what? And why would the mother country want this? Riteota (talk) 15:35, 22 September 2011 (UTC)[reply]
Some of the many ways to do this include handing over land to companies and individuals from the parent country and allowing those companies and individuals to farm and mine that land without compensating the people who lived there and who owned it until it was forcibly taken from them. Why would the parent company want this? It enriched their citizens, it provided inexpensive supplies of strategic raw materials to the parent country, and so on. Another method is to sell manufactured goods from the parent country at an initial loss until the local manufacturing sector is weakened or destroyed, then to raise the price on these goods once the colony has become a captive market. Yet another method is to impose a monopoly on a key good needed by the local population and require them to buy this good from firms or individuals based in the parent country, for which this trade became a source of profit at the expense of the colony. Gandhi's Salt Satyagraha was aimed a such a practice. Marco polo (talk) 15:47, 22 September 2011 (UTC)[reply]
Why do you suppose so many colonies fought so fiercely for independence even if it meant economic collapse? Colonial powers ('mother countries', LOL) were not motivated by altruism. They were motivated by profit. They didn't give a rat's ass about the natives. Defensive armies, LOL. Indeed, to some, they were merely indigenous animals.-- Obsidin Soul 16:05, 22 September 2011 (UTC)[reply]

Post-WW2, I don't recall many colonies fighting fiercely (except India, but not necessarily "fighting")

My initial question related to HK - as far as I can see this territory was a "net-gainer", albeit it meant that UK had a military base, right on China's doorstep.

But what about countries like Brunei, a British protectorate until 1984........in 1976 Jim Callaghan had to go "cap-in-hand" to the IMF for a loan to bail-out Britain, when the country was virtually bankrupt........the Sultan of Brunei was at that time the richest man in the world......do you see where I'm going here......? Jaseywasey (talk) 17:14, 22 September 2011 (UTC)[reply]

In this context, you might also puzzle why "the Government and people of the Federated Malay States" chose to contribute a vast sum of money to buy HMS Malaya for the Empire in 1913. --Demiurge1000 (talk) 18:25, 22 September 2011 (UTC)[reply]
Let me explain why all this is more than a little insulting to people who were once under colonial governments (like mine), and I'm not even nationalistic. You see, this isn't a new idea. In fact, it was the justification used for our second invasion - The White Man's Burden, when America caught the imperialism bug. It was quite brutal and extremely unscrupulous but largely forgotten or ignored in American history today. To claim that establishing colonies were motivated by altruism is more than a little outrageous. You can not merely ignore centuries of blatant exploitation and then claim that you protected them while they gave nothing back in the last decades of their exploitation. You can only justify that if you conveniently forget the events leading up to that situation in the first place. And yes, the UK did in fact encourage decolonization after their economic collapse after WW2. Partly because attempts to prevent it by other countries resulted only in more losses (like the First Indochina War).
Protectorates/protected states were not freeloaders for much of their histories. They had economic importance to the protector. They were to be 'protected' (i.e. puppet governments established or commandeered) in exchange for serving as hosts to colonial economic interests (not merely as places for military bases). Brunei wasn't the tiny state that it is now. In fact, establishing a protectorate was the least Britain could do after they so much as stole an entire island from the Sultanate. See History of Brunei#Relations with Europeans and British Empire#Decolonisation and decline (1945–1997). -- Obsidin Soul 18:26, 22 September 2011 (UTC)[reply]
I feel the need take Obsidian to task over his charge that the European powers left "their colonies to the mercy of the Japanese". The Dutch had two cruisers in the East, both were sunk at the Battle of the Java Sea. Like Britain, the NL only maintained sufficient forces for internal security in her Asian possessions. There was no backup as the rest of the Dutch army had been destroyed by the Germans. 1,300 defenders of Tarakan were all executed by the Japanese after they surrendered. 1,100 defenders of Balikpapan were all killed or captured. The same story repeated all over the Dutch East Indies. Britain and Australia poured troops into Singapore; 134,000 were killed or captured. 840 sailors were killed when a British naval task force was destroyed. Hong Kong, defended by a Canadian brigade, fought on for 2 weeks outnumbered 4 to 1. Not an effective defence but much blood was spilt in the effort. Alansplodge (talk) 19:15, 22 September 2011 (UTC)[reply]
I was talking specifically of Plan Dog memo. But yeah, that was understandable enough and I concede you have a point. Though I still do not see how it would prove Jaseywasey's assertion that the presence of colonial forces were intended primarily to provide defensive benefits for the territories involved at no cost to their subjects. They defended them, but that does not mean that was their reason for colonizing them in the first place.-- Obsidin Soul 21:08, 22 September 2011 (UTC)[reply]
Yes I agree that empires were not there to protect the colonised; but it was the intention of the colonial powers to protect their colonies and the inhabitants of them, but it could not be realised in 1941/42. Thank you for drawing my attention to Plan Dog, however this was a US creation. Reading the war diaries of Alan Brooke, I get the impression that Britain hadn't fully appreciated the Japanese threat until it was too late to do anything but the rushed response that failed so miserably. Alansplodge (talk) 22:39, 22 September 2011 (UTC)[reply]

I really hoped this wouldn't descend into an argument about the "rights & wrongs" of Imperialism. My main query related to post-WW2 HK - this country became very successful whilst Britain was virtually bankrupt. HK has famously low taxation, whilst in Britain during the 70's income tax reached 83% - I appreciate that after WW2, America was "anti-colonialism", however they did concede that the colonies prevented the spread of communism and were therefore tolerated. I think it is reasonable to say, due to a massive change in public opinion, colonies changed from "cash-cows" to financially independent states with increasing levels of autonomy and Britain realised that rather than being seen as assets, they were now liabilities (having the responsibility of defending countries that no longer "pay for themselves"), hence the policy of "peaceful disengagement" Jaseywasey (talk) 06:45, 23 September 2011 (UTC)[reply]

I'm sorry, but the implications of this question itself is inflammatory. It's virtually repeating the colonial mentality that these territories were little more than, as you said, cash-cows that were to be taken advantage of, and once used up, thrown away. Brunei, itself, at the state that it had been reduced to due to British expansion into Malaya, wouldn't have been able to survive as a sovereign nation if it wasn't for the discovery of oil. Why would they be obligated to pay even more?
The UK was honoring old treaties with both Brunei and HK. They couldn't simply throw them away and sever even more diplomatic ties already frayed by WW2. A move like that would have destroyed their credibility completely. The UK did, as already mentioned, let go what they can peacefully, even insisted on it (HK was supposed to be theirs for eternity). For what it's worth, British Overseas Territories are still notorious for being tax havens, notably Bermuda. As for control, see Crown colony.-- Obsidin Soul 17:14, 23 September 2011 (UTC)[reply]

Palestinian state

Palestinian leaders are apparently going to ask the UN to declare Palestine a full UN member state, or something similar. Israel's public statement against this has been that the proper way to attain this goal is to continue peace negotiations with Israel; and that the UN is not the proper venue for this declaration.

Maybe I am missing something on this last point and someone could help. The UN was the exact venue by which Palestine was officially partitioned in the first place and the state of Israel was created. Isn't the Israeli claim (that the UN is not the right venue for this) hypocritical right on its face?

I obviously don't want to start a flame thread here and am not interested in the deeper arguments over the validity of either Israel or the Palestinian state, or who is in the right on this particular issue; I just object to an argument that is so easily countered. What am I missing? Riteota (talk) 15:33, 22 September 2011 (UTC)[reply]

While I am sympathetic with the Palestinian perspective on this issue and see no reason why they shouldn't declare their independence, I will confine myself to explaining the Israeli position. Israel's point is that declarations in the United Nations will not resolve the conflict, because they are not going to influence Israel's actions. (They haven't in the past; why should they now?) The only way for the Palestinians to change the facts on the ground is to engage directly with Israel. Actually, I think it's hard to dispute that, even if engagement with Israel hasn't improved the situation of Palestinians much in recent years, for reasons that are outside the scope of your question. Marco polo (talk) 15:42, 22 September 2011 (UTC)[reply]
Israel feels that the United Nations is not the proper forum in very large part because of the flagrant and widespread anti-Israel bias frequently displayed at the UN -- starting as early as 1961, when Israel was the only member country to be excluded from the UN regional groups that were created at that time, and which have become very important in the internal workings of the UN (Israel still only has provisional membership in WEOG in New York, but not in Geneva or Vienna), and intensifying markedly in 1975, and continuing through the annual ceremonial General Assembly "Two-Minutes-Hate-Against_Israel" meetings (which neither the United States nor Israel has bothered to even attend for many years) and on into the 2000s with the infamous 2001 Durban conference for the promulgation and promotion of racism and the apparently very intentional and deliberate decisions among high UN officialdom to maliciously appoint individuals to prominent middle-east relevant positions such as Cornelio Sommaruga and Richard Falk who are personally completely unacceptable to Israel. The United Nations Commission on Human Rights was abolished in significant part because of its extravagantly flamboyant brazen flagrant hypocrisies (many having to do with Israel), but its successor, the United Nations Human Rights Council, really hasn't done much better... AnonMoos (talk) 18:44, 22 September 2011 (UTC)[reply]
P.S. See Durban III for something going on right now which 14 countries (including the United States) have boycotted over fears that it will devolve into an officially UN-sponsored anti-Israel hatefest (with antisemitic overtones)... AnonMoos (talk) 19:09, 22 September 2011 (UTC)[reply]
Being in conflict is not a valid reason to disqualify a country from UN membership. Under such a guideline the USA would never have qualified. Saying the UN shouldn't have a say because you don't like the way it votes is like an American state saying the Senate shouldn't have a say for the same reason. HiLo48 (talk) 20:45, 22 September 2011 (UTC)[reply]
Umm, if that's in reply to my remarks, it doesn't seem as though you read them very carefully... AnonMoos (talk) 21:51, 22 September 2011 (UTC)[reply]
A more sensible comparison would be the Libyan transitional government, which is intervening in an "internal" affair of the Libyan state. In that case, though, like the switch from recognition of Republic of China vs. People's Republic of China, those are a "one for one" swap instead of a "one for two" swap. The UN doesn't have much authority in the internal affairs of its members, and there are probably odd questions about how occupied territories are handled. Given that Israel more or less ignores the UN anyway (for reasons stated above), the Palestinians appealing to the UN doesn't make much difference. The UN doesn't authority beyond trying to convince its members, and the actual powers either support Israel in the dispute (e.g. the US) or haven't shown much interest in getting involved (e.g. China). Even China might quietly oppose this, since they don't want Taiwan getting any ideas about methods for declaring independence. SDY (talk) 22:47, 22 September 2011 (UTC)[reply]
AnonMoos has answered this question eloquently. I think it's also worth pointing out that the circumstances are quite different from 1947. Back then, the UN was suggesting what to do with the Palestine Mandate after its approaching expiration. The plan was rejected by the Arabs, and in the end Israel won its independence on the battlefield. Today, all sides theoretically support the concept of a Palestinian state; the question is how it should happen. -- Mwalcoff (talk) 23:23, 22 September 2011 (UTC)[reply]

Two simple equations: Israel + Palestine + uninvolved people = shit ton of drama. Israel + Palestine + uninvolved people + Internet = (shit ton of drama)3. I am saying out of what will inevitably become a long drawn-out, annoying and (in the end) pointless debate. Sir William Matthew Flinders Petrie | Say Shalom! 23:36, 22 September 2011 (UTC)[reply]

Your equations as written are unsolvable, which is what I should have expected. Googlemeister (talk) 13:42, 23 September 2011 (UTC) [reply]
This video is a must-see for anyone interested in the history of the Israel-Palestine conflict. 67.169.177.176 (talk) 20:44, 23 September 2011 (UTC)[reply]

The illegitimate children of noblewomen

What was the status of the illegitimate children of female members of the nobility? This is an historical question and refers to Europe pre-1900. It is not very hard to acquire information about the illegitimate children of noblemen, but it is harder to confirm such children of noblewomen. The former was perfectly acceptable, while the latter was considered a disgrace and hidden. This is of course about unmarried noblewomen, as the children of a married woman were assumed to be the children of her husband.

What I wonder about is: did such children count as nobles? The situation was after all different. Illegitimate children to noblemen could be acknowledged or not by their noble father, while the illegitimate children of a noble mother were undoubtedly her children.

If both the parents of illegitimate children were noble, did the children count as noble? If the mother were noble and the father not, did the children count as noble?

The question above, I suppose, is rather of a legal kind, but I also wonder about the place of these children in the social attitude of people. Which social position did they have? Did the attitude toward them differ in any way from the attitude to illegitimate children of male nobles?

I have the impression, that illegitimate children of two nobles were treated fairly well, better than illegitimate children of male nobles and non-noble women. It also seem to me, that illegitimate children of noblewomen and non-noble men were in fact treated somewhat better, when it comes to social attitude, than did illegitimate children of male nobles and non-noble women. Perhaps because there was never any question as to whether they belonged to the family, while a noble father could always deny them? But this is just an impression. I realize this is a difficult question. Thanks! --85.226.43.106 (talk) 16:41, 22 September 2011 (UTC)[reply]

An interesting question to this is also: which name was the child given? The illegitimate child of a nobleman could be given his name if he acknowledged the child. Was the illegitimate child of a noblewoman automatically given her noble name, because she could no deny it as hers? And could the child have a noble name without being considered noble?--Aciram (talk) 17:00, 22 September 2011 (UTC)[reply]
Sometimes, a name may be invented for them. The last name Fitzroy (Anglo-Norman for "Son of the King) was frequently given to illegitimate sons of various English kings, borne most often by noblewomen (because they were who was availible to have illegitimate fun with). --Jayron32 17:04, 22 September 2011 (UTC)[reply]
But those were the children of noble/royal males as well. My question is about illegitimate children of noble females, in particular to children of noblewomen and fathers of the same or lower status than the mother. --85.226.43.106 (talk) 17:29, 22 September 2011 (UTC)[reply]
@Jayron32, I do not think royal men were restricted to only noblewomen to have illegitimate sex with: royal men could perhaps only have noblewomen as their official mistresses, but they could surely have sex with as many female servants and peasant girls as they wished, although this was not recognized and the children in those case were not as often acknowledged. What you say about the name is true, but what would be the case of a name for a noblewoman and a male servant? Did the child get her surname? The name "Son of the King" also describe the child's father as the high status parent, so therefore that case is more about the father than the mother. If it was the mother who had the higher status, then what?--Aciram (talk) 17:33, 22 September 2011 (UTC)[reply]
@Aciram: Please be aware that the phrase "most often" is not identical to the word "always". It appears, from your response, that you didn't recognize the difference. I will concede that a better word may have been "sometimes". But I did not imply in any way that Kings didn't do the deed to non-noble people as well. --Jayron32 17:42, 22 September 2011 (UTC)[reply]
At common law, there was no operation to legitimize a child other than by act of parliament. I provide a quotation form Lord William Blackstone on bastards, writing in the middle of the 18th Century and reflecting upon the law as it existed during the reign of Richard II of England:

I proceed next to the rights and incapacities which appertain to a bastard. The rights are very few, being only such as he can acquire; for he can inherit nothing, being looked upon as the son of nobody; and sometimes called filius nullius, sometimes filius populi. Yet he may gain a sirname by reputation, though he has none by inheritance. All other children have their primary settlement in their father’s parish; but a bastard in the parish where born, for he hath no father…The incapacity of a bastard consists principally in this, that he cannot be heir to any one, neither can he have heirs, but of his own body; for being nullius filius, he is therefore of kin to nobody, and has no ancestor from whom any inheritable blood can be derived. A bastard was also, in strictness, incapable of holy orders; and, though that were dispensed with, yet he was utterly disqualified from holding any dignity in the church: but this doctrine seems now obsolete; and, in all other respects, there is no distinction between a bastard and another man. An really any other distinction, but that of not inheriting, which civil policy renders necessary, would with regard to the innocent offspring of his parents’ crimes, be odious, unjust, and cruel to the last degree: and yet the civil law, so boasted of for its equitable decisions, made bastards, in some cases, incapable even of a gift from their parents. A bastard may, lastly, be made legitimate, and capable of inheriting, but the transcendent power of an act of parliament, and not otherwise: as was done in the case of John of Gant’s bastard children, by statute of Richard the second. (italics in the original)

Both England and the United States reformed these laws finding they were unfair and with the later, a violation of equal protection under the 14th Amendment to the U.S. Constitution. What has endured into modern times is the irrebutable presumption that a child born of a marriage is the legitimate child of the marriage. As of 2011, a man claiming to be the father of a child born in a marriage cannot sue for a paternity test and custody, even if the parents later divorce after the child is born. In quoted text above, filius nullius means "son of no one," filius populi is "son of the people." There was no adoption at common law except by statute, but the practice existed in Rome. As you can see from the passage, the law protected one's birthright. You can imagine the chaos that could errupt if an arranged marriage produced illigitimate offspring when the families had intended that their properties be divided or united in a particularly way. The law did not provide for a distinction among illigitimate children based on the status of the parent, male or female, unless of course, it was the King making the rules as was the case with Richard II. Gx872op (talk) 18:07, 22 September 2011 (UTC)[reply]
That was very helpful, and it answers the legal part of the question, at lest when England is concerned. Thank you! Was it the same way in the rest of Europe, such as France and the Holy Roman Empire?
The other part of my question, however, is more a question about social attitude rather than law. It seem to me that "bastards" of women, that it to say children of noblewomen and men of the same or lower rank, were regarded in a somewhat different way than the when the man was noble. Did the sight upon them differ within society? No illegitimate children may have been legally noble, but which status did they have informally, when it came to the attitudes of society? Did the illegitimate child of a noblewoman get to use her surname? Are there any examples of illegitimate children of noblewomen? And I am referring to noblewomen rather than royal women. this question is truly very interesting. --85.226.43.106 (talk) 20:38, 22 September 2011 (UTC)[reply]

Why are the states of Alaska and Hawaii usually ignored?

Many U.S maps and U.S maps that appear on TV completely leave out Alaska and Hawaii as if they weren’t part of the U.S. The Weather Channel, the meteorologists of the major American news outlets, and the meteorologists of news outlets oversees (I’ve been overseas enough times to notice it), only give the weather of the 48 contiguous states, but they always ignore Alaska and Hawaii. The American news media and the worldwide media only report the news about what’s going on in the 48 contiguous states, but the exceptions are when Obama goes to Hawaii and if Sarah Palin goes to Alaska. As a result of the lack of attention that Hawaii and Alaska are receiving, many people around the world and even many Americans don’t have a basic understanding of these 2 states. I actually know a friend of a friend who went to Alaska years ago and all the time he was there he was looking around for some currency exchange place to exchange his dollars to whatever he thought Alaska’s currency was. Later, he found out that Alaska is a U.S state after his friends made fun of him. I found out that this kind of stuff, plus other stuff, seems to be quite common. I came across an eye opening thread that was started by an Alaska resident who complained about the same thing I’m asking about. Why are the states of Alaska and Hawaii usually ignored? Would be correct to call them “the forgotten states” of America? Willminator (talk) 17:19, 22 September 2011 (UTC)[reply]

Alaskan residents also get a little pushy when you use the term "CONUS" to mean "Continental United States" instead of "Contiguous United States." The "otherness" of AK and HI is even enshrined in US government policy, with both of these states treated in some administrative capacities as territories. For example, per diem rates in AK and HI are set by the Department of Defense rather than the General Services Administration. It's probably a legacy of administrative decisions made during WWII, which predated those two getting their stars on the flag. SDY (talk) 17:32, 22 September 2011 (UTC)[reply]
(edit conflict) Because they are discontiguous and some distance away from the rest of the U.S. Hawaii is almost as far from California as California is from the East Coast, while Alaska is so large that its extreme points are as far from each other as the extreme points of the "lower 48". Maps of France frequently ignore places like French Guiana and Réunion despite the fact that the relationship between those places and Metropolitan France is identical to the relationship between Alaska/Hawaii and the "lower 48" in the U.S. Many maps cram an out-of-scale map of both Alaska and Hawaii as insets into U.S. maps. For certain applications, like on the Weather Channel (which does cover Alaska and Hawaii as part of their regular round-up, I should say), including Alaska and Hawaii in their "national map" doesn't make much sense because the weather affecting the lower 48 often doesn't have any connection to that of Alaska and Hawaii; for various reasons you'd want such a map to be in scale (to show accurate, large scale weather patterns) and such a scale map would be impractical, to say the least. As far as Americans not understanding their own geography, it is a well documented problem which is not at all limited to Alaska and Hawaii. I come from New Hampshire, which I can confirm that many people from other parts of the country don't know exists, or if they do, don't know its a state. And it's on all the maps.--Jayron32 17:37, 22 September 2011 (UTC)[reply]
  • (EC)Per recent discussion on the Ref Desk talk page, when a question contains a false premise, and has the potential to become a debate, the thread should probably be preemptorily closed as was another recent thread. Do you have references discussing this phenomenon, or screen captures, or Youtube links? Here is a map from CBS News which shows all 50 states. Here is a map from NBC news which shows all 50 states. Here is a Fox News map showing all 50 states.Here is a CNN map showing all 50 states. Sometimes they tuck the last two states in the lower left corner. If a map relates to something which only happens in the old 48, then it might make sense to only show those states. Edison (talk) 17:41, 22 September 2011 (UTC)[reply]
Edison, I did not say that Alaska and Hawaii never appear in U.S maps, I only said that in many of the U.S maps I've seen these states don't appear at, not even in the lower left corner. That's what I've observed. Edit: Here's one example, here's another, and another. Willminator (talk) 18:10, 22 September 2011 (UTC)[reply]
Your first example. "Mrs. Fralish's class" has a lot of links. Do you seriously expect the Ref Desk volunteers to defend or to explain the choices of some obscure school teacher from "Auburn High School?" I would be embarrassed to detail all the ways in which my own high school teachers were ignorant. Which particular one of the links supports your premise? It looks irrelevant to your complaint about "Many US maps" in general or "US maps on TV." Number 2 is similarly just "Wired science," which few have ever heard of. Number 3 is also an obscure site. Collapse this thread. Edison (talk) 03:53, 23 September 2011 (UTC)[reply]
Alaska and Hawaii are the 47th and 40th US states in order of population size. How often do you here about things happening in the other states that small and smaller? (ie. Maine, New Hampshire, Rhode Island, Montana, Delaware, South Dakota, North Dakota, Vermont and Wyoming) --Tango (talk) 17:51, 22 September 2011 (UTC)[reply]
Combined, Alaska and Hawaii account for 0.67% of the US population and they are polar opposites in political matters. Add to the fact that they are more then 1,000 miles away from the rest of the US, and the results should not be surprising. I mean, how often do you hear about the Kaliningrad Oblast on major Russian news? Googlemeister (talk) 18:23, 22 September 2011 (UTC)[reply]
I can hardly think you can consider them "forgotten states." Everyone knows that they're there, and what they're about. (Hawaii: small; islands; beautiful; surfing. Alaska: gigantic; wilderness; cold; bears.) Every American schoolchild can find them on maps if they're there. Contrast that with West Virginia, South Dakota, Montana, or Wyoming. I suspect most American adults can't identify Wyoming on a map. Which capital city do you think is better well known, Honolulu or Cheyenne? It's true that Hawaii and Alaska get bumped from representations of the contiguous US, and are not considered to be quite as "normal" as the contiguous US (they aren't really manifest destiny states, they're extras that were acquired by the by), but they are far, far more represented in American culture and politics than many other states. --Mr.98 (talk) 19:34, 22 September 2011 (UTC)[reply]
If you go by the weather maps that appear on the typical TV news program here in the lower 48, all weather systems miraculously disappear once they hit the Canadian boarder. This striking meteorological phenomenon apparently works the other way on TV weather maps in Canada... According to CBC, the US never has any weather. Blueboar (talk) 19:41, 22 September 2011 (UTC)[reply]
A similar phenomenon exists in the UK. Northern Ireland has weather, but the republic to the south never does. --Tango (talk) 19:59, 22 September 2011 (UTC)[reply]
Yes, and this is particularly annoying in places where the weather comes from Canada, like in Buffalo, NY. StuRat (talk) 20:44, 22 September 2011 (UTC)[reply]
I think both states get news coverage. Alaska for Sarah Palin and oil related issues, not to mention the "bridge to nowhere", while Hawaii is often in the news for Obama's time growing up there, and Congressional junkets there at taxpayer expense. StuRat (talk) 20:50, 22 September 2011 (UTC)[reply]

For those who pay any attention at all, the fact that Alaska and Hawaii do get mentioned separately from the continental USA all the time means that more people do at least know of their existence, and roughly where they are, than the number who would know of those earlier mentioned little states. Exactly where IS Wyoming? HiLo48 (talk) 08:21, 23 September 2011 (UTC)[reply]

A quick check searching some states on Google News:

Connecticut returns "About 23,800 results"
Oregon returns "About 20,100 results"
Utah returns "About 19,900 results"
Nebraska returns "About 17,600 results"
Alaska returns "About 17,100 results"
"West Virginia" returns "About 16,700 results"
Hawaii returns "About 16,100 results"
"New Hampshire" returns "About 15,600 results"
Wyoming returns "About 12,700 results"
Vermont returns "About 12,600 results"
"Rhode Island" returns "About 8,960 results"
"North Dakota" returns "About 7,800 results"
"South Dakota" returns "About 6,910 results"

--Colapeninsula (talk) 09:54, 23 September 2011 (UTC)[reply]

Was Feudalism bad?

There is this atheist show (I'm an atheist myself), and in it, a caller asked the hosts what they thought about Buddhism. They didn't really have much to say about it, but one host said that in the past there were Buddhist feudal lords living in Tibet. It got me thinking, well... so what? What's wrong with that? ScienceApe (talk) 19:29, 22 September 2011 (UTC)[reply]

Well, there were various kinds of feudalism. Western European is the best known, and Japanese feudalism was apparently very similar. Tibetan society is another matter, and there's less research into it, at least less that is available in English. One thing that is common to most societies that can be called feudal is that there was a vast gap between the powerful elite and the majority. The elite, and these would be the Buddhist lords in this case, did little or no work, while the majority of people toiled long hours in agriculture. Itsmejudith (talk) 19:44, 22 September 2011 (UTC)[reply]
Well, serfdom is a form of slavery, for one thing. That's usually the main complaint about feudalism. What that has to do with Buddhism, I don't know. Maybe they were trying to point out that Buddhism is not always about peace, love, and the Yin-Yang? I don't know. It's not a very compelling argument. --Mr.98 (talk) 19:39, 22 September 2011 (UTC)[reply]
That's what I was thinking. Most Japanese are Buddhist, but I don't think that had anything to do with the imperialistic nature they had before and during WW2. ScienceApe (talk) 19:52, 22 September 2011 (UTC)[reply]
While there are lots of Buddhist Japanese, I believe Shinto is both the state religion and the imperial religion. StuRat (talk) 20:23, 22 September 2011 (UTC)[reply]
It seems that they are closely tied together. "The vast majority of Japanese people who take part in Shinto rituals also practice Buddhist ancestor worship. However, unlike many monotheistic religious practices, Shinto and Buddhism typically do not require professing faith to be a believer or a practitioner, and as such it is difficult to query for exact figures based on self-identification of belief within Japan. Due to the syncretic nature of Shinto and Buddhism, most "life" events are handled by Shinto and "death" or "afterlife" events are handled by Buddhism—for example". But again I don't fault Shintoism for their imperialistic nature either, I don't think it had anything to do with it. ScienceApe (talk) 20:35, 22 September 2011 (UTC)[reply]
"Bad" is hard to give a definite answer to. It was a reasonably efficient way to organize an agricultural society, though the people at the bottom were basically slaves, they at least nominally chose to swear an oath of fealty rather than simply being owned (assuming it worked like the European system). The amount of coercion involved probably invalidated any "choice." If the rulers chose that form of government because they wanted to exploit the people or just didn't care would be ethically problematic, if they simply weren't aware of a better way to do it or were just following tradition it's more complicated. They might have believed it was their moral obligation to rule in that fashion. I certainly wouldn't want to live that way, and it was "bad" in the sense that it was a pretty lousy way of life, but from an ethical standpoint it's more difficult to condemn. SDY (talk) 19:47, 22 September 2011 (UTC)[reply]
Agreeing with SDY, "bad" is very hard to define in any universal sense. I don't know what the show you were listening to was, but my guess was that the atheist was a secular humanist, while most secular humanists do not accept absolute morality, have an ethical view that humans have an intrinsic right to shape their own life. Personally I think this is more likely an out growth contemporary moral standards rather than a universal idea, but that is another story. Given the view point that people have a right to shape their own lives, feudalism comes across pretty bad, as has been said, it is near slavery for much of the population, very inequitable. From my point of view feudalism isn't really "bad," it is just the form of social relations that tends to emerge from a decentralized agrarian society. It happened all over the world independently and is actually pretty efficient at growing cereals without much central authority. Still it would suck to be a serf. --Daniel 20:21, 22 September 2011 (UTC)[reply]
Feudalism in its ideal form wasn't "bad" at all... in some ways it was quite altruistic. At the bottom of the pyramid there was a social contract between peasants and armed knights for mutual support in a dangerous and uncertain barter based economic world. The peasants provided goods (primarily food) in exchange for protection from invaders (whether rampaging barbarians, or rival lords). The knights and lords had responsibilities towards their peasants and vise-verse. As you move higher up the idealized pyramid, feudal society consisted of a network of mutual obligations between various lords and vassals ... the senior nobles (such as the king) offering land to live on, good laws and even handed justice in exchange for armed service and other defined duties from their vassals. Unfortunately, human nature being what it is, Feudalism rarely (if ever) existed in its true idealized form. Greed has a way of undoing most social contracts. Blueboar (talk) 20:29, 22 September 2011 (UTC)[reply]
As for the atheist show, you can find it here. http://www.youtube.com/watch?v=2pMOKj3hvY4&feature=channel_video_title at the 37:40 mark approximately. ScienceApe (talk) 20:39, 22 September 2011 (UTC)[reply]
I've read some pretty horrendous things about girl serfs in pre-China Tibet. Imagine Reason (talk) 02:13, 23 September 2011 (UTC)[reply]
OK, Buddhism and Shinto are not related in any way whatsoever. Going by what the simple people do is not a way to understand any religion. The Japanese will go to the buddhist temples to 'revere their ancestors', and go to a Shinto shrine to celebrate a new house being built. The same people will go to a fake chapel to have a 'wedding' done by an english teacher who is just there to get some part-time cash-in-hand (usually 10,000 JPY). Do not ever judge a religion or philosophy by the people that are supposedly following it, as you will be wrong in 100% of cases. KägeTorä - (影虎) (TALK) 03:23, 23 September 2011 (UTC)[reply]
This seems to contradict what you're saying, http://en.wikipedia.org/wiki/Shinto#Kokugaku ScienceApe (talk) 05:12, 23 September 2011 (UTC)[reply]
Shintoism is not Buddhism, which has no official god nor gods, in the same way that Christianity is not Judaism nor Islam. Willminator (talk) 17:49, 23 September 2011 (UTC)[reply]
I didn't say they were the same. ScienceApe (talk) 18:43, 23 September 2011 (UTC)[reply]

In this article it says Opposing him were the three Carthaginian generals (Hasdrubal Barca, Mago and Hasdrubal Gisco), who were on bad terms with each other, geographically scattered (Hasdrubal Barca in central Spain, Mago near Gibraltar and Hasdrubal near the mouth of the Tagus river), and at least 10 days away from New Carthage. However in Livy 26.44 it reads: When Mago, the Carthaginian commander, saw that an attack was being prepared both by land and sea, he made the following disposition of his forces. From Livy's description it sounds to me as if Mago is in the city of New Carthage defending it - not some 10 miles away! Am I misinterpreting one description or the other? Was Mago Barca at New Carthage or 10 miles away near Gibraltar?--Doug Coldwell talk 19:36, 22 September 2011 (UTC)[reply]

In Livy 26.46 it goes on to say: From this point Scipio saw the enemy retreating in two directions; one body was making for a hill to the east of the city, which was being held by a detachment of 500 men; the others were going to the citadel where Mago, together with the men who had been driven from the walls, had taken refuge.--Doug Coldwell talk 20:01, 22 September 2011 (UTC)[reply]

The article cites a book as its source, but doesn't quote from it. I can't tell if the material is correct or misquoted. It could be that the information is not necessarily incorrect. Mago could have returned to New Carthage with news of the attack. A horse traveling at 10 miles per hour would have gotten him there in an hour. It is possible to see advancing ships from a number of miles off from a lookout tower. I'm not sure if other sources exist, I am only aware of Livy. The historian may be dealing with concrete facts or some embellishments. Gx872op (talk) 20:20, 23 September 2011 (UTC)[reply]

Ancient Egyptian religions

The religion of the pharaohs seemed to offer very little to the common people. At best, they could hope to be killed when the pharaoh died and be entombed with him to continue to serve him in the afterlife. Was this the reason for the formation of the Sun-worshiping religion ? Was it more egalitarian in the afterlife ? StuRat (talk) 21:53, 22 September 2011 (UTC)[reply]

The Weighing of the Heart by Anubis
Actually that's the hollywood perception of the Ancient Egyptian religion. Originally only pharaohs had access to immortality, sure, but it slowly grew to include commoners as well. By the New Kingdom, Ancient Egyptian religion was already quite egalitarian. Very similar to Greek religions. Everyone were mummified after death though understandably with less fanfare than pharaohs. The ruler of the afterworld was also not the pharaohs nor the sun god, but Osiris. The father of the purported father of the original pharaoh, and the god of rebirth.
Everyone, even pharaohs had to pass the Weighing of the Heart by Anubis. Having their heart weighed in a balance against the feather of truth and justice (a manifestation of the goddess Maat). If they don't pass, their souls get eaten by the demoness Ammit. If they pass, they go on to the afterlife and go boating with Osiris happily ever after LOL, provided that they remember their names and had their body preserved. See Book of the Dead, for a description of the journey of the souls (Egyptians believed in two souls - an earthly one Ba and a heavenly one Ka, all tied to your body and your name).
The very brief monotheistic period in Ancient Egypt was not the result of people converting willingly either. It was forced upon them by Akhenaten who also tried to erase the old religions in the process, completely replacing it with the worship of only one god - the Aten (sun disk), very reminiscent of later Abrahamic religions. Note that while there was already a sun god in Egyptian religions - Ra, he was not equivalent to the sun disk, though Akhenaten originally claimed it so. The Aten was merely the sun itself. It's now known as Atenism and it was quite unpopular
In fact, after his death, his son Tutankhaten was immediately pressured by priests to revert to the old religions. He also changed his name in the process, changing allegiance from Aten to the god of the most powerful cult before his father took charge - that of Amun. He became the boy pharaoh we now famously know as Tutankhamun. All traces of Akhenaten and the worship of Aten was subsequently deliberately erased when Tutankhamun died and was replaced by Horemheb. -- Obsidin Soul 23:06, 22 September 2011 (UTC)[reply]
One of the major questions about Egyptian religion is what ordinary people thought about it and how much their beliefs and practices fit in with the "official" religion, which was overseen by the pharaoh and run by the priests. Modern understanding of those issues is severely hampered by the lack of written evidence (the vast majority of Egyptians were illiterate and didn't write down what they were doing or believing). Even so, most Egyptians seem to have believed that the gods were involved in their daily lives and could be called upon for help. And even though official rituals were closed to the general populace, that populace does seem to have regarded temples as particularly holy places, where they could contact the gods indirectly. See Ancient Egyptian religion#Popular religion and Egyptian temple#Popular worship.
In the afterlife, the Egyptians had all the beliefs Obsidian Soul points out, and more. Atenism, in contrast, doesn't seem to have said anything substantial about the afterlife. Many Egyptologists have said that Atenism's lack of detailed mythology or afterlife beliefs—the colorful elements that made traditional Egyptian religion accessible to ordinary people—was a major factor in its rejection by ordinary Egyptians. If anything, it was more pharaoh-centric than the traditional religion: Atenist writings are mostly about the Aten and its relationship with Akhenaten. Some Egyptologists (e.g. Jan Assmann, Jacobus van Dijk) argue that Akhenaten was attempting to reassert the religious importance of the pharaoh in the face of a growing trend toward direct, personal worship of the gods. A. Parrot (talk) 20:41, 23 September 2011 (UTC)[reply]

September 23

Chardin's students

Fragonard apparently studied in the atelier of the great luminist Chardin, and google gave me Thomas Germain Joseph Duvivier too (No article on any Wikipedia. He's the brother of Pierre-Simon-Benjamin Duvivier, article on French WP only. I found references describing T.G.J. Duvivier as an imitator of Chardin's art). Does anyone have any reference to other painters who studied with Chardin, besides the two I named? ---Sluzzelin talk 00:35, 23 September 2011 (UTC)[reply]

Our page on Per Krafft the Elder says he was a student of Chardin among others. --Antiquary (talk) 19:18, 23 September 2011 (UTC)[reply]

Use of Last Post to signal armistice?

When I was in elementary and middle school, we were taught that the Last Post had been played after the 1918 armistice to deliver the ceasefire order to the soldiers. Was this true? Do any armies still use music to propagate a ceasefire order? NeonMerlin 03:51, 23 September 2011 (UTC)[reply]

No. Written or verbal orders from on high propagate to armies and down to units. Isn't "Last Post" similar in its meaning to "Taps?" It would just be a salute to some soldier having died, and that is hardly a general order to cease hostilities. The noodling on a bugle would just be an epiphenomenon of the end of slaughter in a war. Edison (talk) 04:01, 23 September 2011 (UTC)[reply]
Yes, Last post is similar to taps - there would have been a seperate bugle call for "cease fire" but I very much doubt if that was used by that stage of the war. On Remembrance Sunday, the last post is used to mark the end of the two minutes' silence, which may have given rise to this story. I have seen (on TV) a typed order that was passed to a front-line artillery battery ordering the end of hostilities. Alansplodge (talk) 08:29, 23 September 2011 (UTC)[reply]
An eyewitness account by a US officer here doesn't mention bugles, but this account of the French Army does. Alansplodge (talk) 10:12, 23 September 2011 (UTC)[reply]
It is certainly possible that a bugler somewhere along the front blew "Last Post" as soon as the ceasefire started, but not as any sort of "signal" to ceasefire. Like "Taps", "Last Post" is blown at military funerals and memorials to honor the dead... so it is more likely that someone (possibly the bugler himself) decided it would be appropriate to honor his fallen comrades, now that the fighting had stopped. Blueboar (talk) 13:08, 23 September 2011 (UTC)[reply]

Those unfamiliar with the Last Post can hear it here. Alansplodge (talk) 21:24, 23 September 2011 (UTC)[reply]

Special Operations Executive

Why does the above article link to Plaka in Athens, when the reference is to an historic agreement between EAMS (Aris Velihoutis) and EDES (Napoleon Zervas)under the auspices of the British (Christopher Woodhouse), at a small town near the Plaka bridge in Epirus? I believe the reference is wrong (my father was at the meeting). — Preceding unsigned comment added by CecilEdwardBarnes (talkcontribs) 06:21, 23 September 2011 (UTC)[reply]

If you have good grounds to believe an article is wrong, change it - especially if you can find a published reliable source to reference. --ColinFine (talk) 17:54, 23 September 2011 (UTC)[reply]

Sabbath navigating

Are there any methods of aerial navigation that a Jewish navigator can use on Sabbath without breaking the halakhic laws? (Note that I'm asking about navigating an aircraft, not piloting one: I know that a Jewish pilot can't even start the engines on a Saturday without breaking the ban on lighting fires, unless this is for a rescue mission.) I'm guessing that map reading is prob'ly OK, but celestial navigation is out of the question; is this correct? What about other methods, like inertial or radio navigation? Thanks in advance! 67.169.177.176 (talk) 06:34, 23 September 2011 (UTC)[reply]

Anything that activates electricity through your own action is out of the question (so no radio). You can get a goy to do it if you agree before shabbat to have him do so and he also benefits (by not crashing let's say). Idk about using the stars, but I guess that map-reading would be okay. Bare-in-mind I am reform though, so I don't really know. :p Sir William Matthew Flinders Petrie | Say Shalom! 24 Elul 5771 06:42, 23 September 2011 (UTC)[reply]
Will the pilot and co-pilot be completely lost without navigation and end up flying blindly until they run out of fuel and crash? If so, then a Jew need not worry if their actions break Sabbath rules, so long as lives are saved. Other than that, I'm not sure. I'm curious, though - why do you think celestial navigation would be unacceptable? Avicennasis @ 07:10, 24 Elul 5771 / 07:10, 23 September 2011 (UTC)[reply]
Because it requires complex mathematical calculations, so would violate the ban on "making the finishing touch"? 67.169.177.176 (talk) 20:30, 23 September 2011 (UTC)[reply]
Ah right, I totally forgot about breaking shabbat rules in the event of life-threatening circumstances. Sir William Matthew Flinders Petrie | Say Shalom! 24 Elul 5771 07:13, 23 September 2011 (UTC)[reply]

The navigator wouldn't be able to board the plane on the Sabbath, so would be no help whatsoever. --Dweller (talk) 13:38, 23 September 2011 (UTC)[reply]

Sabbath lighting

I am reminded of a question my friends and I have been asking ourselves for the past 15 years, when we bought a disused synagogue. As well as the usual electric lighting, gas lighting had been installed throughout the large extension to the building that was built in the 1960s (5723 to be exact). We speculated that this was done to somehow enable the staff to get round the shabbat rules. I've read electricity on Shabbat in Jewish law and understand that "turning on an incandescent light bulb violates the Biblical prohibition against igniting a fire" so surely turning on a gas lamp is just the same. Why might gas lighting have been installed in a synagogue in the 1960s?--Shantavira|feed me 08:10, 23 September 2011 (UTC)[reply]

I have no definitive answer, but the following possibility occurs to me. In order to have the synagogue lit on Shabbat without violating the ignition prohibition, one would have to have turned on the lights before the beginning of Shabbat. Since electric lighting is either on or off, this would wastefully require electric lights to be (full) on for a long time (as dimmer switches, even if then available, would not actually reduce the power consumption appreciably). However, regulatable gas lighting, once lit, could presumably be left turned down to a minimal level, and turned up only as needed, without violating the prohibition. {The poster formerly known as 87.81.230.195} 90.197.66.228 (talk) 10:52, 23 September 2011 (UTC)[reply]
In mainstream Orthodox Judaism there would be no advantage, as a Jew can turn off/down neither electric nor gas lighting on the Sabbath. Maybe gas was cheaper then? Maybe the synagogue chairman had shares in a gas company? Maybe the building's electricity supply would have needed an expensive overhaul to power the additional structure? Maybe the more romantic lighting suited a function hall? Maybe lots of things, but I see no benefit in halachik issues. --Dweller (talk) 13:36, 23 September 2011 (UTC)[reply]
I don't know that much about halachik issues but it strikes me that a minor advantage is likely to be that the gas lighting would generally last without problem for the entire shabbat provided there are no gas supply issues. And if there is a problem you should probably do something about it because of the potential risk to life. Incandescent lights can die at any time and it's difficult to know when they're going to die. This sounds like only a minor risk but I guess lights do die and you have a slightly over 1/7 chance of it happening during the shabbat everything else being equal (which it probably isn't, incandescent lights tend to die when they are turned on and off but may also die when on for a long time). Also I don't know what the lifespan of 1960 lights were like. BTW, my understanding of the original message is there was electric lighting (I guess functional and complete) in addition to the gas lighting so I dunno about the 'expensive overhaul' suggestion. However it may be possible the electric lighting was installed after the gas lighting was already there. Nil Einne (talk) 15:12, 23 September 2011 (UTC)[reply]
The problem with bulbs dying is catered for in most synagogues by having an abundance of lights. There's an idea of having more light than you need on the Sabbath (conversely, the lighting is dimmed on Tisha B'Av) so most synagogues can weather multiple bulb failures with ease. --Dweller (talk) 15:22, 23 September 2011 (UTC)[reply]
Well, there's another possible answer: redundancy, so that if either the electricity or the gas supply were to be interrupted, there would still be sufficient light. {The poster formerly known as 87.81.230.195} 90.197.66.228 (talk) 21:51, 23 September 2011 (UTC)[reply]

Heir to the Irish throne

Were the monarchy ever to be restored in Ireland who would be the heir to the throne based on the direct descendancy from Brian Boru? I once read it was a member of the Kavanagh family. Thank you.--Jeanne Boleyn (talk) 08:23, 23 September 2011 (UTC)[reply]

This article claims that "Brian Boru is the ancestor of every English monarch from 1399 to the present. Almost certainly, more people in England can prove descent from Brian Boru than people in Ireland." So the legitimate heir to the Irish throne is... Queen Elizabeth II. Ho hum. --TammyMoet (talk) 09:30, 23 September 2011 (UTC)[reply]
Although due to various removals of English/British monarchs, some of these guys would disagree with Elizabeth II's claim: Pretender#English, Irish, Scottish, Welsh and British pretenders. Franz, Duke of Bavaria is apparently first in line through James VII and II. --Colapeninsula (talk) 10:02, 23 September 2011 (UTC)[reply]
(ec) I don't follow your reasoning, Tammy. Boru is an ancestor of probably millions of people, including yours truly, according to family tradition. Why should whoever happens to be the current monarch of the UK, the succession to which throne has nothing whatsoever to do with descent from Brian Boru, automatically be Boru's heir? -- Jack of Oz [your turn] 10:03, 23 September 2011 (UTC)[reply]
He appears to be an ancestor of mine too, as is Edward III. The logic is contained in the first sentence I quoted - what's your point? --TammyMoet (talk) 11:55, 23 September 2011 (UTC)[reply]
His point, Tammy, is that the rules of succession to the throne of Ireland are not necessarily the same as the ones to UK, and therefore even if at some point there were intermarriage, and one of the descendents became King in England, it doesn't mean the current UK monarch is suitable as per the rules recognised in Ireland. --Lgriot (talk) 12:39, 23 September 2011 (UTC)[reply]
Exactly. -- Jack of Oz [your turn] 20:21, 23 September 2011 (UTC)[reply]
We have Pretender#Ireland, but it's a bit of a mess (both the situation and the article). Grandiose (me, talk, contribs) 09:36, 23 September 2011 (UTC)[reply]
See also Descent of Elizabeth II from William I#The descent from the Irish kings. Although I have the utmost respect for EIIR, the historical claim of descent from Brian Boru is a bit weak, since it relies on descent through the female line at a couple of points, which I understand was not allowed in the succession of the High Kings of Ireland. However, you can still see the Harp of Brian Boru on the Royal Standard of the United Kingdom. Alansplodge (talk) 12:37, 23 September 2011 (UTC)[reply]
I am already aware Elizabeth II is a direct descendant of Brian Boru through the marraige of Aiofe of Leinster and Strongbow. Many noble families in medieval England descended from them as a result of the Marshal sisters' fecundity. I was thinking of Aiofe's brother Donal Kavanagh whose descendants would surely have precedence over those of Aiofe?--Jeanne Boleyn (talk) 13:03, 23 September 2011 (UTC)[reply]
What were the rules on royal inheritance in Ireland? Did the title passed through the female line, or was it restricted to male primogeniture? Was it semi-elected (ie open to which ever member of the royal family could gain support of the nobility and clan chiefs)? Blueboar (talk) 13:34, 23 September 2011 (UTC)[reply]
The articles says "A king would not be succeeded by his own son but would normally be succeeded by his mother's other sons; then by his sisters' sons; then his maternal aunt's sons; and so on, traveling through the female line of the royal house. This combination of male succession through matrilineal descent produced a cumbersome system under which the throne passed cyclically from brother to brother, then uncle to nephew, and then cousin to cousin, before starting over as brother to brother, uncle to nephew, etc" Grandiose (me, talk, contribs) 15:11, 23 September 2011 (UTC)[reply]
Of course, the high king of all Ireland was traditionally elected from amongst the ruling families of the different sub-kingdoms, or at least chosen based on who was the most threatening or generous of the candidates (who were themselves often elected from within each family), so surely there would not be a clear direct heir under this system, and if the Irish were to change the rules by which their high king was chosen, the question then becomes, what system would they use instead. 148.197.81.179 (talk) 16:07, 23 September 2011 (UTC)[reply]
The last claimant to have realistically ruled as the High King of Ireland was Edward Bruce, younger brother of Robert the Bruce. Afterwards, there existed two systems of nobility within Ireland, the Gaelic lords who continued to maintain the old systems (which became somewhat increasingly irrelevent) and the English system, which began under the Lordship of Ireland and became the (English) Kingdom of Ireland. Arguably, the old Gaelic system became officially defunct after the Flight of the Earls. If you're looking to find a suitable "High King" (Gaelic), then you'd first need to round up the current pretenders of the thrones to the various petty Irish kingdoms and then have them elect one. If you're looking for the modern pretender to the English Kingdom of Ireland, that would be fairly unambiguously Queen Elizabeth II. --Jayron32 18:42, 23 September 2011 (UTC)[reply]

Congressional Net Worth

I would like to see a list of all Senators and Congressmen since 1960 containing net worth at entry and upon exit of public office — Preceding unsigned comment added by 69.15.73.34 (talk) 11:57, 23 September 2011 (UTC)[reply]

So would a lot of people... such a list would be fascinating. Unfortunately, sifting through tax records and compiling such information on Wikipedia would violate our WP:No original research policy. We would need someone else (you?) to do the research and publish it ... Wikipedia could then use that published research as the basis for an article. Blueboar (talk) 13:21, 23 September 2011 (UTC)[reply]
Wouldn't tax records only record income, rather then net worth since you are not taxed on what you own (except for real estate taxes I guess), but what you make? At least for people who have not been subject to the death tax? Googlemeister (talk) 13:27, 23 September 2011 (UTC)[reply]
True... taxes alone would not give net worth (thanks for the correction)... but that does not change the point I was making. The IP was requesting an article that we can not provide. The act of compiling the information needed to create such a list would violate our NOR policy... the information would have to be compiled and published elsewhere first. Blueboar (talk) 13:42, 23 September 2011 (UTC)[reply]
The OP never specified that they wanted this list to be on Wikipedia. They simply asked for a list. Dismas|(talk) 13:52, 23 September 2011 (UTC)[reply]
Such a list would be nearly impossible due to all the technicalities of what "net worth" means. Just to pick a dead Senator, consider Strom Thurmond. He had a lot of investments in a lot of different things. Some of his investments were in investment groups. Some were made publicly known. Some only became public after he died. So, you could easily spend a few years trying to calculate his net work when he died (and left office) and still not have a fully accurate number. Imagine trying to get some accurate value for everyone. -- kainaw 14:02, 23 September 2011 (UTC)[reply]
To give concrete numbers, when Thurmond died, his net worth was only $200,000. This was because, starting around 1989, when he net worth was $2 million, he set up a number of trust funds for his children, which are not considered part of his estate's net worth.[27] --Mr.98 (talk) 14:11, 23 September 2011 (UTC)[reply]
Not to be an ass, but $200,000 looks like an estimate, not a concrete number. I seriously doubt he quickly checked his accounting right before he died to ensure his net worth was concretely valued at a perfect multiple of $100,000. -- kainaw 14:39, 23 September 2011 (UTC)[reply]
It's obviously rounded. The important point here is that it's an order of magnitude different from what his "true" assets were. --Mr.98 (talk) 14:49, 23 September 2011 (UTC)[reply]

See this page for a description of what members of Congress have to disclose and what offices hold the records. -- Mwalcoff (talk) 22:23, 23 September 2011 (UTC)[reply]

faces

I have been struggling recently with drawing realistic faces and heads of people, particularly with trying to get all the features in the right place and the right shape. I am wondering if there is anywhere I can find pictures of different sorts of faces facing all different angles and with different expressions that I can practice copying from.148.197.81.179 (talk) 15:59, 23 September 2011 (UTC)[reply]

There are plenty of helpful websites out there. Try this for starters or just use a search engine to look for "how to draw faces".--Shantavira|feed me 16:04, 23 September 2011 (UTC)[reply]
When you get tired of that draw a stick man and watch him come to life.--Shantavira|feed me 16:29, 23 September 2011 (UTC)[reply]

Actually, I didn't ask for advice on learning to draw, I am only after pictures that I can study in detail to see exactly where the different features of different faces are in relation to each other, how large they are, what shape they are and so on, quite difficult to do standing in front of a mirror, and more so on real people passing by. Meanwhile, that site linked to doesn't seem that much help, given that the step by step advice seems to go pretty much 1-find a picture to copy from, 2-draw the eyes, 3-draw the nose, and so on, with little advice on how to draw each of those. 148.197.80.214 (talk) 17:05, 23 September 2011 (UTC)[reply]

If you wish to draw the human body, you need to do more than just observe passing people. Know your human anatomy! That means knowing the bones and the muscles underneath the skin. And yes that framework oval with a crosshair thing is not only applicable for cartoons. It's used for realistic drawings as well for gauging proportions and angle. So don't discount it either.
Anyway, the best resource I can recommend would probably be Drawing the Head and Hands by Andrew Loomis. It's a public domain book, and really a spectacular resource for any artist. They're dated of course, but the principles are applicable even today. His other books are available free online if you want to get more ([28] or [29]). -- Obsidin Soul 18:11, 23 September 2011 (UTC)[reply]

Neoclassical sex?

How are reproductive motivations treated in neoclassical economics? I'm looking for background information on how mating, procreation, nesting, and the like influence individuals' utility maximization and demands. Certainly these are important factors influencing large amounts of consumer demand, involving dating behavior, marriage and related expenses (and savings from cohabitation), conspicuous consumption, prostitution, and child rearing behaviors and expenses, but I can't find any treatment of utility maximization which explicitly addresses anything beyond food, clothing, shelter, and transportation. 69.171.160.21 (talk) 17:02, 23 September 2011 (UTC)[reply]

The classic analysis is Malthus' "preventive checks" (in Britain during the Renaissance or early modern period, working-class people married later if they couldn't afford to marry earlier, so that economic factors could exert a limiting effect on the rate of population growth). There's no Wikipedia article on preventive check or preventive checks, but it seems like there should be. Anyway, the modern theory of sexual selection has a more comprehensive type of analysis which could be called quasi-economic in some aspects... AnonMoos (talk) 18:10, 23 September 2011 (UTC)[reply]
Thanks, that helped me find the material I need for my assignment. 69.171.160.21 (talk) 20:57, 23 September 2011 (UTC)[reply]

X-rated constellations ?

Since most of our constellations come from the Greeks and Romans, who were also known for their erotic art, I'm curious as to the lack of erotic imagery in constellations. Were they censored at some point ? (BTW, I think this is a better place for this Q than the Science Desk, since, while stars are natural phenomena, constellations are entirely from man's imagination.) StuRat (talk) 18:02, 23 September 2011 (UTC)[reply]

I don't know, there's certainly sexual undertones in the Andromeda story. There are lots of overtly sexual parts to Greek stories; I don't know that there are any overtly pornographic constellations per se, insofar as the constellations themselves represent the characters in these stories then one could consider the erotic parts of the stories can't be ignored when considering the constellations. --Jayron32 18:12, 23 September 2011 (UTC)[reply]
As a bit more, you may be interested in digging up an english translation of the Catasterismi, which is basically the ancient text which tells the stories of the origin myths of the constellations. --Jayron32 18:21, 23 September 2011 (UTC)[reply]
Why did you introduce the concept of pornography, Jayron? Stu was asking about erotic imagery. -- Jack of Oz [your turn] 20:16, 23 September 2011 (UTC)[reply]
Actually, out of the current formally-recognized 88 constellations, some of the large and conspicuous ones were inherited from earlier middle-eastern etc. civilizations, while many other constellations (which are either small and lack bright stars, or are only visible from southern latitudes) were filled in centuries after the fall of the Roman empire. So I don't know that there's a dominant overall imprint of Greco-Roman culture... AnonMoos (talk) 18:25, 23 September 2011 (UTC)[reply]
Greek mytholgy held that the milkyway was created when Hera found Heracles biting her breast as he suckled from it as an infant. She pushed him away and the milk splattered across the galaxy forming the milkyway. — Preceding unsigned comment added by 24.38.31.81 (talk) 19:43, 23 September 2011 (UTC)[reply]

Palestinian statehood application

Does anyone know where I can find the full text of President Abbas' letter to the UN Secretary-General which was handed over today? ╟─TreasuryTagconstabulary─╢ 18:30, 23 September 2011 (UTC)[reply]

Would http://www.un.org/Docs/journal/En/lateste.pdf be what you're looking for? Rcsprinter (talk) 19:01, 23 September 2011 (UTC)[reply]
'Fraid not; that's a transcript of Abbas' speech to the General Assembly, not his written letter to Ban Ki-Moon. ╟─TreasuryTagSubsyndic General─╢ 19:06, 23 September 2011 (UTC)[reply]
The letter should be issued as a UN document within a day or so of receipt. At that point it should be listed on this page. It doesn't seem to be there yet, but it should turn up; and if not, it should appear in the Security Council official records (UN membership applications are referred to a Security Council committee of the whole). Newyorkbrad (talk) 19:12, 23 September 2011 (UTC)[reply]

College dating; sociology; hookups vs relationships

I am writing the article College dating. I am trying to determine why hookups are so much more common than relationships in college as compared to dating in high school and post-college. Using this study, http://www.ncbi.nlm.nih.gov/pmc/articles/PMC2932467/ it is clear that hookups are becoming very common. I used the article http://www.npr.org/templates/story/story.php?storyId=105008712 and decided that the main causes were lack of time and effort to commit to another person. I could not find any more information as to why this sudden boom might be happening. Thanks! BonnieNoel (talk) 19:49, 23 September 2011 (UTC)[reply]

Because young people like sex, and in college they have no supervision? --Jayron32 19:59, 23 September 2011 (UTC)[reply]
Note that if you are synthesizing articles to come to your own, unique conclusions, that is a definite violation of WP:NOR. If the theories are not in the literature you are citing, don't add them. --Mr.98 (talk) 20:14, 23 September 2011 (UTC)[reply]
Maybe I'm just an old bloke out of touch with modern language conventions, but can someone please give definitions of hookup and relationship, so that this topic makes sense? HiLo48 (talk) 20:39, 23 September 2011 (UTC)[reply]
Hookup = one-night stand. Relationship = steady dating (generally for a few months or more). Hope this helps. 67.169.177.176 (talk) 20:47, 23 September 2011 (UTC)[reply]
And this is hardly a current trend... Adam Bishop (talk) 20:50, 23 September 2011 (UTC)[reply]
"One night stand" I'm familiar with, but not "hookup". Maybe it's an American thing. (Which I'm not.) HiLo48 (talk) 21:53, 23 September 2011 (UTC)[reply]
That really surprises me, HiLo. Aren't you a teacher? Don't your young charges go on constantly about "hooking up"? Maybe you've taught them so well that they use only formal words and sentences.  :) I'm very familiar with this expression from just my general participation (such as it is) in the Australian community. -- Jack of Oz [your turn] 22:05, 23 September 2011 (UTC)[reply]
"Hooking up" used to mean simply "joining" or "meeting", as in "I'm hooking up with the gang after work." If my grandchildren were to say "I'm hooking up with so-and-so after school" I now understand they'd be more than a little surprised at my laid-back response. Bielle (talk) 22:23, 23 September 2011 (UTC)[reply]

Copyright Law

I recently downloaded an iPod application about Roman Emperors. Each page (copied from wikipedia btw without attribution) links to a series of pictures of busts/paintings etc of each emperor. I'm interested in developing such an app, but have no idea about copyright law in regards to reproducing images legally for such an application. If I were to develop such an app, how would I go about finding images that were either copyright-free, or images that I could (purchase?) gain the use of? Skinny87 (talk) 21:17, 23 September 2011 (UTC)[reply]

First off, before it's said: we can't offer you legal advice. We can't guarantee that anything you read on here will be fine in your legal jurisdiction (which we don't know). So don't take any of this as gospel.
That being said, the images on Wikimedia Commons, which are the source of most of the images you find in Wikipedia articles (but not all!), state their copyright restrictions (or lack thereof) very clearly. Any image you find on Wikipedia can be clicked on, and it will tell you whether it is copyrighted and what your rights are with regards to re-use. Wikipedia tries to be much clearer on this than the rest of the internet. If you are not experienced in judging the copyright status of things on the internet, you should probably assume that most of it is copyrighted. If you'd like to read a nice primer on thinking about copyrights, I recommend Lawrence Lessig's Free Culture, which is, incidentally, free. It's an interesting read and over the course of the book you'll get a much better understanding of what the relevant considerations are. --Mr.98 (talk) 20:58, 23 September 2011 (UTC)[reply]
Thanks, that looks like a good book to work through. I can see a lot of images on Commons that could be used in such an app, but even the ones that seem free to use are confusing. Take this one. The CC notice states that you are free to share – to copy, distribute and transmit the work and refers to You must attribute the work in the manner specified by the author or licensor. But this brings up several questions. Firstly, what are the manners specified by the licensor, the Bundesarchive? And if this were used in an iPod application for which money is charged, is that a legal use, as money would be made off of the image, indirectly at least. I realize legal advice can't be given and I'm not really asking for it, IANAL and so forth, but where would I even go to get these questions answered?Skinny87 (talk) 21:17, 23 September 2011 (UTC)[reply]
In that particular case, those tags say what the attribution needs to be (in that case, "Bundesarchiv, Bild 101I-198-1383-39 / CC-BY-SA"). In cases where they don't specify it, you're safe listing the name of the author. (You also have to list the license, like CC-BY-SA, because of the ShareAlike requirements). You are allowed to make money off of any images on Commons as long as you do everything that is said in the requirements. You don't gain the copyright yourself — you must re-license the image according to the same requirements — so you couldn't sue someone for reusing your version of things. But you can make money off of it. Does that clarify things? These sorts of general questions about licenses don't fall under legal advice, and the CC licenses are meant to be interpreted by people without legal skills, so I think we can help you if you have further questions on those.
You're absolutely correct to think that copyright is a complicated mess of things that someone "outside" of shouldn't try to reason their way through without some guidance. I heavily recommend the Lessig book as a primer to wrapping your brain around the subject — it's one of the things I used when learning to wrap mine around it, anyway. Lessig is also one of the creators of Creative Commons and the CC licenses, so he covers that sort of stuff in the book pretty well. There is a logic to copyright, but it takes a little indoctrination to really grok it. --Mr.98 (talk) 22:36, 23 September 2011 (UTC)[reply]

Prison account

On the Desmond Keith Carter article, it says that:

In July 1993, Carter was sentenced to death for first degree murder and to 40 years imprisonment for robbery with a dangerous weapon. On the eve of his execution, Carter declined a special meal. Instead, he bought two cheeseburgers, a steak sub, and two Cokes from the prison canteen, for which he paid $4.20 from his prison account. He was executed by means of lethal injection on December 10, 2002.

Is this "prison account" phenomenon wide-spread? Is it notable enough to warrant an article? If so then I might start one when I have some free time. — Preceding unsigned comment added by Anonymous.translator (talkcontribs) 21:36, 23 September 2011 (UTC)[reply]

In American prisons (perhaps others, I don't know), prisoners have accounts with prison commissaries that carry basic goods. The accounts are usually filled by people outside of the prison. The commissary article describes it pretty well. I doubt having a separate prison account article is necessary; it seems like something that could just be redirected to the commissary article. --Mr.98 (talk) 22:44, 23 September 2011 (UTC)[reply]

unanswered questions

why is all images of the Lord and savior Jesus Christ depicts him as apolloyon and as being white if he wasn't a gentile? — Preceding unsigned comment added by 71.197.38.64 (talk) 21:49, 23 September 2011 (UTC)[reply]

All?... hardly. This website has many images of Jesus depicted as being non-white. The reason why Jesus has so often been depicted as being white is due to the fact that the common cultural image is based on European religious art. Through most of history Christianity was primarily confined to Europe ... and Europeans painted and sculpted what they were familiar with - ie European features (it's not confined to depictions of Jesus either... Medieval and Renaissance art depicted all biblical characters as being European.) And before you cry "racism"... The same phenomenon can be seen with an Afro-centric take when looking at medieval Ethiopian religious art works, which depicted biblical characters (including Jesus) as having African features. Blueboar (talk) 22:17, 23 September 2011 (UTC)[reply]
I don't know that I'd agree Christianity has been most a Eurpoean thing, although it's true that most of Europe has been Christian for much of the time Christianity has existed, and it is certainly true that most Christian art seen in the Western world comes out of the Western, European tradition. So it's certainly true that the 'white Jesus' effect is from the art being painting as part of a white European tradition, I just don't think it's true to say that Christianity was mostly confined to Europe throughout most of history. Much of Europe was relatively late in joining Christendom, compared to things like the Ethiopian Church and the Syrian Church (the Armenian Church and Constantinople I can see someone arguing are Europe): they just don't tend to feature greatly in our history and art lessons over here. 212.183.128.45 (talk) 23:00, 23 September 2011 (UTC)[reply]
(Edit Conflict) Your assertion that that all images of Jesus are such is simply not true: see the article Depiction of Jesus and in particular its section Range_of_depictions. In Western European art there was a common and long-lasting convention of depicting historical figures with the racial features, clothing styles and surroundings of the intended contemporary viewers rather than authentically. This may sometimes have been due to lack of imagination or knowledge on the part of the artist, sometimes to cater to the (paying) customers' expectations, or sometimes to make a philosophical or theological point about the relevance of religious figures to the present day and the racially different beholders rather than the distant past and the subjects' original race and/or culture. {The poster formerly known as 87.81.230.195} 90.197.66.228 (talk) 22:20, 23 September 2011 (UTC)[reply]